Você está na página 1de 172

PROGRAMA NACIONAL

OLIMPADAS

EXAMES E RESULTADOS
XVII Olimpada
Norte/Nordeste de Qumica

Olimpada
Brasileira de Qumica

16. Olimpada
Ibero-americana de Qumica

43rd International
Chemistry Olympiad

Prof. Jesualdo Pereira Farias


Reitor da Universidade Federal do Cear
Prof. Luiz de Sousa Santos Jnior
Reitor da Universidade Federal do Piau
Prof. Tarcsio Haroldo Cavalcante Pequeno
Presidente da FUNCAP
Prof. Antnio Salvador da Rocha
Pro-Reitor de Extenso da UFC
Profa. Maria da Glria Carvalho Moura
Pr-Reitora de Extenso da UFPI
Newton Mario Battastini
Presidente da Associao Brasileira de Qumica
Prof. Srgio Maia Melo
Coordenador do Programa Nacional Olimpadas de Qumica
Prof. Jos Arimatia Dantas Lopes
Vice-coordenador do Programa Nacional Olimpadas de Qumica e
Coordenador da Equipe Pedaggica (docentes do Estado do Piau)
Prof. Antnio Carlos Pavo
Prof. Cristiano Marcelino Jnior
Prof. Carlos Eduardo G. da Silva
Preparao dos exames experimentais e edio de fitas de vdeo

ISSN: 1809-2012
Imprensa Universitria
Universidade Federal do Cear
Organizao de originais:
Prof. Srgio Melo
Capa:
Maherle
Editorao e Projeto Grfico:
Maherle/Srgio Melo
2011 Programa Nacional Olimpadas de Qumica
Lanamento em 25.11.2011 por ocasio da solenidade de encerramento dos eventos: XVII Olimpada Norte/
Nordeste de Qumica, IV Olimpada Brasileira de Qumica Jnior e Olimpada Brasileira de Qumica - 2011
Tiragem: 15.000 exemplares.

Distribuio gratuita.

ndice
1

Olimpadas Nacionais de Qumica


Calendrio das Atividades realizadas em 2011 . . . . . . . . . . . . . . . 4
Mensagem do Diretor do Instituto de Qumica da Universidade de So Paulo aos
participantes das Olimpadas de Qumica . . . . . . . . . . . . . . . . . . . . 5
OPINIO: O Ano Internacional da Qumica Carlos Vogt. . . . . . . . . . . . 7
XVII Olimpada Norte /Nordeste de Qumica - Exames 2011 . . . . . . . . . . . 9
XVII Olimpada Norte/Nordeste de Qumica - Relao dos aprovados. . . . . . 23
IV Olimpada Brasileira de Qumica Jnior Exames 2011- Fase I. . . . . . . . 31
IV Olimpada Brasileira de Qumica Jnior Exames 2011 - Fase II. . . . . . . . 38
IV Olimpada Brasileira de Qumica Jnior - Relao dos aprovados. . . . . . . . 46
Olimpada Brasileira de Qumica - 2011 Fase III modalidade A. . . . . . . . 49
Olimpada Brasileira de Qumica - 2011 Fase III modalidade B . . . . . . . . 55
OBQ - 2011 Fase III. Soluo dos problemas das Modalidades A e B . . . . . . 60
Olimpada Brasileira de Qumica - 2011 - Resultado e classificao. . . . . . 69

Processo seletivo para compor equipe nas olimpadas internacionais em 2011


Exame sobre conhecimentos de tcnicas de Laboratrio (OBQ-2010 Fase IV). . 74
Curso de Aprofundamento e Excelncia em Qumica (OBQ-2010 Fase V). . . . 76
Exame final para selecionar a equipe brasileira na IChO (OBQ-2010 Fase VI) . . . 76
16 Olimpada Ibero-americana de Qumica - Exame Prtico . . . . . . . . . . 82
16 Olimpada Ibero-americana de Qumica - Exame Terico . . . . . . . . . . 90
43rd International Chemistry Olympiad/2011 - Exame Prtico. . . . . . . . . . 98
43rd International Chemistry Olympiad - Exame Terico . . . . . . . . . . . 112
Destaques olmpicos - 2011 . . . . . . . . . . . . . . . . . . . . . . 120
Depoimentos . . . . . . . . . . . . . . . . . . . . . . 123
Consideraes finais - Prof. Srgio Melo . . . . . . . . . . . . . . . . 125
Endereos dos Coordenadores nos Estados participantes
do Programa Nacional Olimpadas de Qumica . . . . . . . . . . . . . . . . 128

Programa Nacional
Olimpadas de Qumica

Calendrio 2011
Data

Hora

Atividade

01/02/2011

14:00h

Fase IV da OBQ-2010 (Exame sobre tcnicas laboratoriais com o objetivo


de selecionar a equipe que representou o Brasil na Turquia e na OIAQ).

21/03 a 01/04/2011 Curso de Aprofundamento e Excelncia (OBQ -2010 Fase V) para os 15 estudantes selecionados no exame de conhecimentos de laboratrio. Foi ministrado pela UNICAMP.
Estudantes se deslocaram at Campinas.
16/04/2011

9:00h

Exames da Olimpada Brasileira de Qumica - 2010. Fase VI - Questes de


baseadas na lista sugerida pelos organizadores da 43rd IChO.

30/04/2011

Divulgao dos nomes dos quatro estudantes que representam o Brasil nas competies internacionais em 2011.

03 a 21/05/2011

Inscries para a XVII Olimpada Norte/Nordeste de Qumica - XVII ONNeQ. Quarenta


estudantes por estado. Inscries restritas aos coordenadores-estaduais.

04/06/2011

Exames da Olimpada Norte/Nordeste de Qumica - XVII ONNeQ.

03/07/2011

Divulgao de resultados da Olimpada Norte/Nordeste de Qumica - XVII ONNeQ.

06/06 a 26/08/2011 Inscries para a IV Olimpada Brasileira de Qumica Jnior. Escolas inscrevem seus alunos de 8o e 9o anos do ensino fundamental.
09 a 18/07/2011

43a Olimpada Internacional de Qumica, Ankara - Turquia .

01 a 15/08/2011

Inscries para a Olimpada Brasileira de Qumica - 2011. Vinte estudantes por estado
na modalidade A (estudantes da penltima srie do ensino mdio ou srie anterior),
vinte estudantes por estado na modalidade B (estudantes da ltima srie do ensino
mdio). Inscries reservadas aos coordenadores-estaduais.

27/08/2011

8:30h

Exames da IV Olimpada Brasileira de Qumica Jnior - OBQjr, para estudantes de 8o e 9o anos. (Fase I).

27/08/2011

14:00h

Exames da Olimpada Brasileira de Qumica - 2011 Fase III - Modalidades


A e B. Questes analtico-expositivas.

16 a 24/09/2011
01/10/2011

Exames da IV Olimpada Brasileira de Qumica Jnior, OBQjr. (Fase II).

30/10/2011

Divulgao dos resultados da OBQ-2011. A partir de 30.10.2011.

19 a 25/10/2011
30/10/2011
24 e 25/11/2011
4

16 Olimpada Ibero-americana de Qumica, Teresina - Piau.

Semana Nacional de Cincia e Tecnologia.


Divulgao dos resultados da OBQ-2011 e IV OBQjr. A partir de 30.10.2011.
8h30min Reunio do Conselho de coordenadores, em Fortaleza.
19:30h Solenidade de encerramento e premiao, em Fortaleza.

Ano Internacional de Qumica


Olimpada Brasileira
de Qumica 2011

Mensagem

Mensagem do Diretor do Instituto de Qumica da Universidade


de So Paulo aos participantes das Olimpadas de Qumica

anchete em vrios meios de comunicao,


todos ficamos sabendo que nosso planeta
atingiu a marca recorde de sete bilhes de
pessoas vivendo sobre sua superfcie. Com
uma taxa anual de crescimento de aproximadamente
1,1%, quando a gerao que hoje participa da Olimpada Brasileira de Qumica atingir a meia-idade, daqui a 30
trinta anos, no havendo grandes catstrofes mundiais,
prev-se que a populao vai estar se aproximando de
dez bilhes de pessoas. Nesse contexto, algum poderia
perguntar: o que a Qumica tem a ver com isso? Um pouco de exerccio mental vai nos dizer que a Qumica tem
muito a ver com esse aumento populacional. Como alimentar essa massa crescente
de pessoas, principalmente aquelas vivendo em regies pobres e inspitas? Como
tratar de sua sade e se precaver de grandes epidemias? Como abrig-las em ambientes apropriadamente climatizados e com um mnimo de infraestrutura habitacional? Como permitir sua locomoo de um lugar para outro? Como evitar que fontes energticas naturais se degradem e substncias essenciais ao desenvolvimento
no mundo moderno no venham a se exaurir? Como educar a populao para que
no venhamos a ter grandes catstrofes num futuro no muito distante? Poderamos
estender essa lista de perguntas com outras mais especficas, mas essas poucas bastam para podermos situar a Qumica como cincia central nesse contexto. Alimentar
grandes populaes implica desenvolver fertilizantes a baixo custo para que com
planejamento possamos obter do solo vrias colheitas anuais e alimentos mais saudveis e nutritivos; implica sabermos evitar pragas nas plantaes e nas criaes de
animais para corte atravs do uso ambientalmente correto de produtos qumicos. O
tratamento da sade implica aprimorar nosso conhecimento dos mecanismos bioqumicos no corpo humano identificando aqueles fatores desencadeadores de doenas e, consequentemente, a busca de medicamentos que possam contribuir para
sua cura. A educao bsica a respeito de cuidados higinicos pessoais e de ambientes comuns essencial, principalmente em regies de baixo ndice de alfabetizao
para se evitar contaminaes de rios e do ar e o surgimento de focos de doenas.
Habitao, transporte e o crescimento industrial implicam em grande consumo de
energia. Pensar novas fontes de energia renovveis e de baixo custo e em aparelhos
e veculos de menor consumo atividade de pesquisa intensa no mundo moderno.

Ano Internacional de Qumica



| 5

Programa Nacional
Olimpadas de Qumica

Mensagem
No Brasil, fomos pioneiros no desenvolvimento de motores a etanol. Agora, temos o
grande desafio de no s mantermos a produo de cana de acar num patamar
compatvel com o desenvolvimento do pas como tambm de usarmos esse mesmo
etanol na busca de rotas sintticas industriais alternativas quelas oriundas de derivados do petrleo. Nesse exerccio mental, restringindo o olhar ao nosso redor mais
prximo, vemos a qumica se manifestar nos alimentos e bebidas que ingerimos,
na vestimenta das pessoas, nos produtos de cuidados pessoais, nos livros e obras
de arte, nos equipamentos eletrnicos, no colorido da natureza e mesmo no nosso
humor. O jovem participante da Olimpada de Qumica, independentemente de sua
escolha de carreira profissional num futuro prximo, vai ter a oportunidade de constatar esse papel da Qumica como cincia central e de participar tambm como um
ator responsvel pela transformao sustentvel do planeta Terra. A todos, que essa
participao na Olimpada de Qumica possa contribuir no s para aferir conhecimento, e capacidade de raciocnio, mas que tambm oferea uma oportunidade de
autoconhecimento lembrando que disciplina, perseverana e humildade so ingredientes essenciais para o sucesso na vida.
Prof. Dr. Fernando Rei Ornelas (*)
Diretor do Instituto de Qumica da
Universidade de So Paulo
* Bacharel, Licenciado e Mestre em Fsico-Qumica pelo IQ-USP e Ph.D. em Chemical Physics pela Universidade de Indiana, E.U.A.

Segundo site da Aracruz Celulose, 1 rvore com idade de corte (acima de sete
anos), produz 20695 folhas de papel A4 (tais como estas usadas neste exemplar).
Logo, 38 rvores foram cortadas para satisfazer a edio destes Anais do Programa
Nacional Olimpadas de Qumica. Faa bom proveito, recicle o material aps uso
exaustivo. Conforme o Portal SOS Mata Atlntica, 38 rvores resgatam da atmosfera
trs toneladas de CO2 ao longo de cinco anos. Defenda o meio ambiente, engage-se
em campanhas ecolgicas de preservao ambiental.

Ano Internacional de Qumica


Olimpada Brasileira
de Qumica 2011

Opinio

O ano internacional da qumica


Carlos Vogt
Todos concordamos, ou ao menos tendemos a concordar, que a cincia contribui, de uma forma
ou de outra, para a melhoria da qualidade de vida no planeta, embora seja tambm verdade que a
desconfiana das populaes no tenha deixado de acompanhar o desenvolvimento cientfico e as
aplicaes do conhecimento na gerao das novas tecnologias e das inovaes que se incorporam
com frequncia cada vez maior ao cotidiano de nossas vidas.
Alm dos aspectos ligados ao bem-estar social que a cincia pode acarretar, na forma das facilidades que pode oferecer atravs de suas aplicaes tecnolgicas e inovativas, h outra espcie de conforto que diz respeito s relaes da sociedade com as tecnocincias, que envolve valores e atitudes,
hbitos e informaes, com o pressuposto de uma participao ativamente crtica dessa sociedade no
conjunto dessas relaes. A esse tipo de conforto chamo bem-estar cultural, como j tratei em outro
artigo (Cincia e bem-estar cultural, publicado no nmero 119 da revista eletrnicaComCincia).
A qumica, disciplina fundadora da cincia moderna, segue os preceitos acima mencionados, h
muito gerando conhecimentos que promovem o desenvolvimento cientfico e a melhoria da qualidade
de vida da populao, ao mesmo tempo em que vista, em algumas pocas, e em todas as pocas,
por alguns, com certa desconfiana, como se, por si s, a qumica pudesse oferecer algum risco ou se
configurasse como uma ameaa. No obstante os usos que so ou foram feitos da qumica e de seus
produtos cientficos e tecnolgicos, a Organizao das Naes Unidas (ONU) proclamou 2011 como
o Ano Internacional da Qumica, pelos inmeros benefcios da qumica para a humanidade, e com o
propsito de celebr-la no mundo todo.
No livro Dez teorias que comoveram o mundo, de Leonardo Moledo e Esteban Magnani, publicado no Brasil pela Editora da Unicamp, em 2009, em traduo do original argentino, de 2006, a qumica
reverenciada por meio de uma descoberta que no somente mudou os rumos da prpria qumica,
mas que representa tambm um dos marcos da cincia moderna. Entre as teorias escolhidas pelos
autores, como o heliocentrismo, a gravitao universal, o evolucionismo, a teoria atmica, a teoria
da infeco microbiana, a relatividade, a teoria da deriva continental, a gentica e o Big Bang, est
ainda elencada a teoria da combusto. Lavoisier, ao anunciar que na natureza nada se cria, nada se
perde, tudo se transforma, criando o enunciado do princpio de conservao da matria, resolveu um
importante problema para os pesquisadores da rea: a natureza da combusto. E, de quebra, afetou
os rumos, porque no dizer, da prpria humanidade, no sentido de transformao do conhecimento
acumulado at ento.
Com a sua descoberta, o qumico francs entra no paradoxo da comovente histria do conhecimento que, a meu ver, seria simples e transitria como definitiva e complexa a provisoriedade da
vida. Conhecer um ato de coragem que nos leva, de pergunta em pergunta ao confronto de alternativas: ou recusamos o conhecimento como dado, ou nos aventuramos no que nos dado a conhecer.
Neste caso, ainda que a biblioteca de nossos conhecimentos seja peridica, ela ser tambm ilimi-

Ano Internacional de Qumica



| 7

Programa Nacional
Olimpadas de Qumica

Opinio
tada como enunciou Borges sobre a Babel; no outro, seremos somente definitivos e limitados pelos
muros abertos do labirinto de areia do deserto de informaes.
H, assim, pelo menos dois modos de conhecer: aquele que nos abandona e nos perde na planitude da informao acumulada (termo elaborado no texto A inveno da planitude, publicado
no nmero 120 da revista ComCincia), tornando-nos sbios-sabidos; aquele que, mantendo-nos em
estado de ignorncia crtica o que chamei em outro artigo (Cincia e bem-estar cultural) de ignorncia cultural , nos leva a desconfiar da miragem benfazeja do conhecimento dado e nos pe
em constante estado de alerta para o que vem pronto, plano e amide, vale dizer, os monumentos
instantneos das certezas passageiras.
Neste caso, muito provvel que todos no sejamos sbios; certo, contudo, que teremos sabedoria a sabedoria paradoxal que quanto mais aumenta, mais nos faz crescer em conhecimento e mais
nos diminui o conforto passivo das situaes objetivas e subjetivas de cada conquista tica e cultural.
A Organizao das Naes Unidas (ONU) escolheu, com feliz acerto, marcar 2011 como o Ano
Internacional da Qumica, dado que ele tambm o ano do centenrio do Nobel que Marie Curie recebeu pela descoberta dos elementos qumicos rdio e polnio, depois de haver j recebido, com seu
marido Pierre Curie, em 1903, o Nobel de fsica por suas pesquisas no campo da radioatividade.
So muitas as publicaes e comemoraes neste ano desse modo singularizado pela ONU para
enfatizar os grandes avanos e conquistas da cincia atravs de uma de suas expresses mais sofisticadas e mais importantes no campo do conhecimento.
O prprio Labjor j esteve presente nessas homenagens pelas publicaes a ele ligadas, direta
ou indiretamente, como caso da edio, ano 63, n. 1, da revista Cincia & Cultura, dedicado ao tema
e da edio n.8, fev/2011, da revista Pr-univesp, onde, alis, parte deste texto foi tambm publicado
como editorial.
A importncia da qumica to grande para a histria do conhecimento e, portanto, para o bemestar cultural da humanidade, alm, claro de sua enorme relevncia para as transformaes sociais
de nossa histria, que, de algum modo, possvel afirmar que o pensamento antropolgico, caracterstico do homem moderno, no seria possvel sem a descoberta e as descobertas da qumica.
Com a qumica, firmam-se tambm a revoluo industrial e todas as consequncias econmicas,
polticas e culturais dela advindas no plano da organizao da vida social no mundo moderno.
Hoje, a qumica, seguindo a tendncia epistemolgica de agregao de reas do conhecimento
cientfico para a formao de novas reas com caractersticas multidisciplinares predominantes, evolui
para a constituio de campos do saber com os quais ela se encontra, por exemplo, a fsica, a biologia,
a farmacutica, a medicina, a gentica, a genmica, a protemica, e uma grande variedade de estudos
e pesquisas em nanocincias e nanotecnologias.
Nos cem anos do Nobel de qumica de Marie Curie, o primeiro ano da celebrao da dinmica
permanente de uma forma de conhecimento que se tornou definitiva nos processos culturais de permanncia e transformao do homem, da natureza e de suas relaes.

Carlos Vogt Diretor de Redao da Revista ComCincia,


publicao eletrnica da SBPC. Artigo publicado em 10/07/2011.
Ano Internacional de Qumica

Olimpada Brasileira
de Qumica 2011

XVII ONNeQ

XVII Olimpada Norte/Nordeste de Qumica


04/06/2011

Questo 1
Para cada um dos seguintes compostos:
Dissulfeto de carbono
Tricloreto de fsforo
Tetrafluoreto de estanho
Monxido de dicloro
Escreva a frmula molecular
Desenhe a estrutura de Lewis
D a hibridao do tomo central
Preveja a geometria molecular

Questo 2
Um tcnico dispe de um frasco de cido ntrico, em cujo rtulo est
escrito:
Concentrao = 60% em massa
Densidade = 1,48 g.mL-1
a. Escreva a frmula do cido ntrico
b. Escreva a equao qumica correspondente neutralizao do cido
ntrico pelo hidrxido de clcio
c. Determine a concentrao em mol.L-1 do cido ntrico contido no frasco
d. Que volume desse cido ntrico seria necessrio para preparar 500 mL
de uma nova soluo de cido ntrico de concentrao 2 mol.L-1.
e. Que volume de uma soluo de hidrxido de sdio de concentrao

Ano Internacional de Qumica



| 9

Programa Nacional
Olimpadas de Qumica

XVII ONNeQ
20 g.L-1 seria necessrio para neutralizar 20 mL da soluo de cido
ntrico preparada no item anterior (item d)?
Questo 3
Um processo industrial usado para remover cido sulfdrico do gs natural consiste em reagi-lo com dixido de enxofre, conforme a equao
qumica (no balanceada) abaixo:
H2S(g) + SO2(g) S(s) + H2O(g)
a) Reescreva a equao qumica acima balanceada
b) Que volume de SO2(g) , medido a 1 atm e 25 C, necessrio para
produzir 1 kg de enxofre?
c) Em uma reao iniciada com 100 g de cada um dos reagentes acima,
que reagente sobrar, considerando que um deles consumido totalmente?
d) Que massa do reagente em excesso sobrar ao final da reao?
Questo 4
Em um mistura dos gases N2O4 e NO2 em equilbrio, representado pela
equao qumica abaixo, temperatura de 0 C e presso de 1 atm, as
presses parciais desses gases so, respectivamente, 0,8 atm e 0,2 atm.

N2O4(g)

2 NO2 (g)

a) Calcule a constante de equilbrio expressa em presses, Kp.


10 |

Ano Internacional de Qumica


Olimpada Brasileira
de Qumica 2011

XVII ONNeQ
b) Calcule a constante de equilbrio expressa em concentraes, Kc.
c) Calcule o H para o equilbrio acima, a partir dos dados de entalpias
padres de reao a 273,15 K, dados a seguir:
N2(g) + 2 O2(g)

2 NO2(g)

H = + 78,31 kJ/mol

N2(g) + 2 O2(g)

N2O4(g)

H = + 9,67 kJ/mol

d) Um aumento da temperatura levar a uma maior ou menor dissociao de N2O4?


e) Calcule as presses parciais de ambos os gases, aps um novo equilbrio ser atingido ao se comprimir a mistura metade do volume original,
mantendo-se a temperatura constante.
Questo 5
O paracetamol, um dos analgsicos mais consumidos no mundo, pode
ser preparado atravs da seguinte sequncia de reaes:

a) Escreva os nomes dos compostos A, B, C, D e E


b) Escreva um nome sistemtico (IUPAC) para o paracetamol

Ano Internacional de Qumica



| 11

Programa Nacional
Olimpadas de Qumica

XVII ONNeQ
Conforme mostrado no esquema acima, o reagente usado na etapa 2
dessa sequncia de reaes foi o KOH e na etapa 4 foi H2, Pd/C.
c) Quais reagentes foram usados nas etapas 1, 3, 4 e 5
_______________

Dado: R = 0,082 L atm mol-1 K-1

A principal meta da educao criar homens que sejam capazes de fazer coisas novas, no simplesmente
repetir o que outras geraes j fizeram.
Jean Piaget
12 |

Ano Internacional de Qumica


Olimpada Brasileira
de Qumica 2011

Resultados

XVII Olimpada Norte/Nordeste de Qumica


04/06/2011

Resultado final
Nome

Escola

Cidade

UF

Davi Rodrigues Chaves

Ari de S Cavalcante

Fortaleza

CE

Nathianne de Moura de Andrade

Farias Brito

Fortaleza

CE

Raul Bruno Machado da Silva

Farias Brito

Fortaleza

CE

Mateus Braga de Carvalho

Instituto Dom Barreto

Teresina

PI

Thain Nobre Barros Rodrigues

Master

Fortaleza

CE

Breno Saldanha Sousa

Farias Brito

Fortaleza

CE

Davidson Anthony Arago Freire

Ari de S Cavalcante

Fortaleza

CE

Emerson Holanda Marinho

Farias Brito Central

Fortaleza

CE

Taynara Carvalho Silva

Master

Fortaleza

CE

Alex Silva de Cerqueira

IFBA

Salvador

BA

Natlia Arago Dias

Master

Fortaleza

CE

Jos Matheus G. de Alencar Bastos

Instituto Dom Barreto

Teresina

PI

Jos Marques Neto

Ari de S Cavalcante

Fortaleza

CE

Marcos Vincius Nunes de Souza

GGE

Recife

PE

Alynne Mara Alencar Justa

Ari de S Cavalcante

Fortaleza

CE

Sergio Pereira de Oliveira Jnior

Espao Aberto FB

Fortaleza

CE

Tamires Barbosa da Silva

Santa Bartolomea

Macap

AP

Francisco Davi Barbosa dos Santos

Farias Brito Central

Fortaleza

CE

Janaina Gomes Castro

Instituto Dom Barreto

Teresina

PI

Paulo Andr Herculano de Lima

Farias Brito

Fortaleza

CE

OURO

PRATA

Ano Internacional de Qumica



| 13

Programa Nacional
Olimpadas de Qumica

Resultados
BRONZE
Adriel Garcia Maquin Senado

IF-AM

Manaus

AM

Carlos Henrique da Silva

Prof. Adauto Carvalho

Serra Talhada

PE

Letcia Nunes de Oliveira

Instituto Dom Barreto

Teresina

PI

Mariana Camyla Duarte Pontes

Farias Brito Aldeota

Fortaleza

CE

Alisson de Sousa Barreto

Ari de S Cavalcante

Caucaia

CE

Clinton Henry Colao Conegundes

Instituto Dom Barreto

Teresina

PI

Flvio Luis Schneider Jnior

Militar de Manaus

Manaus

AM

Yuri Jernimo Moreira

Farias Brito Central

Fortaleza

CE

Andr Lopes Evangelista Dias

E. P. Madre Maria Villac

Teresina

PI

Bruno Limaverde Villar Lbo

Farias Brito

Fortaleza

CE

Eduardo Fernandes Baima

Ari de S Cavalcante

Fortaleza

CE

Francisco Rodrigues da Cruz Jnior

Instituto Dom Barreto

Teresina

PI

Nicolas Kemerich de Moura

Objetivo

Palmas

TO

Bruno de Oliveira Lima

Col. Aplicao da UFPE

Recife

PE

Gabriel Mathews Viana Pinheiro

Master

Fortaleza

CE

Guilherme Serra Baima

Marista Araagy

So Luis

MA

Jayane Carvalho Borges

Lettera

Teresina

PI

Laio Ladislau Lopes Lima

Farias Brito

Fortaleza

CE

Lara Mulato Lima

Ari de S Cavalcante

Fortaleza

CE

Matheus Braga Furstemberger

Nossa Sra. das Neves

Parnamirim

RN

Rafael Ribeiro Alves

Motiva

Campina Grande

PB

Mateus Juca Pinheiro

7 de Setembro

Fortaleza

CE

Vinicius Lopes Braga

Instituto Dom Barreto

Teresina

PI

Aline Tavora da Silva

Col. Aplicao da UFPE

Recife

PE

Guilherme Patriota Sampaio

Santa Maria

Recife

PE

Tiago Viana e Sousa

Madre Maria Villac

Teresina

PI

Leticia Laura Nobre Nunes Santos

Col. Marista

So Lus

MA

Marco Antonio Costa Nascimento

Escola Lato Sensu

Manaus

AM

Renan Lucas da Silva Custdio

Ari de S Cavalcante

Fortaleza

CE

MENO HONROSA

14 |

Ano Internacional de Qumica


Olimpada Brasileira
de Qumica 2011

Resultados
Tadeu Meneses de Carvalho

Instituto Dom Barreto

Teresina

PI

Adinaildo Gomes Paes Junior

Sistema Elite de Ensino

Barcarena

PA

Clariano Pires de Oliveira Neto

Educator

So Lus

MA

Iago Almeida Neves

Anchieta

Salvador

BA

Jos Ribamar Pereira Neto

Instituto Dom Barreto

Teresina

PI

Pedro Victor Barbosa Nolto

Instituto Dom Barreto

Teresina

PI

Vitria Nunes Medeiros

Farias Brito

Fortaleza

CE

Vitor Juc Policarpo

7 de Setembro

Fortaleza

CE

Alisson Bezerra Gomes

IFBA

Lauro de Freitas

BA

Ana Raquel Ferreira de Azevedo

Farias Brito

Sobral

CE

Filipe Herson Carneiro Rios

Farias Brito

Sobral

Ce

Lvio Moreira Rios

Crescimento

So Luis

MA

Joo Pedro Cavalcante Pereira

Santa Maria

Recife

PE

Antenor Teixeira Neto

Colegio Anchieta

Salvador

BA

Raissa Niuta Freitas de Oliveira

7 de Setembro

Fortaleza

CE

Erika Rodrigues Vieira de Macdo

IFPE

Paulista

PE

Rmullo Randell Macedo Carvalho

Lavoisier

Teresina

PI

Slvio Furtado Ximenes

DAULIA Bringel

Fortaleza

CE

Ana Flvia Galvo Lopes

Instituto Dom Barreto

Teresina

PI

Matheus Fernando Carvalho Lopes

Instituto Dom Barreto

Teresina

PI

Bianca Rohsner Bezerra

Farias Brito

Fortaleza

CE

Carolina Carvalho Tavares

Escola Lato Sensu

Manaus

AM

Rayssa Lima dos Santos

Santo Antnio de Jesus

S Ant. de Jesus

BA

Jos Victor Machado Nascimento

Ari de S

Fortaleza

CE

Luis Felipe Fonseca Dias

Geo Tamb

Joo Pessoa

PB

Wladimir Jos Lopes Martins

Motivo

Recife

PE

Juliana Moyses Poletti

Motiva

Joo Pessoa

PB

Wei Tzon Chang Colares

Lato Sensu

Manaus

AM

Maria Eduarda Kounaris Fuziki

Sartre Coc

Salvador

BA

Fernando Antnio Saraiva Maia

Dulia Bringel

Fortaleza

CE

Patricia Travassos Cutrim

Reino Infantil

So Lus

MA

Lydia Pearce Pessoa de Aguiar

Instituto Dom Barreto

Teresina

PI

to

Ano Internacional de Qumica



| 15

Programa Nacional
Olimpadas de Qumica

Resultados
Andreza Saboia Dantas

Cincias Aplicadas

Natal

RN

Wilson Vieira da Silva Jnior

Dom Barreto

Teresina

PI

Douglas Martins Carneiro

A. E. Profa Noronha

Dom Pedro

MA

Haroldo Nogueira Victoriano Neto

Dulia Bringel

Fortaleza

CE

Jos Marcelino de Souza Netto

Santa Maria

Timbaba

PE

Antonio Guilherme C. Silva Feitosa

Antoine Lavoisier

Timon

PI

Victor otvio Andrade das Neves

Olimpo

Palmas

TO

Oslio Cndido Arajo Limeira Lima Farias Brito

Fortaleza

CE

Yan Pontaja Medeiros da Silva

Santa Rosa

Belm

PA

Joo Pedro Santos Wanderley

Motivo

Recife

PE

Victor Santos de Andrade

Instituto Dom Barreto

Teresina

PI

Rodrigo Medeiros Guercio

Motiva

Joo Pessoa

PB

Ksia Priscilla Ohena Cardoso

IFAL

Maceio

AL

Joo Pedro Alexandre Silva Mota

Dulia Bringel

Fortaleza

CE

Mariana Feitosa Custdio

CEPRON

So Lus

MA

Diogo Farkatt Kabbaz

Genese de Ensino

Recife

PE

Lucas Brito Maynart

Salesiano

Aracaju

SE

Jos Lucas de Alencar Saraiva

Motivo

Timbaba

PE

Gabriel F. P. Arajo

Lavoisier

Teresina

PI

Rennan Martins Viana

E. P. Madre Maria Villac

Teresina

PI

Matheus Salmito Rodrigues Pontes

Farias Brito

Fortaleza

CE

Thereza Helena Azevedo Silva

Salesiano

Aracaju

SE

Artur Leite R Saldanha

Contato

Contato

AL

Janilson da Costa Barros

Fund. Nokia de Ensino

Manaus

AM

Nathrcia Castro Mota

Master

Fortaleza

CE

Janderson Sousa Ferreira

A. E. Profa Noronha

Dom Pedro

MA

Ygor Rodrigo Melo Fontes Santos

COESI

Aracaju

SE

Yan Moura Quintino

IFAL

Maceio

AL

Thaline Almeida Matos Viana

Educator

So Lus

MA

Ramon Santos G da Silva

Ideal

Belm

PA

Maria Gabriela Viana de Sa

Motiva

Campina Grande

PB

Joo Pedro de Carvalho Magalhes

GGE

Recife

PE

16 |

Ano Internacional de Qumica


Olimpada Brasileira
de Qumica 2011

Resultados
Daniel Abrantes Formiga

Motiva

Joo Pessoa

PB

Saymon do Livramento Santos

Elite

Belm

PA

Filipe Mouro Leite

Instituto Dom Barreto

Teresina

PI

Pedro Henrique A. Fraiman

Cincias Aplicadas

Natal

RN

Virna Mendona Sampaio Lima

Anchieta

Salvador

BA

Leo Meira Vital

Motiva

Campina Grande

PB

Carlos Eduardo Grivoljnior

Anchieta

Salvador

BA

rica Regina Rodrigues da Silva

CEPRON

So Lus

MA

Jos Ivan F. de O. Neto

Centro de Ensino Pleno

Belm

PA

Elis Arago Magalhaes

Contato

Caruaru

PE

caro de Almeida Varo

Instituto Dom Barreto

Teresina

PI

Nelly Carmen Ramirez Canelo

IF-AM

Manaus

AM

Naiara de Oliveira Baptista

Master Bezerra

Fortaleza

CE

Isabelle Rodrigues de M. Cmara

GGE

Recife

PE

Ana Mrcia Azevedo de Sousa

CEI

Natal

RN

Jorge Junior Pedroso Jordo

Lato Sensu

Manaus

AM

Caio Eddie de Melo Alves

Lato Sensu

Manaus

AM

Juliany Pires Figueredo

Lato Senso

Manaus

AM

Gabriel Wagner Sales Cavalcante

Motiva

Campina Grande

PB

Jos Santana da Silva Jnior

EREM Luz Alves Silva

S Cruz Capibaribe PE

Jaira de Sousa Arajo

Antoine Lavoisier

Teresina

PI

Marco Antonio S. Cavalcante Filho

Instituto Dom Barreto

Teresina

PI

Matheus Augusto Arajo Castro

Salesiano So Jos

Natal

RN

Wilson Coelho Mendes

Instituto Dom Barreto

Teresina

PI

Nicolas Eduardo da Fonseca Farias

Motiva Joo Pessoa

Joo Pessoa

PB

Amanda de Almeida e Silva

Ideal

Aracaju

SE

Jssica Silva Lopes

Farias Brito

Fortaleza

CE

Pedro Ferreira Torres

Col. Grupo Educ. Ideal

Belm

PA

Alisson Ribeiro Lucena

Motiva

Campina Grande

PB

Aluisio Pereira da Silva Filho

GGE

Recife

PE

Messias Bezerra da Silva Neto

Geo Tamb

Joo Pessoa

PB

Danielle Oliveira de Sousa

Ideal

Belm

PA

ta

Ano Internacional de Qumica



| 17

Programa Nacional
Olimpadas de Qumica

Resultados
Brenda Dias Marques

CEPRON

So Lus

MA

Eduarda Karynne Souza

EREM Luz Alves Silva

Sta Cruz Capibaribe

PE

Leandro Gomes Santana de Souza

Anchieta

Salvador

BA

Adda Morgana Alves Alves

Santa Rosa

Belm

PA

Marina Melo Sousa Mendes Leal

Sagrado Corao de Jesus Teresina

PI

Matheus Dantas de Lucena

Motiva

Campina Grande

PB

Caique Castro Souza

Classe A

Porto Velho

RO

Marcel Henrique Silva Moraes

SARTRE COC

Salvador

BA

Juliana Silva Brasil

Master

Fortaleza

CE

Diogo Silva Santana

COC Imperatriz

Imperatriz

MA

Kalil Lima Jardim Ferraz

Santa Emlia

Olinda

PE

Rafael Moura Andrade

Anglo Lavoisier

Teresina

PI

Heitor Caetano dos Santos

Instituto Brasil

Parnamirim

RN

Paulo Roberto Oliveira Martins

Escola Lato Sensu

Manaus

AM

Victor Ripardo Siqueira

Lato Sensu

Manaus

AM

Alex Mendes Leonel Freire

Contato

Macei

AL

Georgyson Dias Gondim Neo

Militar de Manaus

Manaus

AM

Matheus Leal de Souza

Antnio Vieira

Salvador

BA

Marcos Felyppe Oliveira Castro

MAC

Castanhal

PA

Diego Barros Albuquerque

Lato Senso

Manaus

AM

Irma Csasznik

Lato Sensu

Manaus

AM

Felipe Souza de Andrade

GGE

Camaragibe

PE

Jos Valnir Teixeira Cruz

Sagrado Corao de Jesus Teresina

PI

Raphael Souza de Almeida

Alternativo

Frei Paulo

SE

Brbara Beatriz de Almeida Gama

IFBA

Salvador

BA

Daniel Santos Rocha Sobral Filho

Ideal Militar

Belm

PA

Thiago Augusto Dantas Vila Nova

Inst. Maria Auxiliadora

Natal

RN

Caio Csar Melo Delgado

Geo Tamb

Joo Pessoa

PB

Ana Brbara de Jesus Chaves

Arquidiocesano

Aracaju

SE

Lucas Medeiros Sobrinho de Sousa

IFBA

Salvador

BA

Joo Victor Bulamarqui Coelho

Crescimento

So Luis

MA

Victor Gaspar Silva e Silva

Col. Marista

So Lus

MA

18 |

Ano Internacional de Qumica


Olimpada Brasileira
de Qumica 2011

Resultados
Dborah Nbrega de Farias

Motiva

Joo Pessoa

PB

Erik Leite de Almeida

Santa ursula

Maceio

AL

Matias Daniel F. Batista

Santa Ursula

Maceio

AL

Ezau Silva Ribeiro

Sistema Elite de Ensino

Barcareno

PA

Klaus Anton Tyrrasch

Contato

Maceio

AL

Marcus Di Fabianni F. Lopes Filho

Militar de Manaus

Manaus

AM

Guilherme Henrique M. de Araujo

Crescimento

So Lus

MA

Eliza Edneide O. Souza de Almeida

Imaculada Conceio

Campina Grande

PB

Ruth Ellen F de Castro Dantas

CEI

Natal

RN

Luiz Fernando da Silva Costa Santos Saint-Louis

Aracaju

SE

Danilo Pequeno

Motiva

Queimadas

PB

Emilly Rennale Freitas de Melo

Motiva

Campina Grande

PB

Eleodrio Sales Bonfim Neto

Motiva

Areia

PB

Gabriel Csar Pereira

Nsa. Sra da Conceio

Craibas

AL

Gustavo Henrique M. Ferreira Filho

Contato

Maceio

AL

Henrique Santos de Almeida

Militar de Manaus

Manaus

AM

Rafaela Ges Machado

Anchieta

Salvador

BA

Brbara Oliveira de Andrade

Instituto Dom Barreto

Teresina

PI

Analice Cardoso de Brito

Teotonio Ferreira Brando Cocal dos Alves

PI

Sarah Pereira Martins

Esc. Crescimento

So Lus

MA

Patricia dos Santos Lima Dutra

Motiva

Joo Pessoa

PB

Victor Peres de Melo Goulart

Militar de Manaus

Manaus

AM

Carla Lorena Silva Cardoso

Sistema Elite Vila

Barcarena

PA

Cralos Eduardo Adriano Filho

Mar

Castanhal

PA

Paula Carolline Costa de SantAna

Salvador

Aracaju

SE

Jos Matheus Santos Pereira

IFBA

Salvador

BA

Alan de Andrade Monteiro Neto

Estadual

Recife

PE

Ano Internacional de Qumica



| 19

Programa Nacional
Olimpadas de Qumica

IV OBQ Jnior

IV Olimpada Brasileira de Qumica Jnior


Fase I
8 e 9 anos do Ensino Fundamental
Exame aplicado em 27.08.1011
INSTRUES
1. A prova consta de 20 questes objetivas, cada uma contendo quatro alternativas, das quais apenas uma deve ser assinalada.
2. A prova tem durao de 3 horas
3. Voc receber o gabarito aps 1 hora do incio da prova, para registrar as suas
opes de respostas.
01 Uma pgina da internet contm a seguinte afirmao: O petrleo um lquido escuro, de aspecto oleoso, menos denso que a gua. De acordo com as
caractersticas desse produto, essa afirmativa :
A) correta, pois ele mais leve devido ser uma substncia simples.
B) incorreta, porque o petrleo uma mistura de muitos constituintes, portanto
mais denso que a gua.
C) correta, pois temperatura ambiente o petrleo permanece sempre sobre a
superfcie da gua.
D) incorreta, pois o petrleo uma substncia mais densa do que a gua, doce ou salgada.
02 Um tipo de petrleo bruto extrado do subsolo misturado com gua salgada, areia e argila. Por isso, utilizam-se dois processos distintos (I e II) para a
separao desses materiais. Inicialmente, atravs do processo I, separa-se a gua
salgada. Depois, utilizando-se o processo II, retiram-se a areia e a argila. Seguindo essa sequncia, qual das alternativas abaixo traz uma indicao tecnicamente
mais adequada para os processos I e II, respectivamente?
A) Decantao e filtrao.
B) Destilao e fuso.
C) Filtrao e evaporao.
D) Fuso e destilao.
03 Em um vdeo disponibilizado no YouTube, um recipiente de vidro fechado
que contm alguns cristais de iodo (I2) foi aquecido. Com isso, devido transio
do estado slido para o gasoso, verificou-se a formao de uma nuvem de colorao violeta em seu interior.
Adaptado de http://www.youtube.com/user/VideosdeQuimicaUFF

20 |

Acesso em 31/07/2011

Ano Internacional de Qumica


Olimpada Brasileira
de Qumica 2011

Exames - Fase I
Nessa demonstrao, ocorre a
A) destilao de uma substncia covalente.
C) sublimao de uma substncia covalente.

B) vaporizao de uma substncia inica.


D) recristalizao de uma substncia inica.

04 Uma das etapas para processamento do mel de forma higinica e segura envolve o uso de um equipamento que, atravs do movimento de rotao em torno
de seu prprio eixo, retira esse alimento dos alvolos existentes na colmeia. No
comrcio, esse tipo de equipamento encontrado com vrias capacidades de
extrao, podendo ser com sistema de rotao acionado manualmente ou por
eletricidade, com motor e dispositivos de controle de velocidade de rotao.
Adaptado de http://sistemasdeproducao.cnptia.embrapa.br/FontesHTML/Mel/SPMel/extracaomel.htm
Acesso em 31/07/2011

Que tipo de processo de separao realizado nesse equipamento?


A) Centrifugao.
B) Cristalizao.
C) Flotao.
D) Peneirao.

05 O soro fisiolgico vendido em farmcias um produto que contm 0,9%, em


massa, de NaCl em gua destilada. Esse produto considerado uma
A) mistura heterognea.

B) soluo contendo ons.
C) soluo saturada.
D) substncia composta.
06 Uma chaleira contm apenas gua mineral e foi aquecida em um fogo. Aps
um curto perodo, observou-se uma forte produo de vapor, como uma consequncia da(o)
A) diminuio das interaes entre as molculas da gua.
B) desejo dos tomos presentes na gua em doar ou receber eltrons.
C) rompimento das ligaes covalentes durante a mudana de estado fsico da
gua.
D) decomposio das molculas de gua para produo de hidrognio e oxignio gasosos.
07 Em uma atividade de laboratrio transferiu-se um pouco de acar de mesa
(sacarose) para um bquer. Em seguida, agitou-se com um basto de vidro e,
rapidamente, observou-se que o lquido ficou transparente e lmpido. Sobre esse
processo, foram feitas trs afirmativas, conforme mostrado abaixo.
I. O acar dissolvido pode ser recuperado atravs de um mtodo de separao.
II. O acar reagiu com a gua e formou uma soluo, mas a sua doura continuou a existir.
III. O acar reagiu com a gua e deixou de existir, ou seja, transformou-se em
outra substncia.

Ano Internacional de Qumica



| 21

Programa Nacional
Olimpadas de Qumica

IV OBQ Jnior
Qual(is) dessa(s) afirmativa(s) est(o) CORRETA(S)?
A) I, apenas.
B) II, apenas.
C) II e III, apenas.

D) I e III, apenas.

08 O mendelvio (Z=101; configurao eletrnica: [Rn] 5f13 7s2) um elemento


qumico identificado em 1955, mesmo aps ter sido produzido em quantidade
insuficiente para ser observado a olho nu. O seu nome foi uma homenagem ao
qumico russo Dmitri Mendeleev. O Md possui
A) 101 prtons e 101 eltrons.
B) menos eltrons do que o radnio (Rn).
C) configurao eletrnica idntica aos demais elementos qumicos artificiais.
D) menos prtons do que o elemento qumico natural com maior nmero atmico.
09 Diferentes propostas didticas trazem verses eletrnicas para a tabela peridica, conforme o fragmento mostrado ao lado, que foi retirado de uma delas.
Nesse fragmento h diferentes informaes sobre o alumnio, tais como: nmero
atmico, massa atmica, configurao eletrnica, fonte natural e utilizao na
forma elementar ou como liga. A partir dessas informaes CORRETO afirmar
que
A) a massa atmica do alumnio igual a 13.
B) panelas descartadas so as principais fontes para a
reciclagem do alumnio.
C) o alumnio na forma de uma substncia simples,
elementar, chamado de bauxita e usado em fogos
de artifcio.
D) A localizao do alumnio no 3 perodo da tabela
peridica tem relao com a sua configurao eletrnica.
http://www.abiquim.org.br/tabelaperiodica/tabela_est.asp
Acesso em 31/07/2011

10 Na dcada de 1920, independentemente Ernest Rutherford (1871-1937), na


Inglaterra, William Draper Harkins (1873-1951), nos EUA, e Orme Masson (1858
1937), na Austrlia, propuseram a possvel existncia de uma partcula atmica
sem carga. Porm, apenas em 1932, na Inglaterra, James Chadwick (1891-1974)
comprovou a existncia do ___(I)___. Esse processo exemplifica que a cincia
uma atividade ______(II)________.

22 |

Ano Internacional de Qumica


Olimpada Brasileira
de Qumica 2011

Exames - Fase I
As lacunas (I) e (II) no texto acima podem ser completadas de forma CORRETA e
na mesma sequncia pela opo
A) I eltron; II - prpria do sexo masculino.
B) I nutron; II - individual.
C) I prton; II - extensiva a pesquisadores de vrios pases.
D) I nutron; II - humana.
11 O fsforo tem Z = 15 e todo fsforo encontrado na natureza 31P. Apesar disso, os compostos do fsforo-32 (P-32) so muito empregados como marcadores
radioativos, para compreender os ciclos vitais de plantas e animais, onde haja a
participao de compostos que contm fsforo de massa 31.
Adaptado de PEIXOTO, E.A.M. Fsforo. Qumica Nova na Escola, 15, 51, 2002.

Diante dessas consideraes, CORRETO afirmar que o fsforo encontrado na


natureza e o fsforo utilizado como marcador radioativo so
A) elementos qumicos diferentes.

B) isbaros.
C) istopos.

D) substncias naturais.
12 Em uma atividade experimental realizada na sala de aula, uma professora
transferiu uma pequena quantidade de um lcool, slido temperatura ambiente, para um bquer que continha gua destilada. Aps 1 hora, verificou-se que o
lcool no se dissolveu. Em relao ao que foi observado durante esse perodo,
CORRETO afirmar que
A) esse lcool o etanol.
B) o lcool se dissolveria caso a gua destilada estivesse muito gelada.
C) a diferena de densidade entre as duas substncias impediu a dissoluo do lcool.
D) o sistema formado entre esse lcool e a gua um exemplo uma mistura
heterognea
13 O quadro abaixo traz uma relao entre componentes, caractersticas e usos
de algumas ligas metlicas.
Liga metlica

Componentes

Caracterstica

Exemplo de uso

Ferro e carbono

Resistncia corroso

Utenslios domsticos

Amlgama

Mercrio, prata e estanho

Resistncia mecnica

II

Ouro 18 quilates

Ouro e cobre

III

Jias

Ano Internacional de Qumica



| 23

Programa Nacional
Olimpadas de Qumica

IV OBQ Jnior
Para atender aos dados fornecidos no quadro anterior, as lacunas I, II e III podem
ser completadas de forma CORRETA e na mesma sequncia pela opo
A) solda industrial; ferramentas e moedas; e brilho intenso.
B) ao; restaurao de dentes; e alta ductibilidade e maleabilidade.
C) bronze; soldas de produtos eletrnicos; e baixa condutividade eltrica.
D) lato; plsticos biodegradveis para sacolas; alta condutividade trmica.
14 A ao benfica do oznio ao ser humano bem conhecida. Mas seu efeito
nocivo ou favorvel vida na Terra depende da altitude em que ele se situa. A
maior parte desse gs est na estratosfera (entre 13 e 40 km), onde funciona como
escudo radiao ultravioleta. Mais prximo da superfcie, porm, o O3 um poluente que causa danos ao tecido pulmonar dos animais e prejudica a vegetao.
Na troposfera, o oznio (O3) formado quando molculas de oxignio (O2) so
divididas pela luz solar e os tomos livres desse elemento se ligam a outras molculas de oxignio. Prximo da Terra, o oznio formado pela reao do oxignio
com poluentes urbanos.

http://cienciahoje.uol.com.br/noticias/meteorologia/acao-de-relampagos-modifica-quimica-daatmosfera
Acesso em 31/07/2011

A formao do oznio na atmosfera envolve uma


A) diviso de tomos de oxignio pela luz solar.
B) quebra da ligao inica do gs oxignio na troposfera.
C) reao entre o O2 , poluente e tomos livre do O em baixas altitudes.
D) reao na qual h a participao da sua forma alotrpica em altitudes entre
13 e 40 km.
15 Os diferentes aspectos a serem considerados na concepo de um processo
para cumprir os princpios e obter uma sntese verde envolvem diferentes parmetros como, por exemplo,
A) a potencializao da incorporao dos reagentes no produto
B) o uso de matrias-primas de fontes renovveis, como o petrleo.
C) a gerao de compostos degradveis que liberem metais pesados.
D) a incluso de compostos txicos como reagentes, para estimular a sua retirada
do comrcio.

24 |

Ano Internacional de Qumica


Olimpada Brasileira
de Qumica 2011

Exames - Fase I
16 Observe a charge mostrada abaixo.

http://interagindoquimica.blogspot.com/2011/02/charge-quimica.html
Acesso em 03/08/2011

Corrigindo gramtica e quimicamente os textos dessa charge, trs tipos de metais seriam
A) carbono, potssio e zinco.
B) prata, titnio e cobre.
C) sdio, cloro e mercrio.
D) urnio, hlio e cromo
17 Nos palitos de fsforo que conhecemos, no h presena do elemento fsforo; eles so encontrados na parte spera da caixa. Na ponta do palito (a parte
vermelha) ns temos clorato de potssio (KClO3), responsvel por liberar oxignio para manter a chama acesa, e a outra parte do palito revestida por uma
camada de parafina (mistura de hidrocarbonetos). Na caixa, temos sulfeto de
antimnio, Sb2S3, e trixido de ferro, Fe2O3, para gerar atrito, e o fsforo, para
produzir calor intenso. Quando riscamos o palito na caixa produzimos uma fasca
que em contato com o clorato de potssio libera muito oxignio (O2) que reage
com a parafina gerando uma chama que consome o palito de madeira.
http://www.infoescola.com/elementos-quimicos/fosforo/ (Modificado)

Acesso em 31/07/2011

O acendimento de um palito de fsforo pelo atrito na caixinha de fsforo envolve


A) uma forma alotrpica do fsforo existente na ponta do palito.
B) a liberao de uma substncia inica que reage com a parafina.
C) reaes envolvendo a liberao e consumo de uma substncia simples.
D) uma transformao qumica causada pelos xidos, substncias simples, presentes nas caixas.
18 O conceito de ecomaterial passou a ser disseminado, a partir do incio da
dcada de 1990, como uma resposta frente necessidade de novas tecnologias
para a produo de materiais ambientalmente adequados. Considerando essa

Ano Internacional de Qumica



| 25

Programa Nacional
Olimpadas de Qumica

IV OBQ Jnior
caracterstica e os princpios da qumica verde, qual das alternativas abaixo traz
um exemplo de ecomaterial para a proteo ambiental?
A) Chumbo de bateria de carros para a descolorao de efluentes.
B) Clulas solares produzidas a partir de baterias de telefones recicladas.
C) Cermicas condutoras de energia produzidas a partir de carvo vegetal.
D) Carbono obtido a partir do p de casca de coco para uso na remoo de
corantes.
19 A caracterstica intrinsecamente interdisciplinar de temas como a microeletrnica, vidros especiais, plsticos e aos de alto desempenho e das atuais pinturas
automobilsticas, contriburam para a formao de uma nova rea na Qumica.
Ela envolve qumicos orgnicos, inorgnicos, fsico-qumicos e tambm fsicos,
engenheiros e outros profissionais ligados aos departamentos das universidades,
dos institutos de pesquisas e das indstrias. De acordo com as caractersticas
apontadas, essa rea conhecida como
A) Biotecnologia. B) Eletroqumica. C) Qumica de Materiais. D) Radioqumica.
20 Tambm originada da Qumica clssica, a Qumica Ambiental atualmente
uma cincia interdisciplinar e um dos seus objetivos
A) desenvolver pesquisas que comprovem a influncia humana nos recentes desastres naturais.
B) aumentar as emisses gasosas industriais para controlar os fenmenos naturais atmosfricos.
C) melhorar os processos para que a humanidade retorne a um modo de vida
mais rstico e artesanal.
D) estudar processos qumicos que ocorrem na natureza, tanto naturais quanto
os causados pelo homem.
GABARITO - QUESTES DE MLTIPLA ESCOLHA
1

10

11

12

13

14

15

16

17

18

19

20

26 |

Ano Internacional de Qumica


Olimpada Brasileira
de Qumica 2011

Exames - Fase II

IV Olimpada Brasileira de Qumica Jnior - Fase II


Exame aplicado em 01.10.1011

QUESTES DE MLTIPLA ESCOLHA


01 Os moradores de um bairro organizaram uma campanha sobre coleta seletiva de lixo e reciclagem de materiais. Dentre as aes realizadas, eles conseguiram e distriburam conjuntos de lixeiras de cores diferentes. Em cada
uma delas deveriam ser depositados produtos e objetos de acordo com os
componentes dos seus respectivos materiais:
Amarela - Metal; Azul - Papel; Vermelha Plstico; Verde - Vidro.
Caso seja seguida essa orientao, a maior probabilidade de se encontrar
objetos que ao serem reciclados forneam altas porcentagens de alguma
substncia elementar est na lixeira de cor
A) amarela.

B) azul.
C) verde.
D) vermelha.
02 Observe a charge mostrada ao
lado. Ela faz uma crtica
A) sntese verde de uma substncia
gasosa a partir da gua poluda.
B) etapa poluente do principal processo da industrializao de O2 a
partir de fontes naturais.
C) despoluio de grandes reservatrios naturais com a utilizao
de oxignio comercial.
D) forma de poluio que compromete a dissoluo de uma importante substncia apolar na gua.
Acesso em 31/07/2011

03 Um experimento interativo montado em um museu de cincias trazia um


circuito eltrico contendo uma sirene conectada, atravs de fios de cobre, a
uma bateria e a dois eletrodos. No momento em que os eletrodos eram colocados dentro de algumas solues, o circuito era fechado, havia passagem
de corrente, e a sirene era acionada.

Ano Internacional de Qumica



| 27

Programa Nacional
Olimpadas de Qumica

IV OBQ Jnior
Qual das solues abaixo possui maior probabilidade de acionar essa sirene
de modo mais forte ao serem colocadas em contato com os eletrodos desse
experimento?
A) Soluo de sacarose.

B) Soluo de etanol a 50%.

C) Soluo concentrada de cloreto de sdio.


a 4% (vinagre).

D) Soluo de cido actico

04 A regra do octeto uma das formas mais utilizadas para explicar as ligaes qumicas entre tomos de um mesmo elemento ou de diferentes
elementos qumicos. Porm, h alguns compostos que so excees a essa
regra, mostrando as suas limitaes enquanto um modelo nico para prever
as valncias e as frmulas de todos os compostos. Abaixo, associe as duas
colunas, relacionando alguns tipos de exceo regra do octeto aos seus
exemplos correspondentes.
( ) PCl5, I3-

(I) ons de metais de transio


(II) Compostos de gases nobres

( ) Fe +, Cu +

(III) Molculas e ons contendo tomos com mais de oito eltrons

( ) XeO3, KrCl2


A sequncia CORRETA dessa associao
A) I, II, III

B) II, III, I

C) II, I, III D) III, I, II

05 Tungstnio e argnio so utilizados, respectivamente, como constituintes do


A) filamento e do gs inerte de lmpadas incandescentes.
B) recipiente plstico e do comprimido efervescente usado contra azia.
C) material metlico e do gs combustvel de cilindros usados na soldagem
de peas de ferro.
D) vidro e do lquido de termmetros usados em residncias, para verificar a
temperatura ambiente.

28 |

Ano Internacional de Qumica


Olimpada Brasileira
de Qumica 2011

Exames - Fase II
06

Acesso em http://www.profpc.com.br/Tirinhas%20de%20Qu%C3%ADmica.htm
Acesso em 31/07/2011

Embora a cena retratada na tirinha acima no seja citada nos relatos histricos sobre Dmitri Ivanovich Mendeleev (1834-1907), com um humor, ela
envolve uma das estratgias utilizadas na proposio da tabela peridica por
esse qumico russo. Ele criou uma carta para cada um dos 63 elementos conhecidos at aquele momento e as organizou por ordem
A) cronolgica da identificao de cada elemento qumico e agrupando-as
ao acaso.
B) crescente de massas atmicas e agrupando-as em elementos de propriedades semelhantes.
C) decrescente de nmeros atmicos e agrupando-as de acordo com as letras dos seus smbolos.
D) alfabtica do nome do elemento qumico e agrupando-as pela semelhana dos seus pontos de fuso.

07 O nitrognio pode estar presente na gua sob vrias formas: molecular,


amnia (NH3), nitrito (NO2-), nitrato (NO3-); um elemento indispensvel ao
crescimento de algas, mas, em excesso, pode ocasionar um exagerado desenvolvimento desses organismos, fenmeno chamado de eutrofizao. So
causas do aumento do nitrognio na gua: esgotos domsticos e industriais,
fertilizantes, excrementos de animais.
Adaptado de http://www.ufv.br/dea/lqa/qualidade.htm

Acesso em 31/07/2011

Ano Internacional de Qumica



| 29

Programa Nacional
Olimpadas de Qumica

IV OBQ Jnior
Segundo as informaes contidas no texto acima:
A) a presena de N2 na gua decorrente do processo de eutrofizao.
B) substncias moleculares e inicas de nitrognio so elementos qumicos
indispensveis ao crescimento de algas.
C) o lanamento de excrementos de animais na gua pode levar a um excesso da produo da forma molecular do nitrognio, NO2.
D) esgotos domsticos e industriais e fertilizantes podem aumentar a quantidade de NH3, NO2- e NO3- na gua e contribuir para um exagerado desenvolvimento das algas.
08 Observe as informaes sobre os elementos qumicos indicados abaixo.

Adaptado de http://www.abiquim.org.br/tabelaperiodica/tabela_est.asp

Acesso em 31/07/2011

De acordo com dados apresentados, CORRETO afirmar que


A) ao ganhar um eltron um tomo de flor se torna um tomo de nenio.
B) ao ganhar um eltron um tomo de sdio se torna um tomo de nenio.
C) a formao do composto NaF energeticamente mais favorvel do que a do NeF.
D) os trs elementos qumicos indicados pertencem ao mesmo perodo da
tabela peridica.
09 Durante uma visita organizada por uma escola a um laboratrio de uma
universidade, os alunos realizaram uma anlise de um reagente qumico.
Eles pulverizaram uma pequena quantidade do reagente em um almofariz e transferiram uma amostra para dentro de um pequeno e estreito tubo
de vidro, fechado em uma das extremidades. O reagente foi compactado
at ser obtida uma camada de aproximadamente 1 cm de altura. Depois, o
30 |

Ano Internacional de Qumica


Olimpada Brasileira
de Qumica 2011

Exames - Fase II
tubo foi preso a um termmetro, deixando-se a parte contendo a amostra
junto ao bulbo. Em seguida, mergulhou-se esse sistema em um recipiente
contendo leo mineral, que foi sendo aquecido. Passados alguns instantes,
percebeu-se o aparecimento de algumas gotculas no interior do tubo de vidro e observou-se a temperatura em elevao. Somente depois de 30 (trinta)
minutos dessa primeira observao todo reagente ficou lquido. Esse experimento permitiu verificar a
A) ebulio do reagente.
B) ebulio de uma substncia pura
C) fuso de uma mistura.
D) fuso de uma substncia pura.
10 Em uma atividade experimental foram transferidos para dois bqueres distintos, A e B um mesmo volume de gua (d=1,0 g/mL; p.e. = 100oC), 500 mL.
Depois, adicionou-se 500 mL de etanol, CH3CH2OH (d = 0,789 g/mL; p.e. = 78,4
o
C) ao bquer A e 500 mL de clorofrmio, CHCl3 (d = 1,48 g/mL; p.e. = 61,2 oC)
ao bquer B. Aps agitao, um dos sistemas ficou monofsico enquanto que
o outro ficou bifsico. Sobre esse processo, CORRETO afirmar que
A) no bquer B, a fase inferior era gua.
B) no bquer A, a fase superior era o etanol.
C) possvel separar cada componente do sistema do bquer A atravs de
evaporao dessa soluo.
D) possvel separar as fases componentes do sistema do bquer B utilizando-se um funil de separao.
QUESTES ANALTICO EXPOSITIVAS

11 O grafeno um material composto por


uma folha de grafite. Ele contm uma nica
camada de tomos de carbono (Z=6; Configurao eletrnica: 1s22s22p2) organizados de
forma hexagonal, como favos em uma colmia, conforme indicado na figura ao lado. O
grafeno promissor para o desenvolvimento
de telas sensveis ao toque (touchscreens) e
clulas fotovoltaicas porque flexvel, transparente e condutor de eletricidade.

Adaptado de http://www1.folha.uol.com.br/ciencia/
Acesso em 31/07/2011

De acordo com as caractersticas e propriedades apresentadas pelo grafeno,


CORRETO afirmar que ele uma forma alotrpica de um elemento metlico? Justifique a sua resposta.
Ano Internacional de Qumica

| 31

Programa Nacional
Olimpadas de Qumica

IV OBQ Jnior
12 Realizou-se um experimento de simulao atmica em uma sala de aula
para uma abordagem sobre a natureza microscpica da matria. Utilizando-se uma rgua plstica e outros objetos de baixo custo, montou-se uma
balana. Aps serem colocadas 12
(doze) bolas de isopor em um dos
pratos e 1 (uma) bola no outro prato
da balana, o sistema permaneceu
em uma situao de equilbrio, conforme mostrado ao lado.
Explique essa situao de equilbrio
baseando-se em um tipo de modelo
atmico.
Fonte: Gambo, J. A.; Corso, H.L.; Severno, M.H.
Qumica atractiva en un ingreso a la universidad. Rev. Eureka Ense. Divul. Cien., 6, 3, 423-439, 2009.

13 Durante um determinado processo foram produzidos 88 (oitenta e oito)


gramas de dixido de carbono. Sabendo que esse processo est representado pela equao da reao qumica C + O2
CO2 indique quantos
mols do outro gs sero necessrios para produzir essa quantidade de CO2.
Justifique a sua resposta.
Dados de massas atmicas dos elementos qumicos: C = 12 u; O = 16 u.

GABARITO
1

10

Se um dia voc tiver que escolher entre o mundo e o amor, lembre-se: Se escolher o
mundo ficar sem amor, mas se voc escolher o amor, com ele conquistar o mundo.
ALBERT EINSTEIN

32 |

Ano Internacional de Qumica


Olimpada Brasileira
de Qumica 2011

Questes resolvidas I

OBQJr -2011 - RESPOSTAS ESCOLHIDAS


QUESTO 11
Resoluo de Teresa Virgnia Neves Floriano - Colgio Amadeus, Aracaju-SE
No. Pois o grafeno composto por apenas uma nica camada de tomos de
carbono e, sendo composto por carbono, no pode ser considerado um elemento metlico j que o carbono no um metal. Por esse motivo, no correto afirmar que o grafeno uma forma alotrpica de um elemento metlico.

QUESTO 12
Resoluo de Pedro Henrique Rocha de Freitas - Colgio Militar, Braslia-DF
Utilizando o modelo atmico de Dalton, podemos dizer que as bolas de isopor
representam o elemento qumico X, com massa A. Os tomos desse elemento
so esferas macias e indivisveis, assim como acontece com outros elementos
qumicos, e cada elemento apresenta sua massa prpria (segundo o modelo
atmico de Dalton). J a bola que est do outro lado da balana representa o
elemento qumico Y, com massa 12A. Como de um lado temos 12 tomos de
X, e do outro 1 tomo de Y, chegamos na seguinte equao das massas dos
dois elementos:
12Ax = 1.12Ay
Como essa equao verdade, chegamos concluso de que a massa de 12
tomos de X igual massa de 1 tomo de Y.
Portanto, pelo modelo atmico de Dalton, explicamos que os dois pratos da
balana esto em equilbrio, pois a massa de 1 tomo de Y 12 vezes maior
que a massa de 1 tomo de X.

Ano Internacional de Qumica



| 33

Programa Nacional
Olimpadas de Qumica

IV OBQ Jnior
QUESTO 13
Resoluo de Victria Freitas Vieira da Cunha, Colgio Paraso - Juazeiro do
Norte CE
So necessrios 32 g de O2 para produzir 44 g de CO2. Logo, sero necessrios
X g de O2 para produzir 88 g de CO2 tal que:
O2
CO2
32 g - 44 g
x g - 44 g
x=64 g

1 mol de O2 - 32 g
x mol de O2 - 64 g
x=2 mol

So necessrios 64 g de O2 para obter 88 g de CO2, portanto sero necessrios


2 mols de O2.

Nunca consideres o estudo como uma obrigao, mas como uma


oportunidade para penetrar no belo e maravilhoso mundo do saber.
Albert Einstein
34 |

Ano Internacional de Qumica


Olimpada Brasileira
de Qumica 2011

Resultados
NOME
OURO
Pedro Henrique Rocha de Freitas
Dbora Letcia Nogueira de Oliveira
Isabelle de Sousa Pereira
Marcus Vincius dos Santos Lima
Mateus Vasconcelos Albuquerque
Vittria Nobre Jacinto
P R ATA
Thalles Ferreira da Ponte
Gabriel Demetrius Bertoldo da Silva
Carlos Gabriel Oliveira Freitas
George Henrique Nunes da M. Jnior
Iago Dantas Figueirdo
Julia Praciano Lopes
Daniel Pinheiro Mota da S. Ferreira
Elcio Koodiro Yoshida
Iman Musa Ismil Abdel R. Jadallah
Eduardo Serpa
Felipe DAmorim Barreto
Fernando de Moraes Rodrigues
Gesa Costa Oliveira de M. Santana
Luciano Pinheiro Batista
Priscila Tavares Vitoriano
Rapahel Fernandes Ligrio
BRONZE
Ana Karoline Borges Carneiro
Eric Ian Noronha Junqueira
Maria Paula Tellez Frias
Mateus Arago Esmeraldo
Idlia Maria Barbosa do Nascimento
Victor Sousa Silva
Danielle Mayumi Shiba
Pedro Salazar Costa
Roberto Rebouas Prates Filho
Thas Silva Souza

ESCOLA

CIDADE

UF

Militar de Braslia
Esc. Modelo de Iguatu
Farias Brito
UNISUZ
Santa Ceclia
Farias Brito

Braslia
Iguatu
Fortaleza
Suzano
Fortaleza
Fortaleza

DF
CE
CE
SP
CE
CE

Farias Brito
Objetivo - Cantareira
7 de Setembro
Farias Brito
Anchieta
Farias Brito
Antnio Vieira
Etapa
Militar de Braslia
Master
Anchieta
Farias Brito
Farias Brito
Farias Brito
Farias Brito
Marista Dom Silvrio

Sobral
So Paulo
Fortaleza
Fortaleza
Salvador
Fortaleza
Salvador
So Paulo
Braslia
Fortaleza
Salvador
Fortaleza
Fortaleza
Fortaleza
Fortaleza
Belo Horizonte

CE
SP
CE
CE
BA
CE
BA
SP
DF
CE
BA
CE
CE
CE
CE
MG

Farias Brito
Ari de S Cavalcante
Anchieta
Ari de S Cavalcante
Ass. Educ. Profa. Noronha
Ari de S Cavalcante
Polcia Militar do Paran
Centro Educ.Adalberto Valle
Anchieta
Fund. Itab. Dif. do Ensino - FIDE

Fortaleza
Fortaleza
Salvador
Fortaleza
Dom Pedro
Fortaleza
Curitiba
Manaus
Salvador
Itabira

CE
CE
BA
CE
MA
CE
PR
AM
BA
MG

Ano Internacional de Qumica



| 35

Programa Nacional
Olimpadas de Qumica

IV OBQ Jnior
Gabriel Pereira Penna Andrade
Gustavo Oliveira Martins
Lia Santos Ponce de Leon
Lucas Bastos Oliveira
Alexsander Felipe Alencar Justa
Bruna Luiza Braga Pantoja
Davi Vieira Ferreira
Elo Santiago da Silva Pereira
Joo Pedro Grangeiro Carioca
Joel Ferreira Madureira
Lara Poti Nobre
Lia de Oliveira Domingues
Matheus Bernini
Mayara Melo dos Santos
Narjara Smya Rodrigues Pereira
Ticiana Alencar Noronha
MENSO HONROSA
Juan Freire Dantas Galvo
Pedro Henrique S. de Oliveira
Gabriel de Sousa Correia
Rhayssa Ingrid Pereira Rangel
Vincius Rodrigues Mascarenha
Lueverton Gonalves dos Santos
Matheus Caris Castro
Rodrigo Ceccato de Freitas
Gabriel Cortizo Ferraz
Pedro Macedo Flores
Gabriel Guedes Ferreira de Souza
Joana Laila Vital Carneiro
Ricardo Nbrega Machado
Bruno Robert Vasconcellos Oliveira
Carlos Arthur Grangeiro Sampaio
Danilo Cins SantAna de Lima
Euller Reis Farias
Felipe Martins Gomes
Gabriel Moraes Ramos Studart
36 |

Fund. Itab. Dif. do Ensino - FIDE


Dulia Bringel
Dulia Bringel
Dulia Bringel
Ari de S Cavalcante
7 de Setembro
Paraso
Objetivo Unidade Aquarius
Farias Brito
Farias Brito
Farias Brito
Farias Brito
Pedro II UEEN II
Ari de S Cavalcante
Ass. Educ. Profa. Noronha
Farias Brito

Itabira
Fortaleza
Fortaleza
Fortaleza
Fortaleza
Fortaleza
Juazeiro do Norte
S. J. dos Campos
Fortaleza
Fortaleza
Fortaleza
Fortaleza
Rio de Janeiro
Fortaleza
Dom Pedro
Fortaleza

MG
CE
CE
CE
CE
CE
CE
SP
CE
CE
CE
CE
RJ
CE
MA
CE

Farias Brito
Etapa Valinhos
Farias Brito
Ari de S Cavalcante
CEV GEO Garcia e Brito
Farias Brito
Gabriel Chalupe
Boa Viagem
Militar de Juiz de Fora
Dulia Bringel
Farias Brito
Polcia Militar do Paran
Anchieta
Paraso
Anchieta
Ass. Educ. Profa. Noronha
Master
Ari de S Cavalcante

Fortaleza
Valinhos
Fortaleza
Fortaleza
Teresina
Mossor
Fortaleza
Barueri
Recife
J. de Fora - MG
Fortaleza
Sobral
Curitiba
Salvador
Juazeiro do Norte
Salvador
Dom Pedro
Fortaleza
Fortaleza

CE
SP
CE
CE
PI
RN
CE
SP
PE
MG
CE
CE
PR
BA
CE
BA
MA
CE
CE

Ano Internacional de Qumica


Olimpada Brasileira
de Qumica 2011

Resultados
Juliana Leite Campelo
Mauroclio Rocha Pontes Filho
Samuel Palcio de Oliveira
Guilherme de Freitas Rodrigues
Janderson B. da Fonseca Jnior
Leonardo Gregrio de Andrade
Bruno Davi Rocha
Lorayne Lino Sousa
Lucas Felipe Albuquerque Lins
Maria Mariana Barros de Vasconcelos
Eliabe Bastos Dias
Luan Lincoln Cunha Ribeiro
Ana Letcia Gondim de Carvalho
Ana Luiza Viana Pequeno
Andressa da Silva Figueiredo
Antnio Bruno . de Holanda Lima
Artur Souto Martins
Felcio Holanda Moreira
Isabella Mayumi de Souza Matsura
Larissa Fonseca Chaves
Pedro Jorge L. Alves Cronemberger
Pedro Santos Barbosa
Pedro Souza Faria
Rodrigo Silva de Andrade
Thiago Assis Borges Morais
Victor Almeida Costa
Paulo Vitor Barreiro Gidi
Rene Bezerra Moreira
Francisco Grigore de Amorim Jnior
Luis Cludio Magalhes de Holanda
Alrio Alberto Furtado Corts
Giovanni Elson Rafael de Souza
Gustavo Henrique dos Santos
Izael Francisco de Brito Arajo
Andrey Jhen Shan Chen
Emanuel Gabriel Teodoro

Santa Ceclia
Farias Brito
Paraso
Ari de S Cavalcante
Notre Dame de Lourdes
Militar de Juiz de Fora
SantAna
Esc. Jardim Crescimento
Ari de S Cavalcante
Ari de S Cavalcante
7 de Setembro
Mater Christi
Farias Brito
7 de Setembro
Farias Brito
Centro Educ.Adalberto Valle
Ari de S Cavalcante
Farias Brito
Bandeirantes
Ass. Educ. Profa. Noronha
Sagrado Corao de Jesus
COESI
Farias Brito
Santo Antnio de Jesus
Dom Barreto
Farias Brito
Sartre COC
Farias Brito
Militar de Braslia
Dulia Bringel
Prevest
Luza Tvora
COESI
U.E. Teotnio F. Brando
Etapa
Polcia Militar do Paran

Fortaleza
Sobral
Juazeiro do Norte
Fortaleza
Cuiab
J. de Fora - MG
Sobral
So Lus
Fortaleza
Fortaleza
Fortaleza
Mossor
Fortaleza
Fortaleza
Fortaleza
Manaus
Fortaleza
Fortaleza
So Paulo
Dom Pedro
Teresina
Aracaju
Fortaleza
S. Ant. de Jesus
Teresina
Fortaleza
Salvador
Fortaleza
Braslia
Fortaleza
Goiania
Fortaleza
Aracaju
Cocal do Alves
Campinas
Curitiba

CE
CE
CE
CE
MT
MG
CE
MA
CE
CE
CE
RN
CE
CE
CE
AM
CE
CE
SP
MA
PI
SE
CE
BA
PI
CE
BA
CE
DF
CE
GO
CE
SE
PI
SP
PR

Ano Internacional de Qumica



| 37

Programa Nacional
Olimpadas de Qumica

IV OBQ Jnior
Henrique Martinez Rocamora
Luiz David Botero Alessi
Artur Melo Catunda
Bruno Almeida Costa
Guilherme Anitele Silva
Juliana Ramos Teixeira Bonfim
Mariana Lima Vieira
Mateus Almeida Farias dos Santos
Mateus Freire Bezerra
Nathlia Maria Fonseca Fres
Paulo Roberto Pereira de F. Filho
Thamires Stoppelli Ribeiro
Carlos Augusto Jardim Chiarelli
Igor Viana Ferreira
Joo Pedro Veras Muniz Farias
Matheus Bittencourt Braga
Bruno Gomes Ribeiro
Rafael da Costa Araripe
Arthur Carneiro Moura
Caio Felipe Siqueira Gomes
Deborah Giovanna Santana Rabelo
Enrico Pascucci Loffel
Fernando Antnio S. de Arago Filho
Flora Morgana C. do Bonfim Gorender
Glicia Rodrigues Ferreira
Gustavo Gentil Leite de Arajo
Ilzane Maria de Oliveira Morais
Isabella Freitas Figueiredo
Joo Maurcio Almeida Nos
Joo Paulo Mota Telles
Josu Silva Colho de Oliveira
Juliana Sampaio Saraiva de Oliveira
Lucas Almeida Linhares
Mrlon Neres de Moura
Matheus Carioca Sampaio
Matheus de Almeida Costa
38 |

Etapa
Bom Jesus N. S de Lourdes
Batista Santos Dumont
Farias Brito
Anglo Prudentino
Santo Antnio de Jesus
Educandrio Christus
Dom Barreto
Mater Christi
Santo Antnio de Jesus
Farias Brito
Farias Brito
E.M.E.F. Profa. Arlinda R. Negri
Magnum Cidade Nova
Ari de S Cavalcante
Militar de Porto Alegre
Deciso Anglo - ICM
Petroplis
Farias Brito
7 de Setembro
Farias Brito
Petroplis
7 de Setembro
Anchieta
Master
Santa Ceclia
Santa Ceclia
Magnum Agostiniano
Petroplis
Anchieta
Dom Barreto
Master
Farias Brito
Curso Pensi - Ponto de Ensino
Farias Brito
7 de Setembro

So Paulo
Curitiba
Fortaleza
Fortaleza
Junqueirpolis
S. Ant. de Jesus
Piripiri
Teresina
Mossor
S. Ant. de Jesus
Fortaleza
Fortaleza
Dumont
Belo Horizonte
Fortaleza
Porto Alegre
Icm
S. B. do Campo
Fortaleza
Fortaleza
Fortaleza
S. B. do Campo
Fortaleza
Salvador
Fortaleza
Fortaleza
Fortaleza
Belo Horizonte
S. B. do Campo
Salvador
Teresina
Fortaleza
Fortaleza
Rio de Janeiro
Fortaleza
Fortaleza

SP
PR
CE
CE
SP
BA
PI
PI
RN
BA
CE
CE
SP
MG
CE
RS
SP
SP
CE
CE
CE
SP
CE
BA
CE
CE
CE
MG
SP
BA
PI
CE
CE
RJ
CE
CE

Ano Internacional de Qumica


Olimpada Brasileira
de Qumica 2011

Resultados

Nair Cammila Bencio de L. Lopes


Teleyos
Fortaleza
CE
Paulo Davi Borges Esteves
Farias Brito
Fortaleza
CE
Ricardo Patrcio Honorato Almeida
Farias Brito
Fortaleza
CE
Sarah Barreto Ornellas
Anchieta
Salvador
BA
Tino Miro Aurlio Marques
Ari de S Cavalcante
Fortaleza
CE
Gustavo Henrique A. B. Melo
Boa Viagem
Recife
PE
Lucas Cavalcante do Nascimento
Farias Brito
Fortaleza
CE
Lucas Eneas Gomes Pinheiro
Farias Brito
Fortaleza
CE
Pedro Angelo Vaz de Carvalho
Bernolli
Belo Horizonte
MG
Thayana Taynara Andrade dos Santos Santo Antnio de Jesus
S. Ant. de Jesus
BA
Italo Lesione de Paiva Rocha
Master
Fortaleza
CE
Phablo Wemeson Figueiredo de Souza Paraso
J. do Norte
CE
Ana Luiza Nogueira Moror
Anchieta
Salvador
BA
Ana Valria Gonalves Torres Incio
Paraso
J. do Norte
CE
Giovanna Carvalho F. Figueirdo
Monteiro Lobato
J. do Norte
CE
Giselle Furtado Silva
SantAna
Sobral
CE
Joo Pedro Izidrio Lima
Lavoisier - Anglo
Teresina
PI
Joo Lucas Miranda Francelino
Ari de S Cavalcante
Fortaleza
CE
Joilson de Jesus Barreto Jnior
Santo Antnio de Jesus
S. Ant. de Jesus
BA
Lucas Resende
Esc. Municipal Paula Assis
Resende de Costa MG
Andr Miranda Onofre
Etapa
So Paulo
SP
Alcia Mouro Vieira
Ari de S Cavalcante
Fortaleza
CE
Beatriz Prazeres Cmara
Boa Viagem
Recife
PE
Caio Matheus Teixeira Brito
Sistema Elite de Ensino
Belem
PA
Carlos Augusto F. F. de Carvalho
Anchieta
Salvador
BA
Daniel Crisstomo Wainstock
A. Liessin
Rio de Janeiro
RJ
Felipe Pinheiro Mendes
Santa Ceclia
Fortaleza
CE
Gabriel Maia Bezerra
Batista Santos Dumont
Fortaleza
CE
Gabriella Brando Pita
Sartre COC
Salvador
BA
Giselle Silvestre de Jesus
Paraso
J. do Norte
CE
Heitor Augusto Serafim
Petroplis
S. B. do Campo
SP
Iago Henrique Gomes Silva de Jesus
Centro de Excelen. Master
Aracaju
SE
Igor Lobato do Nascimento
Brigadeiro Newton Braga
Rio de Janeiro
RJ
karo Vinhas Fernandes
Farias Brito
Fortaleza
CE
Joo Felipe Souza Ribeiro
Anchieta
Salvador
BA
Lucas Cunha Agustini
Petroplis
S. B. do Campo
SP
Lucas de Freitas Martins
Anchieta
Salvador
BA
Ano Internacional de Qumica

| 39

Programa Nacional
Olimpadas de Qumica

IV OBQ Jnior
Lucas Levy Alves Moraes
Lucas Pinheiro de Marchi
Luciano Costa Brito
Luiza Ferreira Lopes
Miguel Torrres de Azevedo Maia
Paula Fernanda P. Teixeira de Oliveira
Pedro Lucas de Souza Silveira
Renan Nogueira Pinto
Rodrigo Resende de Vasconcelos
Thaissa Lannes Paula Souza
Vanessa Pereira de Souza
Vitor Dias Gomes Barrios Marin
Andressa Paula Silva Coelho
Bruno Cicone de Almeida
Raquel Souza Caminha Bret
Dbora Lusa Bez
Jlio Csar Ferreira Filho
Pedro Coleto Beserra dos Anjos
Pedro Teotnio de Sousa
Abner Pinto Soares
Daniela de Sousa Gomes
Gisele Christine Araujo Nascimento
Miguel Woodall Silva de Assis
Joo Vitor Rocha Almeida
lvaro Pessoa Menezes
Amanda Ribeiro Fonseca de Souza
Filipe Padilha de Miranda
Juliane Maria Moreira Aguiar
Leonardo da Silva Oliva
Lucas Rodrigues Arajo
Rhanyelle Pereira de Jesus
Sarah Roque Sanz
Amanda Gomes de Vasconcelos
Giovanna Coimbra Porto
Izadora Maria Leal Gomes
Joo Victor Bezerra Firmiano
40 |

Sagrado Corao de Jesus


Ari de S Cavalcante
U.E.B. Alfredo Silva
Bandeirantes
Prevest
Ari de S Cavalcante
Mater Christi
Farias Brito
Master
Notre Dame de Lourdes
Militar de Juiz de Fora
Anglo Prudentino
Dom Barreto
Objetivo Unidade Aquarius
Farias Brito
Dulia Bringel
Farias Brito
Integral de Atibaia
Ari de S Cavalcante
Salesiano S. Jos
Farias Brito
Educandrio Christus
Coleguium
Lavoisier - Anglo
COESI
Batista Getsmani
Boa Viagem
SantAna
Deciso - Anglo
Lavoisier - Anglo
U.E.B. Alfredo Silva
Prevest Sul
Farias Brito
Prevest
Ass. Educ. Profa. Noronha
Farias Brito

Teresina
Fortaleza
Pao do Lumiar
So Paulo
Goiania
Fortaleza
Mossor
Fortaleza
Aracaju
Cuiab
Juiz de Fora
Junqueirpolis
Teresina
S. J. dos Campos
Fortaleza
Fortaleza
Fortaleza
S. Ant. de Jesus
Fortaleza
Natal
Fortaleza
Piripiri
Belo Horizonte
Salvador
Aracaju
Belo Horizonte
Recife
Sobral
Nova Granada
Teresina
Pao do Lumiar
Goiania
Fortaleza
Goiania
Dom Pedro
Fortaleza

PI
CE
MA
SP
GO
CE
RN
CE
SE
MT
MG
SP
PI
SP
CE
CE
CE
BA
CE
RN
CE
PI
MG
BA
SE
MG
PE
CE
SP
PI
MA
GO
CE
GO
MA
CE

Ano Internacional de Qumica


Olimpada Brasileira
de Qumica 2011

Resultados
Stfania de Figueiredo Coutinho
Raquel Paes Patrcio T. Ferreira
Dbora Romo Pinto Santos
Otvio Cunha da Silva
Robson Camilotti Slompo
Alexandre Messias Mendes Filho
Alexya Nunes Fonsca
Amanda de Souza Mota
Ana Cecilia Venncio Venceslau
Ana Lusa Lemos Bessa de OLiveira
Andr Lus Moreti Frana
Arthur Felipe Lustosa
Bernardo Barcellos de Castro Cunha
Camilla Rocha de O. Fontoura
Carlos Henrique Jardim Chiarelli
Cntia Andrade Costa Fonsca
Danilo Viana Figueiredo
Francisco Cludio Almeida da S. Jnior
Gabriel Pereira Souza da Silva
Gabriella Fernandes Rampinelli
Joo Marcos Lima Garcia
Joo Pedro Silva Soares
Joo Victor Ferreira da Costa
Jlio Cezar Castro Lins Barroso
Kleberson John Santos de Maria
Leonardo Santamaria Ferraz Souto
Luana Sad Souto Maior
Lucas Moura Andrade
Lus Milani
Marcos Aurlio de Oliveira Jesus
Matheus Nunes Gonsalves
Pedro Kasprzykowski S. Magalhes
Thadeu Henrique C. V. A. S. Costa
Bethania Tavares Barreto dos Reis
Gabriel Griep
Bruno Campos Santos

Inst. Educ. So Jos da Esccia


Brigadeiro Newton Braga
Paraso
Fund. Itab. Dif. do Ens. - FIDE
Polcia Militar do Paran
Petroplis
U.E.B. Alfredo Silva
Deciso Anglo - ICM
7 de Setembro
Farias Brito
Farias Brito
Dom Barreto
Farias Brito
Brigadeiro Newton Braga
E.M.E.F. Profa. Arlinda R. Negri
Mater Christi
Sete de Setembro - COC
Ari de S Cavalcante
Curso Opo
Gabriel Chalupe
Ass. Educ. Profa. Noronha
Sagrado Corao de Jesus
E.M.E.F. Monteiro Lobato
Centro Educ.Adalberto Valle
EEEFM Solon de Lucena
Jardim So Paulo
Sartre COC
Lavoisier - Anglo
Bandeirantes
Bandeirantes
Deciso Anglo - ICM
Santa Ceclia
Pedro II UEEN II
Militar de Juiz de Fora
Maria Auxiliadora
Magnum Agostiniano

Poos de Caldas
Rio de Janeiro
J. do Norte
Itabira
Curitiba
S. B. do Campo
Pao do Lumiar
Icm
Fortaleza
Fortaleza
Fortaleza
Teresina
Fortaleza
Rio de Janeiro
Dumont
Mossor
Poos de Caldas
Fortaleza
Camaragibe
Barueri
Dom Pedro
Teresina
So Paulo
Manaus
Camp. Grande
So Paulo
Salvador
Teresina
So Paulo
So Paulo
Icm
Fortaleza
Rio de Janeiro
Juiz de Fora
Canoas
Belo Horizonte

MG
RJ
CE
MG
PR
SP
MA
SP
CE
CE
CE
PI
CE
RJ
SP
RN
MG
CE
PE
SP
MA
PI
SP
AM
PB
SP
BA
PI
SP
SP
SP
CE
RJ
MG
RS
MG

Ano Internacional de Qumica



| 41

Programa Nacional
Olimpadas de Qumica

IV OBQ Jnior
Erick Alves Moitinho
Nara Lvia Carlos de Castro Pinheiro
Tawana Cntia Henrique de Lima
Ana Maria Juc Novaes Ramalho
Arthur Guy Furtado Colfort
Luis Fernando Freire Figueiredo
Camille de Moura Andrade
Joo Gabriel Matos Serra
Letcia Gonalves Caldeira
Lucas Andrade de Toledo
Mariana Moraes Pacheco
Ndia Maria Pires Silva
Paulo Ddimo Camura V. Filho
Rafael Hiplito de Farias
Renata Oliveira Batista
Rodrigo Sobral de Athayde Rocha
Vitor Melo Rebelo
Viviane Barbosa Aires Leal
Leonardo Gomes Gonalves
Gabriel Albuquerque
Gabriel Oliveira Martins
Lusa Luzardo Starling
Renata de Oliveira Melo
Susana Silva Lima
Patrick Castelo B. Ramada Campos
Thayara Aline Nadal Afonso
Victor Hugo Pinheiro Santos
Antnio Romilson Pires Rodrigues
Brenda Bezerra Vasconcelos
Bruno Vilas Boas Fahel
Caio Luis Pinheiro Arenas
Cristian Philippe B. Dias Eichstaedt
Eduardo Geraldo Assis
Felipe Kutait
Francisco Arthur Bomfim Azevedo
Gabriel Prisco Diogo de Holanda
42 |

E.M.E.B - Gov. Andr F. Montoro


Mater Christi
Dulia Bringel
Ari de S Cavalcante
Dulia Bringel
Bernolli
Ari de S Cavalcante
Ass. Educ. Profa. Noronha
Magnum Agostiniano
Anchieta
Magnum Agostiniano
Glucia Costa
Santa Ceclia
Farias Brito
U.E. Teotnio F. Brando
Anchieta
Dom Barreto
Dulia Bringel
Militar de Braslia
Organ. Educ. Lopes Pereira
Dulia Bringel
Galois
Coleguium
Educandrio Christus
Ari de S Cavalcante
Centro Educ.Adalberto Valle
Lavoisier - Anglo
E.F. Padre Jos Roberto
Farias Brito
Anchieta
Anchieta
Jardim So Paulo
Esc. Municipal Paula Assis
So Jos
Ari de S Cavalcante
Farias Brito

Valinhos
Mossor
Fortaleza
Fortaleza
Fortaleza
Belo Horizonte
Fortaleza
Dom Pedro
Belo Horizonte
Salvador
Belo Horizonte
Timon
Fortaleza
Fortaleza
Cocal do Alves
Salvador
Teresina
Fortaleza
Braslia
Fortaleza
Fortaleza
Braslia
Belo Horizonte
Piripiri
Fortaleza
Manaus
Teresina
Martinpole
Sobral
Salvador
Salvador
So Paulo
R. de Costa
Bauru
Fortaleza
Fortaleza

SP
RN
CE
CE
CE
MG
CE
MA
MG
BA
MG
MA
CE
CE
PI
BA
PI
CE
DF
CE
CE
DF
MG
PI
CE
AM
PI
CE
CE
BA
BA
SP
MG
SP
CE
CE

Ano Internacional de Qumica


Olimpada Brasileira
de Qumica 2011

Resultados
Joo Vitor Bueno Peixoto
Lorena Oliveira de Andrade
Lucas Henrique Campos Mendes
Lus Felipe Vieira Soares Barradas
Newton Bezerra de Oliveira
Rodrigo Zanette de Magalhes
Ruan Machado Coelho Rossato
Victor H. Mirkhan
Victor Paulo Nery Ribeiro
Victria Freitas Vieira da Cunha
Vincius Takiuti Miura
Lucas T. Kabayashi
Victor Barbosa do Monte
Ana Beatriz de Queiroz Leite
Ian Zaerorosin Quadros Vieira
Isabela Marques
Paulo Victor Cordeiro Alves
Pedro B. Belisario
Thalys Xavier Batista
Walter Ulbrich Neto
Yan Cruz Fernandes
Yuri de Souza Dutra
Alcsia Madeiro Martins
Caio Jos Prado
Gabriel Gomes dos Santos
Amanda Ferreira Brasil
Ana Caroline Sales e Souza
Arthur Oliveira Vale
Beatriz Vaz Sampaio de Carvalho
Bianca Arago Silva
Caio Teixeira de Queiroz
Camila Machado Figueiredo Menezes
Carlos Alberto Gomes da Costa Jnior
Carlos Henrique M. Saboia Pompeu
Danilo Maciel Arajo
David Azevedo

Bandeirantes
Dom Barreto
Paulo Freire
Dom Barreto
Ari de S Cavalcante
Etapa
Etapa
Augusto Laranja
Centro Educ.Adalberto Valle
Paraso
UNISUZ
So Jos
Centro Educ.Adalberto Valle
NEC - Ncleo de Educ. Criativo
Coleguium
Anchieta
Farias Brito
Bernolli
Farias Brito
E. M. Prof. Edgar M. Castanheira
xito do Cariri
Coleguium
Inst N. S. Auxiliadora
Farias Brito
Certus
Educare Rede Pitgoras
Farias Brito
Militar do Recife
Anchieta
Farias Brito
Master
Militar de Braslia
Motiva Ambiental
SantAna
Teleyos
Anchieta

So Paulo
Teresina
Jundia
Teresina
Fortaleza
So Paulo
So Paulo
So Paulo
Manaus
J. do Norte
Suzano
Bauru
Manaus
Natal
Belo Horizonte
Salvador
Fortaleza
Belo Horizonte
Fortaleza
Joinvile
Fortaleza
Belo Horizonte
Baturit
Fortaleza
So Paulo
Betim
Fortaleza
Recife
Salvador
Fortaleza
Fortaleza
Braslia
Joo Pessoa
Sobral
Fortaleza
Salvador

SP
PI
SP
PI
CE
SP
SP
SP
AM
CE
SP
SP
AM
RN
MG
BA
CE
MG
CE
SC
CE
MG
CE
CE
SP
MG
CE
PE
BA
CE
CE
DF
PB
CE
CE
BA

Ano Internacional de Qumica



| 43

Programa Nacional
Olimpadas de Qumica

IV OBQ Jnior
Fernanda de Figueiredo Gomes
Gabriel Cicalese Bevilaqua
caro de Azevdo Alexandre
Joo Victor Arajo Dande
Leonardo Victor Costa Lima
Luiz Victrio de Castro Cruz Martins
Marco Enrique dos Santos Abensur
Marlia de Moraes Barros
Matheus Henrique Martins Costa
Myqueias do Nascimento Silva
Odilio Ferreira Lima Neto
Priscilla Dias Lima Vicentini
Raphael ZImmermann Torres
Renan Leite Vieira
Valter Augusto Di Profio Felix
Vanessa Ferreira Baldoino
Vinicius Mendes Barros
Yuri Schenkel Toledo
Deniel Desconzi Moraes
Giancarlo Tomazzoni de Oliveira
Rodrigo Estevam Mendes Dantas
Emanuela Martins Bezerra Soares
Gabriel Toledo Guerra
Gabriela Lucena de A. Romeiro
Gleiciany da Conceio Nunes
Juliana Guimares de M. Furtado
Lucas da Silva Frana
Lucas Melo de Costa
Lucas Silva Braga
Luis Augusto Irineu Aguiar Ramos
Mateus Nunes de Oliveira
Nairton Tavares Lima
Tereza Cristhina de Matos Pereira
Thas Cristina Leite Moraes
Breno Dal Bianco
Isabela Yang
44 |

Magnum Agostiniano
Col. Aplicao da UFPE
Ari de S Cavalcante
Magnum Agostiniano
Educ. Paraso da Cultura
Bandeirantes
Col. Integ. Objetivo
Ass. Educ. Profa. Noronha
Boa Viagem
UE Jos Narciso da R. Filho
Master
Anchieta
Militar de Juiz de Fora
Farias Brito
Bandeirantes
Dom Barreto
de Aplicao - UGB
Paulo Freire
Militar de Braslia
Salesiano Dom Bosco
Batista Santos Dumont
Monteiro Lobato
Deciso Anglo - ICM
Boa Viagem
U.E.B. Alfredo Silva
Sagrado Corao de Jesus
Curso Pensi - Ponto de Ensino
Salesiano S. Jos
Farias Brito
Motiva Jardim Ambiental
Farias Brito
Plos
Ass. Educ. Profa. Noronha
U.E.B. Alfredo Silva
Bom Jesus
Etapa

Belo Horizonte
Recife
Fortaleza
Belo Horizonte
Crato
So Paulo
Mogi das Cruzes
Dom Pedro
Recife
Piripiri
Fortaleza
Salvador
Juiz de Fora
Fortaleza
So Paulo
Teresina
Volta Redonda
Jundia
Braslia
Porto Alegre
Fortaleza
J. do Norte
Icm
Recife
Pao do Lumiar
Teresina
Rio de Janeiro
Natal
Fortaleza
Joo Pessoa
Fortaleza
Iguatu
Dom Pedro
Pao do Lumiar
Curitiba
So Paulo

MG
PE
CE
MG
CE
SP
SP
MA
PE
PI
CE
BA
MG
CE
SP
PI
RJ
SP
DF
RS
CE
CE
SP
PE
MA
PI
RJ
RN
CE
PB
CE
CE
MA
MA
PR
SP

Ano Internacional de Qumica


Olimpada Brasileira
de Qumica 2011

Resultados
Anderson Santos de Freitas
Andr Luiz Nunes Amaru
ngelo Maciel Oliveira
Antnio Emanuel Marques dos Santos
Beatriz Melo de Albuquerque
Bruno Martins Aboud
Bruno Vasconcelos Silva
Eenane Pedro Matos Barros
Emanuel Anselmo N. Segundo
Gabriel Queiroz Moura
Iracema Capistrano Costa Fook
Joo Czar Oliveira C. de Sousa
Juliana Novaes Bueno de Camargo
Letcia Lara Ramos Lima
Letcia Tutihashi
Maria Clara Silveira Gontijo
Pedro Victor Alves Barbosa
Renato Correia Borges Lopes
Sadi Kneipp Neto
Sophia Gaspar C. da S. V. Trvia
Tiemi Beatriz Fernandes Nagazawa
Maria Aline do Nascimento do Carmo
Nolia Yesenia Rojas Cruz
Rassa Maria Bastos Vasconcelos
Caio Souza Pimentel
Jonas Martins de Abreu
Alexandre Satyro de Medeiros
Arthur Medeiros Grigoli
Felipe Anderson Nunes Lima
Gustavo Freitas de Abreu
Igor Tasuke Corra de A. Morimoto
Joo Pedro Marques Azevedo
Mariana Dias Carvalho
Rebecca Peixoto Holanda
Sandro Luis Vieira Colho
Saulo Gonalo Brasileiro

Esc. Rodrigues Alves


Pedro II UEEN II
Santa Ceclia
Teleyos
Master
Augusto Laranja
Farias Brito
Dom Barreto
Sagrado Corao de Jesus
Dom Barreto
Esc. Jardim Crescimento
Farias Brito
Augusto Laranja
Ass. Educ. Profa. Noronha
Col. Integ. Objetivo
Magnum Agostiniano
Sagrado Corao de Jesus
Sistema Elite de Ensino
Farias Brito
Ari de S Cavalcante
Notre Dame de Lourdes
Anchieta
Batista Getsmani
Farias Brito
Farias Brito
Mater Christi
Galois
Ari de S Cavalcante
Integral
Magnum Agostiniano
Esc. Internacional de Aldeia
Marista Dom Silvrio
Militar de Juiz de Fora
Juvenal de Carvalho
U.E.B. Po da Vida
Col. Aplicao da UFPE

Sta. M. do Sul
Rio de Janeiro
Fortaleza
Fortaleza
Fortaleza
So Paulo
Fortaleza
Teresina
Teresina
Teresina
So Lus
Fortaleza
So Paulo
Dom Pedro
Mogi das Cruzes
Belo Horizonte
Teresina
Belem
Fortaleza
Fortaleza
Cuiab
Salvador
Belo Horizonte
Fortaleza
Fortaleza
Mossor
Braslia
Fortaleza
Teresina
Belo Horizonte
Camaragibe
Belo Horizonte
Juiz de Fora
Fortaleza
Pao do Lumiar
Recife

RS
RJ
CE
CE
CE
SP
CE
PI
PI
PI
MA
CE
SP
MA
SP
MG
PI
PA
CE
CE
MT
BA
MG
CE
CE
RN
DF
CE
PI
MG
PE
MG
MG
CE
MA
PE

Ano Internacional de Qumica



| 45

Programa Nacional
Olimpadas de Qumica

IV OBQ Jnior
Vinicius Leonardi Pinheiro
Eduardo Soares e Silva Britto
Eduardo Andreatti Takashe
Vincius Francisco Vieira Ferreira
Anderson Carlos Felix
Bernardo de S Teles Passos
Ed kio Santos Passos
Gabriela Fior Ribeiro
Gleybson Roberto Arajo dos Santos
Gustavo Alencar Silva A. Dantas
Henrique Matos Campos
Jonathan Ivens Silva dos Santos
Lia de Freitas Arajo Alves
Lucas Tadeu Rocha Santos
Mrcio Normando Borges C. Filho
Paula Abiko Navarro Carrion
Rafaela Mayumi Yamasaki
Samantha Alves de Oliveira
Vanessa Pelasini Guimares Mello
Yasmin Pissolate Mattos Bretz
Mathews Correia do Nascimento
Giovana Landal de Almeida Lobo
Joo Paulo Nunes Soares
Alexandre Ribeiro Carneiro
Alysson Faanha Moreira
Ana Beatriz Anchieta Seixas
Breno Di Gregorio
Camila Machado de Arajo
Isabella Rossana Almeida S. Soares
Joo Victor Soares Carvalho
Juan Felipe Siqueira Silva
Larissa Rodrigues Esmeraldo Carneiro
Marina Catunda Pinheiro Juc
Milenna Maclinks de Macedo
Pedro Paixo Borges
Vitria Lara Martins Marques
46 |

Atibaia
Anchieta
Polcia Militar do Paran
Master
Ari de S Cavalcante
Sartre COC
Dom Barreto
Magnum Cidade Nova
Dulia Bringel
Gabriel Chalupe
Col. Pr-Universitrio
Militar de Juiz de Fora
Farias Brito
Farias Brito
Militar de Fortaleza
Charles Darwin
Salesiano So Gonalo
Militar de Braslia
Petroplis
Magnum Cidade Nova
Ari de S Cavalcante
Positivo Jnior
Fundao Bradesco
Acesso
Inst. de Educao Castro
Adventista do Guar
Sanrta Mnica
Esc. Virgem de Lourdes
Boa Viagem
Ari de S Cavalcante
SESI Hanleto Magnavacca
Ari de S Cavalcante
Ari de S Cavalcante
Juvenal de Carvalho
Pedro II UEEN II
Farias Brito

Atibaia
Salvador
Curitiba
Fortaleza
Fortaleza
Salvador
Teresina
Belo Horizonte
Fortaleza
Barueri
Ilha Solteira
Juiz de Fora
Fortaleza
Sobral
Fortaleza
Linhares
Cuiab
Braslia
S. B. do Campo
Belo Horizonte
Fortaleza
Curitiba
Braslia
Feira de Santana
Aquiraz
Braslia
Mogi das Cruzes
Camp. Grande
Recife
Fortaleza
Belo Horizonte
Fortaleza
Fortaleza
Fortaleza
Rio de Janeiro
Fortaleza

SP
BA
PR
CE
CE
BA
PI
MG
CE
SP
SP
MG
CE
CE
CE
ES
MT
DF
SP
MG
CE
PR
DF
BA
CE
DF
SP
PB
PE
CE
MG
CE
CE
CE
RJ
CE

Ano Internacional de Qumica


Olimpada Brasileira
de Qumica 2011

Resultados
Ugor Tomaz Fernadez
Alexandre Mendona Cardoso
Antonio Victor Gouveia A. dos Santos
Caroline Areque Ucha Maciel
Giovanna Maria Pereira de Sampaio
Igor Moreira Eduardo
Letcia Eduarda de Arruda Prado
Rafael Santana Brito
Aline Alves Rodrigues
Anderson Digenes Souza Ricardo
Anne Teresa Damasceno Costa
Geovane Tomaz Batista
Hiago Veras Arajo Soares
Iago Santos Rodrigues
Igor Coelho Barbosa
Ingra Bezerra de Melo Gonalves
Jssica Tenfen Wazilerski
Joo Felipe dos Santos Sales
Joo Henrique Alves Taveira
Joo Marcos Mesquita Viana
Juliana Ferreira de Oliveira
Kevin Saymon A. Santos Cerqueira
Keyla Maria Rodrigues Gomes
Leonardo Cairo Carvalho Neto
Lucas Fernando Barbosa Mendes
Lucas Tavares Costa Meira
Lcia Oliveira Lindso
Luis Arthur Coutinho de Brito
Luiz Matheus Ferreira Fernandes
Marcos Vincius Pitombeira Noronha
Matheus Henrique de Arajo Dutra
Matheus Prado de Paula
Matheus Ribeiro Felippin
Matheus Rocha Barbosa
Mayra Regina Teixeira de Miranda
Milena Scur Wagner

Salesiano S. Jos
Militar de Salvador
Ari de S Cavalcante
Fundao Bradesco
Positivo Jnior
Farias Brito
Polcia Militar do Paran
Militar de Salvador
Col. Integ. Objetivo
Master
Profa. Alice do C. de Melo
Esc. 13 de Junho Objetivo
Dom Barreto
Dom Barreto
Ari de S Cavalcante
7 de Setembro
Marista de Cascavel
Luciano Feijo
Esc. Modelo de Iguatu
UE Jos Narciso da R. Filho
Santo Antnio de Jesus
Jos de Anchieta
Esc. Senador Nilo Coelho
Anchieta
Marista Dom Silvrio
Imaculada Conceio
Farina do Brasil
Ass. Educ. Profa. Noronha
Dom Barreto
Esc. Normal R. de Limoeiro
Prevest
Magnum Agostiniano
Militar de Braslia
Glucia Costa
Sagrado Corao de Jesus
Sebastio de Abreu

Natal
Salvador
Fortaleza
Braslia
Curitiba
Fortaleza
Curitiba
Salvador
Mogi das Cruzes
Fortaleza
So Benedito
Junqueirpolis
Teresina
Teresina
Fortaleza
Fortaleza
Cascavel
Sobral
Iguatu
Piripiri
Sto. Ant. de Jesus
Feira de Santana
Picos
Salvador
Belo Horizonte
Camp. Grande
So Lus
Dom Pedro
Teresina
Lim. do Norte
Goiania
Belo Horizonte
Braslia
Timon
Teresina
Maranguape

RN
BA
CE
DF
PR
CE
PR
BA
SP
CE
CE
SP
PI
PI
CE
CE
PR
CE
CE
PI
BA
BA
PI
BA
MG
PB
MA
MA
PI
CE
GO
MG
DF
MA
PI
CE

Ano Internacional de Qumica



| 47

Programa Nacional
Olimpadas de Qumica

IV OBQ Jnior
Tarik H. Ashmawi
Thays Gomes Vasconcelos Brito
Victor Machado
Vincius Xavier Garcia
Heitor Martins Oliviera
Douglas Rafael Silva Sousa
Felipe Jos de Castro Santos
Gabriela Rebecca Hack
Alexandra Kazumi Kameoka
Gabriela Ferro da Cunha
Isabelli Alves Fernandes
Joo Victor Marques Gomes Rodrigues
Karen Lumi Ii
Lucas Eduardo Porto de Toledo
Marlia de Oliveira Cunha
Mirna Maria Alves Porto
Nathlia Ucha de Castro Bessa
Alan Andr Goldenberg
Ana Carolina Lobato Saldanha
Ana Ceclia Fernandes Cavalcante
Ana Letcia Carvalho Lobo
Anna Heloisa de Vasconcelos
Antonio Jaques Pedreira Neto
Armindo Augusto de N. Albuquerque
urea Soares Zica
Bruna Queiroz Allen Palcio
Caio Machado Vieira Borges
Caio Vitor Saunders Barreto
Camila Rodrigues Ferreira
Eduardo Antnio Dias Cristino
Francisco Alan Holanda Almeida
Guilherme Faro Corra Reis
Gustavo Crubellati Nunes
Ighor Belm Moreira de Oliveira
Isabella Frana Moreira
Italo Bezerra
48 |

Bandeirantes
Farias Brito
Integral
Pedro II UEEN II
Coleguium
So Lucas
Master
Positivo Jnior
Exponente
Pedro II UEEN II
Deciso - Anglo
Marista de Goinia
Pilar do Sul
Objetivo Unidade Aquarius
Esc. Modelo de Iguatu
Frei Joo Pedro VI
Ari de S Cavalcante
Bandeirantes
Marista Dom Silvrio
Farias Brito
Boa Viagem
Farias Brito
Anchieta
N. S. das Neves
Magnum Agostiniano
Farias Brito
Ari de S Cavalcante
Santa Ceclia
Sag. Corao de Maria
Farias Brito
Farias Brito
Liceu de Estudos Integrados
Centro Educ.Adalberto Valle
Farias Brito
Magnum Agostiniano
Esc. Normal R. de Limoeiro

So Paulo
Fortaleza
Teresina
Rio de Janeiro
Belo Horizonte
Picos
Fortaleza
Curitiba
Recife
Rio de Janeiro
Nova Granada
Goiania
Pilar do Sul
S. J. dos Campos
Iguatu
Fortaleza
Fortaleza
So Paulo
Belo Horizonte
Fortaleza
Recife
Fortaleza
Salvador
Natal
Belo Horizonte
Fortaleza
Fortaleza
Fortaleza
Vitria
Sobral
Fortaleza
Aracaju
Manaus
Fortaleza
Belo Horizonte
Lim. do Norte

SP
CE
PI
RJ
MG
PI
CE
PR
PE
RJ
SP
GO
SP
SP
CE
CE
CE
SP
MG
CE
PE
CE
BA
RN
MG
CE
CE
CE
ES
CE
CE
SE
AM
CE
MG
CE

Ano Internacional de Qumica


Olimpada Brasileira
de Qumica 2011

Resultados
Jair Adhonai Correia dos Santos
Jos Wellington Saraiva Sousa Jnior
Joyce dos Santos Monteiro
Lara Maria Alexandre de Arajo
Lucca Severo Furtado
Nicolas Martins Aguiar
Ricardo Amaral Mendes da Conceio
Vivian Barbosa da Silva
Gabriel Ferreira Lopes
Marcos Gabriel Lima da Costa
Francisco Mauro Falco M. Filho
Jos Otvio de Oliveira Vidal
Paulo Bruno de Sousa Cardoso
Rafael Maest Prado
Rafaela de Albuquerque Cunha
Samantha Souza Napoleo
Vitor Borges Cataldo
Beatriz Paula da Rocha
Carina Lobarinhas Correia
Ricardo Ribeiro da Cruz
Abner Nbrega Maia Aires
Bruna Greenhalgh de M. Ebrahim
Caio Santos Cabral
Carlos Felipe L. de Moraes
Isabella Coelho Ferraz
Lucas Renato Alberti Franco
Orlando Wozmiok de L. Nogueira
Pedro Rodrigues Machado dos Santos
dan Barbosa Ribeiro
Alexandre Seung Yung Kwon
Allan Batista Marton
Amanda Lima de Jesus
Ana Marta V. R. Furtado Estanislau
Andreza Ferreira Malta
Beatriz Rafaela Rodrigues Leito
Bruna Drummond de Moura Miguez

Geo Sul
Ari de S Cavalcante
Contato
Esc. Virgem de Lourdes
N. S. das Neves
Farias Brito
OLIMPO - DF
Dulia Bringel
E. M. .E. F. Profa. Iracema..
Acesso
SantAna
Militar de Manaus
Luza Tvora
Objetivo Unidade Aquarius
Farias Brito
Ari de S Cavalcante
Curso Pensi - Ponto de Ensino
Alfa
Marista So Jos
Pilar do Sul
Farias Brito
Dulia Bringel
Dulia Bringel
Etapa
Etapa
Polcia Militar do Paran
Positivo Jnior
U.E. Teotnio F. Brando
Dom Barreto
Bandeirantes
Educare Rede Pitgoras
Esc. Tcnica Walter Belian
Anchieta
Magnum Agostiniano
Ass. Educ. Profa. Noronha
Magnum Agostiniano

Joo Pessoa
Fortaleza
Macei
Camp. Grande
Natal
Fortaleza
Braslia
Fortaleza
Lindia
Feira de Santana
Sobral
Manaus
Fortaleza
S. J. dos Campos
Sobral
Fortaleza
Rio de Janeiro
Duque de Caxias
Rio de Janeiro
Pilar do Sul
Fortaleza
Fortaleza
Fortaleza
So Paulo
So Paulo
Curitiba
Curitiba
Cocal do Alves
Teresina
So Paulo
Betim
So Paulo
Salvador
Belo Horizonte
Dom Pedro
Belo Horizonte

PB
CE
AL
PB
RN
CE
DF
CE
SP
BA
CE
AM
CE
SP
CE
CE
RJ
RJ
RJ
SP
CE
CE
CE
SP
SP
PR
PR
PI
PI
SP
MG
SP
BA
MG
MA
MG

Ano Internacional de Qumica



| 49

Programa Nacional
Olimpadas de Qumica

IV OBQ Jnior
Charles Allan Duarte Lima Filho
Daniel Cordeiro de Alencar Filho
Daniel Ponte Frota
Dvila Lorraine Marques e Silva
Diego Costa de Freitas
Eduardo Matos Linhares
Eduardo Rgis Barreira de Figueirdo
Flvio Meireles Moura Pessoa
Flvio Tvora Dix-huit Rosado Ventura
Gabriel Caetano Visconti
Gabriel Viana Figueiredo
Gladys Dantas Borges
Guilherme Roberto Naves Miranda
Gustavo Ferreira da Silva Montes
Gustavo Gomes Arruda
Hlio Mendes de Carvalho C. Lima
Hrculles Teixeira Silva
Jade da Costa Wang
Joo Pedro Aparecido da Silva
Joo Ricardo do Egito S. Gonalves
Julio Csar Lopes da Silva
Karolina Kotsuho
Lara Rocha Sobral
Luana Carvalho Mendes
Luana de Lima Matoso
Lucas Filipe Sousa Ramos
Luiz Jos Bezerra Pinheiro
Maria Teresa dos Santos Silva
Mariane de Jesus Arajo
Marina Rajo Santiago Maciel
Mayara Cristina Siqueira Lima
Olavo Carvalho Haase
Pedro Henrique Santana Sousa
Rafael Fontes de Carvalho Amorim
Raquel Gurgel Lins Ramalho
Ricardo Kaoru Komeso
50 |

Ari de S Cavalcante
Paraso
Farias Brito
E. M. Padre Ccero de Castro
Reino do Saber
Sagrado Corao de Jesus
Ari de S Cavalcante
Santa Ceclia
Marista de Natal
Bandeirantes
Sete de Setembro - COC
Profa. Alice do C. de Melo
Galileu
de Aplicao - UGB
Centro Educ. C. El Shadai
Sagrado Corao de Jesus
Educandrio Christus
Marta Falco
Esc. Profissional Dom Bosco
Parthenon
Esc. Tcnica Walter Belian
Lettera
Farias Brito
Certo
Marista Dom Silvrio
E.E.F. Profa Estefnica Matos
Farias Brito
Magnum Agostiniano
SantAna
Magnum Agostiniano
Integral
Coleguium
Sagrado Corao de Jesus
Dom Barreto
Farias Brito
Objetivo - Cantareira

Fortaleza
J. do Norte
Sobral
Teresina
Piquet Cordeiro
Teresina
Fortaleza
Fortaleza
Natal
So Paulo
Poos de Caldas
So Benedito
Anpolis
Volta Redonda
Santo Andr
Teresina
Piripiri
Manaus
Poos de Caldas
Guarulhos
So Paulo
Teresina
Fortaleza
Teresina
Belo Horizonte
Itapaj
Fortaleza
Belo Horizonte
Sobral
Belo Horizonte
Teresina
Belo Horizonte
Teresina
Teresina
Fortaleza
So Paulo

CE
CE
CE
PI
CE
PI
CE
CE
RN
SP
MG
CE
GO
RJ
SP
PI
PI
AM
MG
SP
SP
PI
CE
PI
MG
CE
CE
MG
CE
MG
PI
MG
PI
PI
CE
SP

Ano Internacional de Qumica


Olimpada Brasileira
de Qumica 2011

Resultados
Rodolfo Eduardo Santos Carvalho
Teresa Virgnia Neves Floriano
Thiago Lucas Oliveira Honorato
Vanessa Gabrielly Pedrosa Dantas
Wendel Matos da S. Pedreira
Gabriel Santos Gonalves
Paula Ladeia Barros
Pedro Ribeiro da Costa
Arthur de Melo Barbosa Bittencourt
Camila Faria Garcia
Carlos Antnio Canabrava Jnior
Carlos Henrique Arajo Bezerra
Dhouglas Lucena Arajo
Emilly Amorim Dantas
Felipe Prisco Diogo de Holanda
Giancarlo M. Massoni
Igor Teles Fortuna
Isadora de Lima Santos
Joo Victor Morais do Couto
Jlia Nogueira Guedes Monteiro
Ktia da Silva Souza
Las Martins Queiroz
Matheus Machado Diniz
Melquesedeque Ferreira Morais
Thalys Heriqui Andrade da Silva
Tom Seichi Da Nbrega Guenka
Ury Matheus Serpa de F. Ramalho
Gabriella Maria Radke Chaves
Eva Alice da Silva Pinheiro
Ebenzer Pinto Bandeira Neto
lvaro Brcio A. Almeida e Silva
Ana Beatriz Valdivino Calixto de Aguiar
Augusto Csar O. Serra Pinto Filho
Bianca Desidrio Matos Jacana
Camilla da Rocha Bezerril
Catarina Carvalho Haase

Salvador
Amadeus
Ari de S Cavalcante
Integral
Gnesis
Jos de Anchieta
Anchieta
Anchieta
Magnum Agostiniano
Inst. Educ. S. Jos da Esccia
Marista de Goinia
Ari de S Cavalcante
Ari de S Cavalcante
Esc. Jardim Crescimento
Farias Brito
Atibaia
Santa Ceclia
Magnum Cidade Nova
Magnum Agostiniano
Damas da Instruo Crist
Plos
Prevest
Anchieta
Batista Santos Dumont
Ari de S Cavalcante
Galois
Militar do Recife
E. M. Anna Maria Harger
SESI Hanleto Magnavacca
Ari de S Cavalcante
Farias Brito
Farias Brito
Motiva Ambiental
Farias Brito
Integral
Coleguium

Aracaju
Aracaju
Fortaleza
Teresina
Feira de Santana
Feira de Santana
Salvador
Salvador
Belo Horizonte
Poos de Caldas
Goiania
Fortaleza
Fortaleza
So Lus
Fortaleza
Atibaia
Fortaleza
Belo Horizonte
Belo Horizonte
Recife
Iguatu
Goiania
Salvador
Fortaleza
Fortaleza
Braslia
Recife
Joinvile
Belo Horizonte
Fortaleza
Fortaleza
Fortaleza
Joo Pessoa
Fortaleza
Teresina
Belo Horizonte

SE
SE
CE
PI
BA
BA
BA
BA
MG
MG
GO
CE
CE
MA
CE
SP
CE
MG
MG
PE
CE
GO
BA
CE
CE
DF
PE
SC
MG
CE
CE
CE
PB
CE
PI
MG

Ano Internacional de Qumica



| 51

Programa Nacional
Olimpadas de Qumica

IV OBQ Jnior
Dimas Macedo de Albuquerque
Eduardo dos Santos Oliveira
Eullio Sotero Galvo Jnior
Gabriela Guimares R. dos Santos
Gabrielle Soares Salomo
Guilherme Braga de Paula
Jhones Gonalves de Aguiar
Jos Gabriel Paes Santana
Josu Fernandes de Oliveira Jnior
Jlia Carolina Madeira Boffa
Marcos Vinicius da Silva
Marina Santos Moulin
Matheus de Oliveira Camelo
Otvio Augusto Scariotto
Oton Gonalves de S Neto
Pedro Henrique da Silva Abreu
Rafael de Almeida Baziotti
Rodrigo Coelho Leite
Tiago Rodrigus Seniuk
Victor Lander Arajo Bandeira
Victor Matheus de Lima
Vitor Rodrigues Barbosa

52 |

Farias Brito
CEMP- C. de Ed. Mpal de Pacajus
Integral
Bandeirantes
So Jos
Farias Brito
E.E.E.F.M. Gisela S. Fayet
Amadeus
Inst. de Educao Castro
Parthenon
Marista de Cascavel
Magnum Agostiniano
N. S. das Neves
Marista de Cascavel
Esc. Jardim Crescimento
Lettera
Sete de Setembro - COC
Magnum Agostiniano
Col. Est. N. S. Ftima
Farias Brito
Militar de Manaus
Charles Darwin

Fortaleza
Pacajus
Teresina
So Paulo
Pouso Alegre
Fortaleza
Dom. Martins
Aracaju
Aquiraz
Guarulhos
Cascavel
Belo Horizonte
Natal
Cascavel
So Lus
Teresina
Poos de Caldas
Belo Horizonte
Iva
Fortaleza
Manaus
Vila Velha

CE
CE
PI
SP
MG
CE
ES
SE
CE
SP
PR
MG
RN
PR
MA
PI
MG
MG
PR
CE
AM
ES

Ano Internacional de Qumica


Olimpada Brasileira
de Qumica 2011

Exames Mod. A

Olimpada Brasileira de Qumica - 2011


MODALIDADE A ( 1 e 2 anos )

PARTE A - QUESTES MLTIPLA ESCOLHA

QUESTO 1
comum encontrarmos objetos que brilham no escuro, particularmente,
brinquedos de crianas. Tais objetos podem apresentar o sulfeto de zinco
em sua constituio. Este fenmeno ocorre em razo de que alguns eltrons
destes tomos absorvem energia luminosa e com isso saltam para nveis de
energia mais externos. Esses eltrons retornam aos seus nveis de origem,
liberando energia luminosa e, no escuro, possvel observar o objeto brilhar.
Essa caracterstica pode ser explicada considerando o modelo atmico proposto por:
a) Thomson
c) Bohr
e) Planck

b) Rutherford
d) Marie Curie

QUESTO 2
At 1982, a presso padro era tomada como uma atmosfera (1 atm ou
101325 Pa) e a temperatura como 0 C (273,15 K) e, portanto, o volume molar de um gs nas CNTP era 22,4 L/mol. A partir de 1982, a Unio Internacional de Qumica Pura e Aplicada (IUPAC) alterou o valor da presso padro, de
forma que as novas condies normais de temperatura e presso passaram a
ser: t = 0 C ou T = 273,15 K e p = 100.000 Pa ou 1 bar. Assim, o valor recomendado hoje pela IUPAC, para o volume molar :
a) Vm = 0,021631 m3 mol -1
c) Vm = 0,022711 m3 mol -1
e) Vm = 0,027211 m3 mol -1

b) Vm = 0,035845 m3 mol -1
d) Vm = 0,035745 m3 mol -1

Ano Internacional de Qumica



| 53

Programa Nacional
Olimpadas de Qumica

OBQ - 2011 Fase III


QUESTO 3
Dentre os principais fatores que influenciam na produo agropecuria, podemos citar: o clima, o material gentico, o manejo de pragas, as doenas e
plantas daninhas e o manejo do solo, com especial nfase no manejo qumico como base para a nutrio das plantas. Em razo da produo de alimentos em escala cada vez maior, os nutrientes do solo que do vida s plantas
vo se esgotando. Para supri-los, produtos qumicos conhecidos como fertilizantes so incorporados terra em quantidades crescentes. Para correo da
acidez do solo, o procedimento de rotina a calagem atravs da incorporao de um xido bsico. correto afirmar que esse xido bsico pode ser:
a) MgO2 b) CaO
c) SO2 d) NaO
e) CO

QUESTO 4
A porcentagem de lcool adicionado gasolina regulamentada por Lei, e
recentemente foi estabelecido um novo padro que de 18 a 24% (volume/
volume). Quando 50 mL de gua forem misturados a 50 mL de gasolina comercializada nos postos de servio com o mximo teor permitido de lcool,
ser observada a formao de:
a) 100 mL de uma mistura homognea
b) Duas fases de 50 mL cada
c) Duas fases, sendo a mais densa de 38 mL
d) Duas fases, sendo a mais densa de 62 mL
e) Duas fases sendo a mais densa de 74 mL

QUESTO 5

Supondo que a seguinte reao ocorra por meio de um mecanismo de uma


nica etapa elementar nas duas direes:
A + 2B
AB2
Sendo k1 e k2 as constantes, a uma determinada temperatura, para as reaes
direta e inversa, respectivamente, assinale a alternativa correta.
Ano Internacional de Qumica
54 |

Olimpada Brasileira
de Qumica 2011

Exames Mod. A
a) k1/k2 = k = [AB2]/[A][B]2
b) k1/k2 = k = [A][B]2/[AB2]
c) k2/k1 = k = [AB2]2/[A][B]
d) k2/k1 = k = [A][B]2/[AB2]2
e) k2 + k1 = k = [AB2]/[A][B]2
QUESTO 6
O volume de cido ntrico 0,1 mol.L-1 necessrio para neutralizar uma mistura
0,40 g de hidrxido de sdio e 1,71 g de hidrxido de brio
a) 20 mL
b) 30 mL
c) 50 mL
d) 200 mL
e) 300 mL

QUESTO 7
O etanol anidro, ou seja, etanol isento de gua, pode ser obtido a partir do
etanol 96 GL por tratamento com cal virgem CaO. A cal virgem reage com
a gua conforme a equao abaixo, desidratando o etanol.
CaO(s) + H2O (dissolvido em lcool) Ca(OH)2(s).
Sobre esse processo CORRETO afirmar que:
a) o hidrxido de clcio formado reage com o etanol
b) o xido de clcio reage com etanol para retirar a gua
c) o hidrxido de clcio formado pode ser separado por filtrao
d) a mistura obtida aps a reao uma mistura homognea
e) o xido de clcio atua como um agente redutor

QUESTO 8
A combusto completa do propano ocorre segundo a reao, representada
pela equao no balanceada, abaixo:
C3H8(g)

O2(g)

CO2(g)

H2O(g)

A relao entre os volumes de O2(g) consumido e de CO2(g) produzido de:


a) 1:1

b) 2: 3 c) 3:1

d) 3:2

e) 5:3

Ano Internacional de Qumica



| 55

Programa Nacional
Olimpadas de Qumica

OBQ - 2011 Fase III


QUESTO 9
A quantidade de gs carbnico produzida pela respirao de seres vivos
pode ser acompanhada atravs da reao desse gs com uma soluo saturada de hidrxido de brio, medindo-se o aumento de massa dessa soluo.
Esse aumento de massa se deve :
a) formao de carbonato de brio
b) adsoro do CO2 adsorvido pela gua
c) formao de cido carbnico
d) precipitao do hidrxido de brio
e) formao de xido de brio

QUESTO 10
Uma amostra de um cido diprtico pesando 12,25 g foi dissolvida em gua
e o volume da soluo completado para 500 mL. Se 25,0 mL desta soluo
so neutralizados com 12,5 mL de uma soluo de KOH 1,00 mol.L-1, a massa
molar desse cido, considerando que os dois prtons foram neutralizados,
igual a:
a) 2,25
b) 24,5
c) 49,0
c) 98,0
e) 122,5

O talento no uma propriedade privada, uma propriedade pblica e ningum tem o direito de desperdi-lo.
Wladimir Efroimson, geneticista russo.

56 |

Ano Internacional de Qumica


Olimpada Brasileira
de Qumica 2011

Exames Mod. A

Parte B - QUESTES ANALTICO - EXPOSITIVAS


QUESTO 11

B
C

Considerando os 3 elementos (A, B e C) indicados na tabela peridica acima,


determine:
a) o tomo que apresenta
a.1) o maior raio atmico.
a.2) a maior afinidade eletrnica
a.3) a maior energia de ionizao.
b) a frmula do composto formado entre:
b.1) o tomo A e o tomo B.
b.2) o tomo B e o flor
c.3) o tomo C e o oxignio
QUESTO 12
Para cada uma das espcies qumicas abaixo:
I) NH4+
II) PCl3
III) CO32-
IV) NO3-
V) BF4a) Escreva a estrutura de Lewis
b) Indique:
b.1) a hibridao do tomo central
b.2) o nmero de pares de eltrons no ligantes sobre o tomo central
b.3) o nmero de ligaes sigma e de ligaes pi envolvendo o tomo central
b.4) a geometria em torno do tomo central

Ano Internacional de Qumica



| 57

Programa Nacional
Olimpadas de Qumica

Exames Mod. A
QUESTO 13
Uma macromolcula biolgica foi isolada de uma fonte natural em quantidade muito pequena e sua massa molar foi determinada como sendo 4,0
x 105 g.mol-1. Para uma soluo preparada pela dissoluo de 0,8 mg dessa
macromolcula em 10,0 g de gua.
a) Calcule
a.1) o ponto de congelamento
a.2) a presso osmtica, a 25 C
b) Suponha que a massa molar dessa macromolcula no fosse conhecida
e que se pretendesse calcul-la a partir da determinao da presso osmtica da soluo citada acima e que fosse cometido um erro de 0,1 torr
na medida dessa presso osmtica, qual seria o valor encontrado para a
massa molar da macromolcula?
QUESTO 14
O carbeto de silcio (SiC), tambm conhecido como carborundum, uma substncia dura empregada como abrasivo, pode ser obtido a partir da reao de
SiO2 com carbono, a altas temperaturas, conforme a equao qumica (no
balanceada) abaixo:
SiO2 (s) + C(s)
SiC(s) + CO(g)
a) Reescreva a equao qumica acima, devidamente balanceada
Em um experimento colocou-se para reagir 6,01 g de SiO2 e 7,20 g de carbono.
b) Qual ser o reagente limitante?
c) Que massa de carborundum poder ser obtida, considerando o consumo
completo do reagente (rendimento de 100%)?
d) Que massa restar do reagente em excesso?
e) Se, no experimento acima, obtm-se 2,56 g de SiC, qual o rendimento da reao?
QUESTO 15
Para a reao: 2 NO (g) + Br2 (g) 2 BrNO (g), tem-se um Kc = 0,21 L mol-1
a 350 C.
Sobre esta reao, responda as questes abaixo.
a) Se 2,0 x 10-3 mols de NO, 4,0 x 10-3 mols de Br2 e 4,0 x 10-3 mols de BrNO
so introduzidos em um recipiente de volume igual a 50,0 mL, 350 C, em
que sentido ocorrer a reao? Justifique sua resposta
58 |

Ano Internacional de Qumica


Olimpada Brasileira
de Qumica 2011

Exames Mod. A
b) Qual o valor Kp para essa reao a 350 C?
c) Se o mesmo recipiente contm, no equilbrio, 1,4 x 10-3 mols de NO e 1,4 x
10-4 mols de BrNO a 350 C, que quantidade de Br2 estar presente?
d) Se ao sistema descrito em (c) se adiciona um gs inerte, de modo que a
presso total dentro do recipiente passe a ser de 3 atm, a 350 C:
d.1) Ocorrer mudanas nas concentraes dos componentes da mistura?
d.2) O equilbrio ser deslocado?
d.3) Se ocorre deslocamento, em que sentido ser?
Justifique suas respostas.
QUESTO 16
Um tcnico de laboratrio dispe de uma soluo de NaOH, que no era
utilizada h muito tempo, e em cujo rtulo est escrito: NaOH 0,25 mol.L-1.
Como est soluo instvel, antes de us-la o tcnico decidiu titular 25,0
mL dessa soluo com uma soluo de HCl 0,25 mol.L-1 e gastou 22,5 mL
desta ltima soluo. Responda:
a) Por que a soluo de NaOH instvel?
b) A concentrao indicada no rtulo est correta?
c) Como voc poderia preparar 250 mL de uma soluo exatamente 0,25
mol.L-1 a partir da soluao anterior? (considere que voc dispe de gua
destilada e de NaOH slido e que a adio de NaOH slido no altera o
volume da soluo).
Dados: R= 0,082 dm3.atm.K-1.mol-1
Constante crioscpica da gua (KC) = 1,86 Kkg.mol-1 ou

1,86 Ckg.mol

Massas atomicas aproximadas:

H = 1,0; C = 12,0; O = 16,0; Na = 23,0; Si = 28,1; Ba = 137,3

Nmeros atmicos:

H = 1; B = 5; C= 6; N = 7; O = 8; F = 9; P = 15; Cl = 17
Gabarito:
1

10

Ano Internacional de Qumica



| 59

Programa Nacional
Olimpadas de Qumica

Exames Mod. B

Olimpada Brasileira de Qumica - 2011


MODALIDADE B ( 3 ano )
PARTE B - QUESTES MLTIPLA ESCOLHA

QUESTES 1 a 7
Ver as mesmas questes propostas para a Modalidade A.
QUESTO 8
Os compostos aromticos, por serem bastante estveis do preferencialmente reaes de substituio, e, dentre essas, as reaes de substituio
eletroflicas so as mais favorveis. Considerando essas reaes, indique a
alternativa que apresenta os compostos em ordem crescente de reatividade,
numa reao de nitrao
a) Clorobenzeno, Anilina, Benzeno eTolueno
b) Tolueno, Benzeno, Anilina e Clorobenzeno
c) Benzeno, Anilina,Tolueno e Clorobenzeno
d) Benzeno, Tolueno, Clorobenzeno e Anilina
e) Clorobenzeno, Benzeno, Tolueno e Anilina
QUESTO 9
O nome sistemtico (IUPAC) do paracetamol, um dos analgsicos mais consumidos em todo o mundo, N-(4-hidroxifenil)etanamida. O paracetamol
pode ser obtido a partir da reao de
a) Um cido e um anidrido
b) Uma amina e um anidrido
c) Um cido e um ster
d) Uma amina e um lcool
e) Um cido e um lcool
QUESTO 10
Um composto orgnico foi submetido a anlise por espectrometria de massas (EM) por infravermelho (IV) e por ressonncia magntica de carbono-13
(RMN-13C ) e as seguintes informaes foram obtidas:
I) Por EM, sua massa foi definida como sendo 102 g.mol-1.
II) Por IV constatou-se a presena de uma carbonila
III) Por RMN-13C, deduziu-se que a molcula contm 2 carbonos metlicos
(CH3), 2 carbonos metilnicos (CH2) e um carbono no hidrogenado
60 |

Ano Internacional de Qumica


Olimpada Brasileira
de Qumica 2011

Exames Mod. B
Esse composto pode ser:
a) O butanoato de metila
c) O 2-metilpropanoato de metila
e) O 2-hidroxi-3-metil-butanal

b) A pentanona
d) O cido pentanico

Gabarito:
1

10

PARTE B - QUESTES ANALTICO-EXPOSITIVAS


QUESTES 11 a 14
Ver as mesmas questes propostas para a Modalidade A.
QUESTO 15
O mentol, tambm denominado de cnfora de hortel-pimenta, uma substncia que apresenta um sabor de menta e que encontrada em alguns
leos essenciais, como por exemplo: o leo de hortel-pimenta. O nome
sistemtico (IUPAC) do mentol : 2-isopropil-5-metilciclohexanol.
a) Desenhe a estrutura do mentol sem levar em conta a estereoqumica
b) Escreva a frmula molecular do mentol
c) Indique quais so os carbonos assimtricos presentes na molcula
d) Indique o nmero possvel de estereoismeros
e) Sabendo que o carbono ligado hidroxila tem configurao R e que os
grupos metila e isopropila esto, respectivamente, em posio CIS e posio
TRANS em relao hidroxila, faa um novo desenho da estrutura do mentol,
agora com a estereoqumica corrreta.
QUESTO 16
Um hidrocarboneto insaturado (composto A) produz, por ozonlise, propanona e 2-metilpropanal.
a) Escreva a estrutura e o nome do composto A
b) Escreva as estruturas e os nomes dos compostos orgnicos produzidos nas
reaes do composto A com:
b.1) B2H6, seguido de Zn, H+.
b.2) H2 / Pd
b.3) KMnO4, OH-, a frio
b.4) H2O, H+
b.5) Br2, H2O

Ano Internacional de Qumica



| 61

Programa Nacional
Olimpadas de Qumica

Questes resolvidas I

Questo 11 Modalidade A - Resoluo de Daniel Arjona


de Andrade Hara, Objetivo - Cotia.

Temos A = Na, B = S e C = Ca
A1) O maior raio atmico do Ca (elemento C) j que est localizado no quarto perodo, enquanto os outros esto localizados no terceiro perodo.
A2) Maior carga nuclear efetiva do S (elemento B), pois est mais a direita
na tabela peridica, os eltrons mais internos tem maior atrao com o
ncleo, logo ter maior capacidade de acomodar cargas negativas.
A3) Maior carga nuclear efetiva do S faz com que seus eltrons estejam
mais ligados com o ncleo logo necessrio mais energia para arrancar
eltrons do S, se compararmos com Na e Ca.
B1) O sdio forma ctions Na+ e o enxfre forma nions S2- logo a frmula
do composto Na2S.
B2) O enxfre forma diversos compostos covalentes com o flor como SF2,
SF4 e SF6. Segundo a regra do octeto o tomo de S faria duas ligaes
covalentes com o flor, resultando na seguinte estrutura, F-S-F.
12 modalidade
B3)Questo
O clcio forma
Ca2+ o oxignioB
forma nions O2-, logo o composto CaO.

Cdigo 133

Questo 12

Modalidade B - Resoluo de Bruno Limaverde Vilar Lbo, Farias Brito - Fortaleza.

A)A)

I - NH4+
+

H
H

H
62 B.1
|

- sp3

Ano Internacional de Qumica


H
B.3 - 4 ligaes sigma ( ) e nenhuma pi ( ).

- sp3
B.4B.1
- Tetradrica

Olimpada Brasileira
de Qumica 2011

B.2 - nenhum par de eltrons no ligantes.


Questes resolvidas I
II -B.3
PCl-334 ligaes sigma ( ) e nenhuma pi ( ).
B.1 - sp
B.4
- Tetradrica
B.2
- nenhum
par de eltrons no ligantes.
Cl - 4Pligaes
Cl sigma ( ) e nenhuma pi ( ).
B.3
B.4 - Tetradrica

Cl

- PCl
3
II -IIPCl
3
B.1 Cl
- sp3P Cl
B.2 - 1 par de eltrons no ligantes.

Cl

B.3 - 3 ligaes sigma ( ) e nenhuma pi ( ).


3

B.1
- sp- sp3
B.1
B.4
Piramidal
B.2 - 1 par de eltrons no ligantes.

B.3
- 3-ligaes
sigma
( ) e nenhuma
pi ( ).
B.2
1 par de
eltrons
no ligantes.
B.4 - Piramidal

23 ligaes sigma ( ) e nenhuma pi ( ).


III B.3
-CO-32-

III -CO3

B.4 - Piramidal2O
C
IIIO
-CO32-O

2
2B.1--3spligaes
B.3
sigma ( ) e 1 pi ( ).
B.1
- sp2 O

B.2 - nenhum par de eltrons no ligantes.

B.3---Trigonal
3
ligaes
sigma
) e 1 pi ( no
). ligantes.
C Planar
B.4
B.2
nenhum
par
de( eltrons
B.4 - Trigonal Planar
O
O
V - BF4V - BF4-

B.1 - spF2

B.2 - nenhum par de eltrons no ligantes.


F B F

F
B.1- sp3

B.1- sp3

Ano Internacional de Qumica



| 63

B.2 - nenhum par de eltrons no ligantes.

Programa Nacional
Olimpadas de Qumica

Questes resolvidas I
B.2 - nenhum par de eltrons no ligantes.
B.3 - 4 ligaes sigma ( ) e nenhuma pi ( ).
B.4 - Tetradrica.
Questo 13 Modalidade B

Questo 13

Modalidade B - Resoluo de Lara Mulato

Inscrio 505

Lima, Ari de S - Fortaleza

a)
a.1) tc = Kc W

Kc = 1,86 Kgmol

X = macromolcula

mH2 O = 0,01 Kg

nx = 2 107 molal

mx = 8 107 Kg

Mx = 4 10 gmol

I)

W=

nx

mH2 O

2109 mol
0,01 Kg

= 2 107 molal

tc = 1,86 2 107 = 3,72 107

II)

tc = 273,15 3,72 107 = 273,1499996 K

III)

a.2) = M. R. T
=

T = 298 K dH2 O = 1 gmL vH2 O

2 109
0,082 298
0,01

= 4,887 106 atm

b) Nova presso osmtica = 1,364 104 atm


= M. R. T

M=
= 5,5846 106 molL
RT

nx = 5,5846 106 106 0,01 = 5,5846 108 mol

Mx =

64 |

mx
8 104 g
Mx =
= 1,4325 104 gmol
nx
5,5846 108 mol

Ano Internacional de Qumica


Olimpada Brasileira
de Qumica 2011

Questes resolvidas I

Questo 14 Modalidade B - Resoluo de Carlos Eduardo Grivol Jnior, Anchieta - Salvador

a) 1 SiO2(s) + 3C(s) 1SiC(s) + 2C(g)


b) M1 = 28,1 + 2 16 = 60,1 gmol de SiO2
M2 = 12 gmol de C(s)
m1 = 6,01 g

m2 = 7,2 g

60,1 =

12 =

6,01
n1 = 0,1 mol de SiO2
n1

7,2
n2 = 0,6 mol de C(s)
n2

Como a proporo do reagente SiO2(s) para o reagente C(S) de 1:3,


obeserva-se que 0,1 mol de SiO2(s)

ir reagir com 0,3 mol de C(s)

apenas. Desse modo, C(S) est em excesso e o reagente limitante


SiO2(s).

c) De acordo com a equao balanceada, se houver consumo completo


de 0,1 mol de SiO2(s) , ser produzido 0,1 mol de C(S)
MX = 28,1 + 12 = 40,1 gmol de carborudum
mx
mx = 4,01 de carborudum
40,1 =
0,1

d) Como o reagente limitante determinou que apenas 0,3 mols de C(s)


reagiu, podemos calcular seu excesso:

Ano Internacional de Qumica



| 65

n = 0,6 0,3 n = 0,3 mol de C(s)

de 0,1 mol de SiO2(s) , ser produzido 0,1 mol de C(S)


MX = 28,1 + 12 = 40,1 gmol de carborudum

Programa Nacional
mx
Olimpadas de Qumica

40,1 =

0,1

mx = 4,01 de carborudum

Questes resolvidas I

d) Como o reagente limitante determinou que apenas 0,3 mols de C(s)


reagiu, podemos calcular seu excesso:
n = 0,6 0,3 n = 0,3 mol de C(s)
12 =

mr
mr = 3,6 g de C(s)
0,3

Restar uma massa de 3,6 g de C(S) em excesso.

e) J foi calculado acima que com um rendimento de 100% ser


produzida uma massa de 4,01 g de SiO2(s), portanto:
100
p
=
p = 63,84%
4,01 2,56

O rendimento da reao ser de 63,84 %.

66 |

Ano Internacional de Qumica


Olimpada Brasileira
de Qumica 2011

Questes resolvidas I

Questo 15 Mod. B - Resoluo de Bruno Limaverde Vilar


Lbo, Farias Brito - Fortaleza

a)

Clculo das concentraes dos gases:

[NO] = 2,0 x 10-3 mol/ 0,05 L = 0,04 mol L-1


[Br2] = 4,0 x 10-3 mol/ 0,05 L = 0,08 mol L-1
[BrNO] = 4,0 x 10-3 mol/0,05 L = 0,08 mol L-1

Clculo do Qc, quociente de reao:

Qc = [BrNO]2/[NO]2 . [Br2]
Qc = (0,08 mol L-1)2/(0,04 mol L-1)2 . (0,08 mol L-1)
Qc = 50 mol L-1

(Como Qc > Kc, a reao ocorrer no sentido dos

reagentes, de modo que Qc = Kc)

b) Kp = Kc (RT) n
Kp = 0,21 x (0,082 x 623)-1
Kp = 4,11 x 10-3

c)

Clculo das concentraes de NO e de BrNO:

[NO] = 1,4 x 10-3 mol/ 0,05 L = 0,028 mol L-1


[BrNO] = 1,4 x 10-4/ 0,05 L = 0,0028 mol L-1

Clculo da concentrao de Br2:


Ano Internacional de Qumica

| 67
Kc = [BrNO]2/ [NO]2 . [Br2]

c)
Programa
Clculo
das concentraes de NO e de BrNO:
Nacional
Olimpadas de Qumica

[NO] = 1,4 x 10-3 mol/ 0,05 L = 0,028 mol L-1

Questes resolvidas I

[BrNO] = 1,4 x 10-4/ 0,05 L = 0,0028 mol L-1

Clculo da concentrao de Br2:

Kc = [BrNO]2/ [NO]2 . [Br2]


[Br2] = [BrNO]2 / Kc . [NO]2
[Br2] = (0,0028 mol L-1)2 / 0,21 mol L-1 . (0,028 mol L-1)2
[Br2] = 0,0476 mol L-1

d)
d.1) Ao adicionarmos o gs inerte, no ocorrer mudanas nas
concentraes dos componentes da mistura, pois nem as quantidades dos
gases e os volumes dos recipientes so alterados.
d.2) O equilbrio no ser deslocado, pois como as concentraes dos
gases participantes do equilbrio permanecem constantes, Qc permanece
igual a Kc.

68 |

Ano Internacional de Qumica


Olimpada Brasileira
de Qumica 2011

Questes resolvidas I

Questo 15

Mod.B - Resoluo de Francisco Rodrigues da


Cruz Jnior, Dom Barreto - Teresina

OH

a)

b) Mentol C10H22O

OH
c)

O 2 carbono da cadeia principal


d) 2n = 21 = 2 estereoismeros

Ano Internacional de Qumica



| 69

Programa Nacional
Olimpadas de Qumica

Questes resolvidas I

Questo 16 Modalidade A -

+ e OH-, o Na
+ ser
+ e
orque a)
quando
uma
soluouma
aquosa
dissocia-se
Na
que
Porque
quando
soluo
aquosa em
dissocia-se
em de
NaNathrcia
OH-, hidratado,
oCasNa+ serjhidrat
Resoluo
-, o Na+
tro Mota,
Master - em
Fortaleza
+ ser
a) vrias
Porque(devido
quandoo uma
soluo
aquosa
Na+redor
e OHdo
rias hidroxilas
se
meio
agruparam
ao
hidratado
hidroxilas
(devido
oaquoso)
se
meiosedissocia-se
aquoso)
se agruparam
ao redor
dohidrat
Na+

+
vrias
hidroxilas
(devido aquosa
o se meio
aquoso)
ao+ ser
redor do Na+
ificultando
suaquando
locomoo.
Porque
uma
soluo
dissocia-se
emse
Naagruparam
e OH-, o Na
dificultando
sua locomoo.
hidratado, j que vrias hidroxilas (devido o se meio aquoso) se agruparam
dificultando sua locomoo.

NaOH +b)HClNaOH
NaCl
+ H
2ONaCl + H2O
+ HCl

NaOH
+ HCl
NaCl + H2O
5 mL de
b)NaOH(aq)
25
mL de
NaOH(aq)

25
mL
HCl 0,25
mol/L
NaOH(aq)
22,5
mL 22,5
0,25
mL 1000 mL
HClde
0,25
mol/L
mLmol
1000
0,25 mol
HCl 0,25 mol/L 22,5 mL

0,25
mL
1000
x mol
22,5
x mL
22,5 mL

-3 mol
x
22,5
mL
-3 mol
x = 5,625 x 10
x = 5,625
x 10

x = 5,625 x 10-3 mol

1 mol de NaOH
1 mol
de
HCl
1 molde
NaOH
1 mol de HCl

-3 mol
1 mol
de NaOH
de de
HCl
-3 mol de HC
y
5,625
HCl
y x110mol
5,625
x 10

M=

MM

y mol
de
5,625
x 10-3 mol de HC
y = 0,225 mol
NaOH
y =de
0,225
NaOH

0,225
0,225 3 y = 0,225
3 mol de NaOH
= M = =m5,625
mol de
NaOH
= 10
= 5,625
10
mol de NaOH
40
MM
40
m
0,225
= n3 =5,62510
5,6253
103 mol de
5,62510
MM
M = 0,225 M
M =340== 0,225 3
V
2510
V
2510
5,625103
dificultandonsua locomoo.

M
n =
M= =

ao redor do Na+ hidratado


= =
= 0,225 M
3 mol/L.
sta: No,
j que No,
a concentrao
correta M0,225
V mol/L.
Resposta:
j que a concentrao
correta
2510
0,225

NaOH


Resposta:
No,a j
que a concentrao
0,225 mol/L.
Resposta:
No, j que
concentrao
correta correta
0,225 mol/L.

]=

n
v

n
= molL =n =n 0,0625
] = molL 0,25
mol
n = 0,0625 mol
c) [ 0,25
0,250
v
n

0,250
n

m c) [ ] =m m
0,25 molL m
=
n = 0,0625 mol
v
0,250
=
n =0,0625
=
= 2,5 g=de NaOH

0,0625
=
2,5
g
de NaOH
MM
40
MM
40
m
m
n = 2,5 g de NaOH
0,0625em
= um=balo
2,5 g de NaOH 250 mL de gua destilada.
, acrescentando
MM
Ento, acrescentando
2,5 g de40
NaOH emcontendo
um balo contendo 250 mL de gua destilada

Ento, acrescentando 2,5 g de NaOH em um balo contendo 250 mL de gua destilada


Ento, acrescentando 2,5 g de NaOH em um balo contendo 250 mL de gua
destilada.
70 |

Ano Internacional de Qumica


Olimpada Brasileira
de Qumica 2011

Questes resolvidas I

Questo 16

Modalidade A - Resoluo de Guilherme


Costa G. Fernandes, Colgio Militar - Braslia

a) Sabendo
a) Sabendo
que aque
ozonlise
a ozonlise
de alcenos
de alcenos
quebra
quebra
a dupla
a dupla
ligao,
ligao,
formando
formando
dois compostos
dois compostos
carbonilados,
carbonilados,
o composto
o composto
A temA atem
seguinte
a seguinte
frmula
frmula
estrutural:
estrutural:

Nomenclatura:
Nomenclatura:
2,4-dimetilpent-2-eno
2,4-dimetilpent-2-eno

b)

b)

b.2) Reao
b.2) Reao
de hidrogenao
de hidrogenao
cataltica:
cataltica:
H2
Pd

H2
Pd

2,4-dimetilpentano
2,4-dimetilpentano

b.3) Oxidao
b.3) Oxidao
branda
branda

KMnOKMnO
4
4

OH

OH
2,4-dimetilpentano-2,3-diol
2,4-dimetilpentano-2,3-diol

OH, frio
OH, frio
OH

OH

b.4) Adio
b.4) Adio
de gua
de gua
em meio
em meio
cidocido
A ligao
A ligao
se desloca
se desloca
preferencialmente
preferencialmente
para para
o carbono
o carbono
3, pois
3, pois

Ano Internacional de Qumica



| 71

assimassim
formado
formado
o carboction,
o carboction,
tercirio
tercirio
que que
relativamente
relativamente
estvel
estvel

OH

KMnO4
Programa Nacional
Olimpadas de Qumica

2,4-dimetilpentano-2,3-diol

OH, frio

Questes resolvidas I
OH

b.4) Adio de gua em meio cido


A ligao se desloca preferencialmente para o carbono 3, pois
assim formado o carboction, tercirio que relativamente estvel

2,4-dimetil-pen
OH

b.5) Adio de bromo aquoso


Em gua o bromo sofre heterlise:

Br2 (aq)
A ligao do alceno

Br

Br

se desloca analogamente ao exemplo

anterior, para que a carga positiva fique mais aliviada

72 |

Ano Internacional de Qumica


Olimpada Brasileira
de Qumica 2011

Resultado - Fase III

Olimpada Brasileira de Qumica - 2011

OURO
Bruno Matissek Worm

Modalidade A
Tiradentes

Porto Alegre

RS

Vitria Nunes Medeiros


Leonardo de Andrade Mesquita
Vinicius Lopes Braga
Emerson Holanda Marinho
Jlio Csar Prado Soares

Farias Brito
Militar
Dom Barreto
Farias Brito
Militar

Fortaleza
Porto Alegre
Teresina
Fortaleza
Braslia

CE
RS
PI
CE
DF

PRATA
Victor Santos de Andrade
Daniel Arjona de Andrade Hara
Francisco Davi Barbosa dos Santos
Joaquim Ivo Vasques D. Landim
Matheus Salmito Rodrigues Ponte
Gabriel Matheus Viana Pinheiro
Lvia Rodrigues de Arajo
Andr Santos Fernandes
Mariana Camyla Duarte Pontes
Sergio Pereira de Oliveira Jnior
Rogrio Yuuki Motisuki
Brandon Wahib Bogossian Khalil
Joo Pedro Alexandre Silva Mota
Raissa Niuta Freitas de Oliveira

Dom Barreto
Objetivo
Farias Brito
Paraiso
Farias Brito
Mster
Farias Brito
Albert Sabin
Farias Brito
Farias Brito
Etapa
IF-RJ
Farias Brito
7 de setembro

Teresina
Cotia
Fortaleza
J. do Norte
Fortaleza
Fortaleza
Fortaleza
Osasco
Fortaleza
Fortaleza
So Paulo
Rio de Janeiro
Fortaleza
Fortaleza

PI
SP
CE
CE
CE
CE
CE
SP
CE
CE
SP
RJ
CE
CE

BRONZE
Gabriel Berlingieri Polho
Laio Ladislau Lopes Lima
Filipe Herson Carneiro Rios
Renato Frankiley da Silva Lima
Nicholas de Souza Costa Lima
Ramon Santos Gonalves da Silva

Renovao
Farias Brito
Farias Brito
EREM Prof. Adauto Carvalho
Ari de S
Ideal Militar

So Paulo
Fortaleza
Fortaleza
Recife
Fortaleza
Belm

SP
CE
CE
PE
CE
PA

Ano Internacional de Qumica



| 73

Programa Nacional
Olimpadas de Qumica

Resultado - Fase III


Kelvin Azevedo dos Santos
Dbora Barreto Ornellas
Iohanna Moreira Nunes Ribeiro
Vinicius Lima Ferreira
Nathrcia Castro Mota
Giuvanni Mutton
Isabel Theresa de Holanda Freitas
Henrique Magri Maron
Renata Santos Martins
Leonardo Kazunori Tsuji
Pedro Henrique Almeida Fraiman
MENO HONROSA
Breno Cassaro Mouro
Vinicius Alves Duarte da Silva
Pedro Silveira Quintana
Romullo Randell Macedo Carvalho
Drvylla de Sousa Lima
Enoch Yang
Nilton Leal Arajo
Jos Ivan F. de O. Neto
Marcus Vincius Martins de Almeida
Alexandre Andrade C. de Almeida
caro de Almeida Varo
Gabriel Augusto Ginja
Henrique Gasparini F. Nascimento
Driele Neves Ribeiro
caro Sampaio Viana
Dimitri Reis de Matos
Renan Ferreirinha Carneiro
Rafael Ribeiro Alves
Ana Raquel Ferreira de Azevedo
Jssica Silva Lopes
Bernardo Guimares L. de Carvalho
Rayssa Oliveira do Nascimento
Eduardo Poleze
74 |

Uniclass Objetivo
Anchieta
IF-RJ
IF-RJ
Master
Etapa
COLUNI
Etapa
Dom Barreto
Etapa
Cincias Aplicadas

Goinia
Salvador
Paracambi
Rio de Janeiro
Fortaleza
So Paulo
Viosa
Valinhos
Teresina
So Paulo
Natal

GO
BA
RJ
RJ
CE
SP
MG
SP
PI
SP
RN

IF-RJ
IF-RJ
Politcnico da UFSM
Lavoisier
Teleyos
Presbiteriano M. Tambor
Dom Barreto
Ideal
Etapa
Col. de Aplicao-UFPE
Dom Barreto
Etapa
Militar
Col. de Aplicao-UFPE
7 de Setembro
Anchieta
Militar
Motiva
Farias Brito
Farias Brito
Santo Antnio
IF-RJ
Bertoni

Nilpolis
Maracan
Santa Maria
Teresina
Fortaleza
Sant. de Paraiba
Teresina
Belm
So Paulo
Recife
Teresina
So Paulo
Braslia
Recife
Fortaleza
Salvador
Rio de Janeiro
Campina Grande
Fortaleza
Fortaleza
Belo Horizonte
Rio de Janeiro
Medianeira

RJ
RJ
RS
PI
CE
SP
PI
PA
SP
PE
PI
SP
DF
PE
CE
BA
RJ
PB
CE
CE
MG
RJ
PR

Ano Internacional de Qumica


Olimpada Brasileira
de Qumica 2011

Resultado - Fase III


Flvio Luis Schneider Junior
Militar
Manaus
Gustavo Rosa Gameiro
Universitario
Londrina
Leonardo Hautrive Medeiros
Politcnico da UFSM
Santa Maria
Laura Martins Pina de Almeida
IF-RJ
Rio de Janeiro
Vitor Juc Policarpo
7 de setembro
Fortaleza
Daniel de Oliveira S. Vasconcelos e S
GGE
Recife
Lucas Henrique Carvalho Furquim
Dom Barreto
Teresina
Matheus Brum Moraes
Ateneu
Londrina
Victor Sousa Silva
Ari de S
Fortaleza
Renata Segatto Pigatto
Politcnico da UFSM
Santa Maria
Bruno Alves Caremiro
CEFET
Rio de Janeiro
Luiz Felipeh Aguiar de Lima Alves
Militar
Rio de Janeiro
Lucas Gabriel de Barros Silva
Militar
Porto Alegre
Gustavo Xavier de Jesus
Militar
Rio de Janeiro
Ramon Silva de Lima
Objetivo
So Paulo
Andreza Saboia Dantas
Cincias Aplicadas
Natal
Gean Almeida da Silva
IF-RJ
Rio de Janeiro
Israel Wilson Nobre Barreto
Master
Fortaleza
Rogrio Jos Menezes Alves
Charles Darwin
Vila Velha
Raphael Souza de Almeida
Alternativo
Aracaju
Emanoela Bitencourt Varjo
Amadeus
Aracaju
Gabriel Antnio Flores Chies
Liberato Salzano
Novo Hamburgo
Felipe Brando Forte
Farias Brito
Fortaleza
Rodrigo Castiel
Motivo
Recife
Diego Tartglia L. de Andrade
Pedro II
Rio de Janeiro
Mateus Juc Pinheiro
7 de setembro
Fortaleza
Joo Victor Burlamaqui Coelho
Crescimento
So Lus
Haroldo Nogueira Victoriano Neto
Dulia Bringel
Fortaleza
Patricky Albani de Souza
C E E Profissional de Curitiba
Curitiba
Maria Eliza Machado Romeros
CEFET
Belo Horizonte
Marina Melo Sousa Mendes Leal
Sag. Corao de Jesus
Teresina
Vitria Espindola Leite Borges
Santo Agostinho
Goinia
Rafael Reinaldo Gracez Nedel
Maria Auxiliadora
Canoas
Bruno de Lima Santos
IFES
C. de Itapemirim
Aurimar Bezerra M. de Sousa Filho
Dom Barreto
Teresina
Ana Bearice Bonganha Zanon
Unidade Jardim
Santo Andr

AM
PR
RS
RJ
CE
PE
PI
PR
CE
RS
RJ
RJ
RS
RJ
SP
RN
RJ
CE
ES
SE
SE
RS
CE
PE
RJ
CE
MA
CE
PR
MG
PI
GO
RS
ES
PI
SP

Ano Internacional de Qumica



| 75

Programa Nacional
Olimpadas de Qumica

Resultado - Fase III


Jos Valnir Teixeira Cruz
Paulo Augusto de Paiva Silva
Victor Venturi
Matheu Coelho Ferraz
Erick Leonardo de Souza Monteiro
Diego Rodrigues Carvalho
Emille Renalle Freitas de Melo
Pedro Guimares Martins
Matheus Compart Hemerly
Carolina Konrdorfer Rangel
Matheus de Moura Amaral
Henrique Martins Tavares
Nicolas Eduardo da Fonseca Farias
Matheus Rocha
Fagner Lucio de Toledo
Abdon Moutinho
Renato Xavier de Jesus
Fernando Antnio Saraiva Maia
Filipe Ferracioli
Lucas Stefan Minuzzi Neumann
Filipe Mouro Leite
Gabriel Henrique Kuupfer
Mariana Lcia Branco Zeitune
Francisco Markan Nobre de Souza
Vitor Souza Guimares
Ana Beatriz Marques
Helosa Maria Machado Massaro
Camila de Lima Magalhes
Thiago Pereira Canoco
Guilherme da Silva Arajo
Bartira Magalhes Rodrigues
Ivan Tadeu F. Antunes Filho
Leonardo Yuji Miakawa
Valria Karine de Azevedo Ferreira
Elaine Costa
Daniel Costa Xavier de Oliveira
76 |

Sag. Corao de Jesus


IFET
Etapa
Etapa
Fundao Nokia
Antonio Vieira
Motiva
Bernoulli
Charles Darwin
Liberato Salzano
Charles Darwin
Marista Rosrio
Motiva
Charles Darwin
COLUNI
Exponente
Militar
Dulia Bringel
Etapa
Politcnico da UFSM
Dom Barreto
SENAI Norte II
IF-RJ
Farias Brito
So Francisco Xavier
IFET
Bruno Giorgi
Olimpo
7 de Setembro
CEFET
Modulo
Objetivo Paulista
Etapa
Lato Sensu
Militar
Olimpo

Teresina
Niteri
Campinas
So Paulo
Manaus
Salvador
Campina Grande
Belo Horizonte
Vitria
Novo Hamburgo
Anchieta
Porto Alegre
Joo Pessoa
Vitria
Viosa
Recife
Rio de Janeiro
Fortaleza
Vinheda
Santa Maria
Teresina
Joinville
Rio de Janeiro
Fortaleza
Belo Horizonte
Barbacena
Mococa
Goinia
Fortaleza
Belo Horizonte
Salvador
Lins
So Paulo
Manaus
Recife
Goinia

PI
MG
SP
SP
AM
BA
PB
MG
ES
RS
ES
RS
PB
ES
MG
PE
RJ
CE
SP
RS
PI
SC
RJ
CE
MG
MG
SP
GO
CE
MG
BA
SP
SP
AM
PE
GO

Ano Internacional de Qumica


Olimpada Brasileira
de Qumica 2011

Resultado - Fase III


Alex Mendes Leonel Freire
Samuel Incio Oliveira Arajo
Camila Piovesan Wiethan
Wederson Santos Silva
Raphael Lima Santos
Victor de Mello Alves Nunes
Ana Carolina F. Gondin Guimares
Arthur Shiniti Cato
Rodrigo Lins SantAna de Lima
Marcio Santos Souza Caroso
Naiara de Oliveira Baptista
Gabriel Jos Moreira da Costa Silva
Juliany Pires Figueiredo
Felipe Henrique Moura Chupel
Tales Mesquita Muniz
Larissa Farnetti Pinto
Diego Rodrigues da Ponte
Matheus Wenzel S Gonalves
Igor Barboza Cunha
Matheus Freitas Rocha Bastos
Koody Andr Hassemi Kitawara
Joo Marcos C Namos da Slva
Fellipe Francisco da Silva
Joana Loureno de Carvalho
Pedro Igncio Nogueira Ges
Alan Barbosa Costa
Pedro Henrique G. Carvalho
Maria Emanuella Moura Silva
Gabriel Rezende Nahas
Wagner Thiele Fracassi
Marina Albuquerque B. de Almeida
Joo Pedro O.Freitas
Joo Pedro Silva Kirmse
Morgana Garibaldi Diefenthaeler
Murilo Freitas Yonashiro Coelho
Rafael Eller Cruz

Contato
Colgio Arnaldo
Politcnico da UFSM
Motiva
Alternativo
IF-RJ
COLUNI
Etapa
Anchieta
Sartre Coc
Mster
Contato
Lato Sensu
Militar
Farias Brito
Coleguim
Dulia Bringel
Do Salvador
Salesiano
Militar Dom Pedro II
Universitario
Prevest
IF-RJ
Motiva
Ari de S
IF-AL
Santa Ceclia
GGE
Bionatus
Liberato Salzano
Mater Christi
Salesiano
Leonardo da Vinci
Marista Rosrio
Etapa
Sag. Corao de Maria

Macei
Belo Horizonte
Santa Maria
Campina Grande
Aracaju
Rio de Janeiro
Viosa
So Paulo
Salvador
Salvador
Fortaleza
Macei
Manaus
Braslia
Fortaleza
Belo Horizonte
Fortaleza
Aracaju
Aracaju
Braslia
Londrina
Goinia
Nilpolis
Joo Pessoa
Fortaleza
Macei
Fortaleza
Recife
Campo Grade
Novo Hamburgo
Mossor
Aracaju
Braslia
Porto Alegre
So Paulo
Braslia

AL
MG
RS
PB
SE
RJ
MG
SP
BA
BA
CE
AL
AM
DF
CE
MG
CE
SE
SE
DF
PR
GO
RJ
PB
CE
AL
CE
PE
MS
RS
RN
SE
DF
RS
SP
DF

Ano Internacional de Qumica



| 77

Programa Nacional
Olimpadas de Qumica

Resultado - Fase III


Lucas Rafael Mendes Almeida
Livio Moreira Rios
Diego Guedes Chaves
Nilo Parente Pessoa Dias
Heitor Augusto C. de Almeida
Meline Gabriele de Paula Santos
Leonardo Enrico M. Mendes
Mara de Paula Nunes
Lucas Fernando Silva Hess
Armando M. Nader
Matheus Ravel Timo Barbosa
Lusa Naomi Castro Suda
Amanda Duarte de Souza
Juliana Schenhel Zotti
Ezau Silva Ribeiro
Pedro Otvio de Carvalho Ramos
Douglas Martins Carneiro
Joo Pedro Maia Sadi
Cassiano Sampaio Campana
Guilherme Henrique M. de Araujo
Sarah Segantini Campos de Souza
Rodrigo Dias Garcia
Victor Hugo Queiroz Rebello
Uriah Martelli Glaza
Ricardo Dognoni Hudsmann
Ruan Lucas Juc Azevedo
Andr Tayt Sohn Ferioli
Lara Ramos Monteiro Silva
Tiago da Silva Nunes
Lucas Gondim Briand Vieira
Eleodorio Sales Bomfim Neto
Victor de Arajo Rocha
Marcus Vinicius Pessoa de Almeida
Eros Gimenes de Queiroz
Lucas Hage
Julio Cesar Oliveira L. Filho
78 |

Sag. Corao de Jesus


Crescimento
Motiva
Farias Brito
Motiva
Pedro II
Olimpo
Marista Dom Silvrio
CEFET
Olimpo
Militar
So Francisco Xavier
Senai
E E E M Gustavo Biazus
Elite Vila dos Cabanos
Dom Bosco
Cepron
Loyola
Maxi
Crescimento
Santa Maria Pampulha
Sartre Coc
Nacional
Acesso
SENAI Norte II
Elite
Pedro II
Crescimento
Arquidiocesano
Farias Brito
Motiva
Anchieta
Ari de S
Leonardo da Vinci
Santa Bartolomea
Batista

Teresina
So Lus
Joo Pessoa
Fortaleza
Joo Pessoa
Niteri
Braslia
Belo Horizonte
Belo Horizonte
Goinia
Braslia
Belo Horizonte
Catalo
Tupanci do Sul
Belm
So Lus
Dom Pedro
Belo Horizonte
Londrina
So Lus
Belo Horizonte
Salvador
Vitria
Curitiba
Joinville
Belm
Niteri
So Lus
Aracaju
Fortaleza
Campina Grande
Salvador
Fortaleza
Vitria
Macap
So Lus

PI
MA
PB
CE
PB
RJ
DF
MG
MG
GO
DF
MG
GO
RS
PA
MA
MA
MG
PR
MA
MG
BA
ES
PR
SC
PA
RJ
MA
SE
CE
PB
BA
CE
ES
AP
MA

Ano Internacional de Qumica


Olimpada Brasileira
de Qumica 2011

Resultado - Fase III


Marcus Di Fabianni F. Lopes Filho
Wilson Santos Craveiro Rosa
Oslio Cndido de Arajo L. Lima
Mariana Braga Lacerda
Vitor Hugo Fernandes Breder
Daniel Ferreira Matos
Ana Beatriz Costa Cavalcanti
Murillo Henrique de M. Rodrigues
Mateus Henrique Schneider
Thalles Emannuel Batista Pinheiro
Jos Renato Nascimento Gomes
Matheus Sonego Temp
Leonardo Jorge Bessa Tajra Filho
Nicolas Chiu Ogassavara
Luana dos Anjos Assis
Hrica Caroline Mathiel
Pedro Pereira de Almeida
Wendrey Lustosa Cardoso
Gustavo Henrique de M. F. Filho
Ksia Priscila Omena Cardoso
Henrique Trivisan Rolo
Luana Pompeu dos Santos Rocha
Marielly Rezende Santos
Rafael Campos do Nascimento
Arthur Lira Araujo

Militar
Lettera
Farias Brito
Motiva
Dom Barreto
Sartre Coc
GGE
Senai
Marista Rosrio
Motiva
Unifev
Politcnico da UFSM
Juvenal de Carvalho
Etapa
Arquidiocesano
Profa. Julieta Mota Santos
Anchieta
Militar
Contato
IF-AL
Positivo
Dom Barreto
IF-MT
So Francisco de Assis
Motiva

Manaus
Teresina
Fortaleza
Joo Pessoa
Teresina
Salvador
Recife
Catalo
Porto Alegre
Joo Pessoa
Votuparanga
Santa Maria
Fortaleza
So Paulo
Aracaju
Coscim
Salvador
Manaus
Macei
Macei
Curitiba
Teresina
Cuiab
Apucarana
Campina Grande

AM
PI
CE
PB
PI
BA
PE
GO
RS
PB
SP
RS
CE
SP
SE
MS
BA
AM
AL
AL
PR
PI
MT
PR
PB

Ano Internacional de Qumica



| 79

Programa Nacional
Olimpadas de Qumica

Resultado - Fase III

Olimpada Brasileira de Qumica - 2011

Nome
OURO
Taynara Carvalho Silva

Modalidade B
Escola

Cidade

UF

Mster

Fortaleza

CE

Guilherme Costa G. Fernandes

Militar

Braslia

DF

Carolina de S Mendona

Militar

Rio de Janeiro

RJ

Dom Barreto

Teresina

PI

Alex Silva de Cerqueira

CEFET

Salvador

BA

Igor Franzoni Okuyama

Olimpo

Goinia

GO

Joo Pedro Cavalcanti Pereira

Santa Maria

Recife

PE

PRATA
Arthur Eduardo Pastore de Lima

Medianeira

Medianeira

PR

Mateus Braga de Carvalho

Dom Barreto

Teresina

PI

Raul Bruno Machado da Silva

Farias Brito

Fortaleza

CE

Vitor Lucena Carneiro

Olimpo

Goinia

GO

Davi Rodrigues Chaves

Ari de S

Fortaleza

CE

Renan Lucas da Silva Custdio

Ari de S

Fortaleza

CE

Lara Mulato Lima

Ari de S

Fortaleza

CE

Militar

Rio de Janeiro

RJ

Objetivo

Mogi das Cruzes

SP

Charles Darwin

Vitria

ES

Danilo Moreira Simes

Objetivo Integrado

So Bernardo

SP

Jayane Carvalho Borges

Lettera

Teresina

PI

Ryan Gimenes de Souza

Militar

Rio de Janeiro

RJ

Jos Matheus G. de A. Bastos

Fabricio da Silva Costa


William Tutihashi
Vitor Marques Assad

80 |

Ano Internacional de Qumica


Olimpada Brasileira
de Qumica 2011

Resultado - Fase III


BRONZE
Marco Antonio da C. Nascimento
William Teixeira Miranda

Lato Sensu

Manaus

AM

CEFET

Belo Horizonte

MG

Anchieta

Salvador

BA

GGE

Recife

PE

Farias Brito

Fortaleza

CE

Davidson Anthony Arago Freire

Ari de S

Fortaleza

CE

Rafaela Ges Machado

Anchieta

Salvador

BA

Liberato Salzano

N. Hamburgo

RS

Letcia Nunes de Oliveira

Dom Barreto

Teresina

PI

Slvio Furtado Ximenes

Dulia Bringel

Fortaleza

CE

Dom Bosco

Curitiba

PR

Fernando Barbosa Neto

Nacional

Vila Velha

ES

Alisson de Sousa Barreto

Ari de S

Fortaleza

CE

Rodrigo Medeiros Guercio

Motiva

Joo Pessoa

PB

Leonardo dos Anjos Cunha

Mater Amabilis

Guarulhos

SP

Daniel Graciano Costa

Expresso

Goinia

GO

Igor Santos Ghignotti

Charles Darwin

Vila Velha

ES

Dom Barreto

Teresina

PI

Bernoulli

Belo Horizonte

MG

Santa Maria

So Paulo

SP

Carlos Eduardo Grivol Jnior


Marcos Vincius Nunes de Souza
Bruno Limaverde Vilar Lbo

Greice Calloni

Germano Brunatto F. Miranda

Pedro Victor Barbosa Nolto


MENO HONROSA
Fabiano Malard de Arajo
Luiz Felipe Guain Teixeira
Aline Tvora da Silva

Col. de Aplicao-UFPE

Recife

PE

Col. Marista

So Lus

MA

Juliana Barbosa Echenique

IF-RS

Pelotas

RS

Luiz Henrique Freire Arajo

Militar

Rio de Janeiro

RJ

Marina Toms Teixeira Carvalho

CEFET

Belo Horizonte

MG

Bom Jesus Lourdes

Curitiba

PR

Mater Amabilis

Guarulhos

SP

Guilherme Serra Baima

Emanuel Cassou dos Santos


Artur Carvalho Santos

Ano Internacional de Qumica



| 81

Programa Nacional
Olimpadas de Qumica

Resultado - Fase III


rica Saldanha Freire Simes

Ari de S

Fortaleza

CE

Gabriel de Souza Vieira Bi

Olimpo

Goinia

GO

Natlia Arago Dias

Mster

Fortaleza

CE

Charles Darwin

Vila Velha

ES

Olimpo

Braslia

DF

Marista So Jos

Rio de Janeiro

RJ

Agostiniano

Goinia

GO

Gabriel Challub Pires

Militar

Porto Alegre

RS

rica Regina Rodrigues da Silva

Cepron

Dom Pedro

MA

Wei Tzon Hackan Chang Colares

Lato Sensu

Manaus

AM

Militar

Recife

PE

Farias Brito

Fortaleza

CE

Unidade Jardim

So Bernardo

SP

Gustavo Haddad F. Sampaio Braga

Objetivo

S. Jos dos Campos

SP

Nathianne de Moura de Andrade

Farias Brito

Fortaleza

CE

Douglas Ramalho Q. Pacheco

Elite Vale do Ao

Ipatinga

MG

Pedro Lobo de C. Nascimento

Olimpo

Goinia

GO

Saulo Czar O. Parreira

Olimpo

Goinia

GO

Mackenzie

Braslia

DF

Alisson Bezerra Gomes

CEFET

Salvador

BA

Marcele Pereira Dias

Militar

Rio de Janeiro

RJ

Rodrigo Gomes Demeterko

Militar

Rio de Janeiro

RJ

Paulo Andr Herculano de Lima

Farias Brito

Fortaleza

CE

Jorge Junio Pedrosa Jordo

Lato Sensu

Manaus

AM

Motiva

Joo Pessoa

PB

Arthur Bernardo Flores Moreira

IF-RJ

Rio de Janeiro

RJ

Virna Mendona Sampaio Lima

Anchieta

Salvador

BA

IFS

Aracaju

SE

Bionatus

Campo Grande

MS

Leonardo Tessarolo Silva


Rafael Takeshi Sasaki Okida
Pedro Paulo C. A. Chaves
Jlia Resende Kanno

Wladimir Jos Lopes Martins


Breno Saldanha Sousa
Rafael Ferreira Antonioli

Rodolfo Prado Torres

Paloma Lins Barros

Jos Alexandre Arago Melo


Mariane Higa Shinzato
82 |

Ano Internacional de Qumica


Olimpada Brasileira
de Qumica 2011

Resultado - Fase III


Jayme Mendona Ramos

Nacional

Serra

ES

Dom Barreto

Teresina

PI

IF-RJ

Rio de Janeiro

RJ

Dom Barreto

Teresina

PI

Thales Galvo Costa

IF-RJ

Rio de Janeiro

RJ

Lucas Koji Matsuzaki

Universitario

Londrina

PR

CEFET

Belo Horizonte

MG

Gustavo Henrique R. Guimares

Bernoulli

Belo Horizonte

MG

Murilo Vieira Santa Brbara

Olimpo

Goinia

GO

Leonardo da Vinci

Vitria

ES

Marcos Alcino S. Siqueira Marques Jr.

Dom Barreto

Teresina

PI

Vincius Cleves de Oliveira Carmo

Helyos

Feira da Santana

BA

Arthur Chang Nogueira

Bernoulli

Belo Horizonte

MG

Jos Marques Neto

Ari de S

Fortaleza

CE

Motiva

Campina Grande

PB

Liberato Salzano

Novo Hamburgo

RS

Objetivo

Palmas

TO

William Veber Moiss da Silva

Militar

Porto Alegre

RS

Bruno Lunardi Gonalves

Militar

Braslia

DF

Boa Viagem

Recife

PE

Charles Darwin

Vitria

ES

Bom Jesus Lourdes

Curitiba

PR

Master

Fortaleza

CE

Leonardo da Vinci

Vitria

ES

Dom Barreto

Teresina

PI

Bruno de Oliveira Lima

Col. de Aplicao-UFPE

Recife

PE

Antenor Teixeira Neto

Anchieta

Salvador

BA

Universitario

Curitiba

PR

Anchieta

Salvador

BA

Francisco Rodrigues da Cruz Jr.


Gabriel Moraes Silva
Jos Ribamar Pereira Neto

Mariana Gabriela de Oliveira

Joo Victor Fragoso Dias

Maria Gabriela Viana de Sa


Diego Del Fabro Kunzler
Nicolas Kemerich de Moura

Victor Matheus Galvo


Paloma Alves Bezerra Morais
Lucas Pietro Galvam
Tain Nobre Barros Rodrigues
Henrique Ton Azevedo Giacomin
Tadeu Meneses de Carvalho

Jean Lucas Ramos


Iago Almeida Neves

Ano Internacional de Qumica



| 83

Programa Nacional
Olimpadas de Qumica

Resultado - Fase III


Eduardo Vincius Colman da Silva

Marista

Cascavel

PR

Amanda Sara Cavalcante Souza

Expresso

Apar. de Goinia

GO

Jose Victor Machado Nascimento

Ari de S

Fortaleza

CE

Janaina Gomes C. E. Mascarenhas

Dom Barreto

Teresina

PI

Joo Pedro Coelho de Sousa Lima

Elite Vale do Ao

Ipatinga

MG

Victor Augusto Souza de Oliveira

IF-RJ

Rio de Janeiro

RJ

Daniel Roediger

Etapa

So Paulo

SP

Victor Hugo V. S. Oliveira

IF-RJ

Rio de Janeiro

RJ

Educao Criativa

Belo Horizonte

MG

Charles Darwin

Vitria

ES

Dom Barreto

Teresina

PI

Etapa

So Paulo

SP

Adinaildo Gomes Paes Jr.

Elite Vila dos Cabanos

Belm

PA

Caio Eddie de Melo Alves

Lato Sensu

Manaus

AM

Andr Lopes Evangelista Dias

Madre Villac

Teresina

PI

Magnum Agostiniano

Belo Horizonte

MG

Alternativo

Aracaju

SE

Charles Darwin

Vila Velha

ES

IFES

C. de Itapemirim

ES

Ari de S

Fortaleza

CE

Lato Sensu

Manaus

AM

Mariana Brito do Couto

IF-RJ

Rio de Janeiro

RJ

Isabel Lopes da Silveira

Bom Jesus Lourdes

Curitiba

PR

Bernoulli

Belo Horizonte

MG

Motiva

Campina Grande

PB

Martina Vitria Flach Dietrich

Liberato Salzano

Novo Hamburgo

RS

Wilson Vieira da Silva Jnior

Dom Barreto

Teresina

PI

Matheus Leal de Souza

Antonio Vieira

Salvador

BA

Bom Jesus Centro

Curitiba

PR

Gleyca Lazarino de Almeida


Douglas Toledo Camilo
Lydia Pearce Pessoa de Aguiar
Tbata Cludia Amaral de Pontes

Larissa Dures Caminhas


Raphael Nercessian Corradini
Igor Pittol de Carvalho
Joo Lucas Maehara Said dos Reis
Alyne Mara de Alencar Justa
Carolina Carvalho Tavares

Luciana de Oliveira Botelho


Bruno Alves Mota Rocha

Mariana Sanchez Malagutti


84 |

Ano Internacional de Qumica


Olimpada Brasileira
de Qumica 2011

Resultado - Fase III


Mariana Feitosa Custdio

Cepron

Dom Pedro

MA

GGE

Recife

PE

Raphael Galate Baptista Ribeiro

Poliedro

Ribeiro Pires

SP

Mariana Lopes Ferreira Gomes

Millenium Classe

Goinia

GO

Jos Lucas de Alencar Saraiva

Motivo

Recife

PE

Adriel Garcia Maquin Senado

IF-AM

Manaus

AM

Anglo Vinci

Santana de Paraiba

SP

Liberato Salzano

Novo Hamburgo

RS

Gabriel F. P. Arajo

Lavoisier

Teresina

PI

Felipe Duarte Freire

Ponto de Ensino

Rio de Janeiro

RJ

Daniel Abrantes Formiga

Motiva

Joo Pessoa

PB

Bianca Rohsner Bezerra

Farias Brito

Fortaleza

CE

Marista de Cascavel

Toledo

PR

Tatiana Novaes Theoto

Leonardo da Vinci

Jundia

SP

Jairo de Sousa Araujo

ANGLO

Teresina

PI

Matheus Henrique D. Menezes

Agostiniano

Goinia

GO

Joo Henrique de A. V. Sistema

ELITE de Ensino

Rio de Janeiro

RJ

Matheus Salgueiro Castanho

Charles Darwin

Vila Velha

ES

Paula Nascimento Choinski

Bom Jesus Lourdes

Curitiba

PR

Santa Emlia

Olinda

PE

Bernoulli

Belo Horizonte

MG

Nsa Sra das Neves

Parnamirim

RN

Escola Tupy Sociesc

Joinville

SC

Sagrada Famlia

Blumenau

SC

Radharani Gita Kanesiro

Etapa

So Bernardo

SP

Victor Peres de Melo Goulart

Militar

Manaus

AM

Paulo Roberto Oliveira Martins

Lato Sensu

Manaus

AM

Nelly Carmen Ramirez Canelo

IF-AM

Manaus

AM

Universitario

Londrina

PR

Diogo Farkatt Kabbaz

Andr Caran Lima


Luis Fernando Esteves

Leonardo Bressan Anizelli

Kalil Lima Jardim Ferraz


Aline Lima Alves
Matheus Braga Furstenberger
Gustavo Cervi
Luiz Henrique Minatti

Hanna Tatsuta Galassi

Ano Internacional de Qumica



| 85

Programa Nacional
Olimpadas de Qumica

Resultado - Fase III


Thaynara Machado dos Santos

Agostiniano

Goinia

GO

Rayana Lorena Santos Florncio

Amadeus

Aracaju

SE

Letcia de Arajo Carvalho

Agostiniano

Goinia

GO

Ana Flvia Galvo Lopes

Dom Barreto

Teresina

PI

Victor Ripardo Siqueira

Lato Sensu

Manaus

AM

Mathues Dantas de Lucena

Motiva

Campina Grande

PB

Eduardo Cintra Simes

Damas

Recife

PE

Bruno Silva Mucciaccia

Leonardo da Vinci

Vitria

ES

So Jos

Apucarana

PR

Fundao Bradesco

Laguna

SC

Juliana Moises Poletti

Motiva

Joo Pessoa

PB

Isabella Gomes Santos

Charles Darwin

Serra

ES

Salesiano

Aracaju

SE

Alisson Ribeiro Lucena

Motiva

Campina Grande

PB

Pedro Henrique Favaro

So Jos

Apucarana

PR

Sag. Corao de Maria

Vitria

ES

Anchieta

Salvador

BA

Educao Criativa

Ipatinga

MG

Lucas Prado Chagas

Lato Sensu

Manaus

AM

Carolina Bordinho

Liberato Salzano

Novo Hamburgo

RS

Hlder Lima da Rocha

Aplicao

Aracaju

SE

Tarso Saideles Pizarro

Politcnico da UFSM

Santa Maria

RS

Laerson Saraiva Verissimo

Motiva

Campina Grande

PB

Felipe Sousa de Andrade

GGE

Recife

PE

Danielle Oliveira de Souza

Ideal

Belm

PA

EREM Prof. Adauto Carvalho

Recife

PE

Motiva

Joo Pessoa

PB

Ismael Roggia Silva

Politcnico da UFSM

Santa Maria

RS

Joo Pedro Lima da Costa

Politcnico da UFSM

Santa Maria

RS

Klynsmann D. C. Bagatini
Augusto Henrique dos Santos

Lucas Bito Maynart

Ana Carolina T. R. de Medeiros


Leandro Gomes Santana de Souza
Mariana Rezende de Oliveira

Carlos Henrique da Silva


Giovanni Simoes Medeiros

86 |

Ano Internacional de Qumica


Olimpada Brasileira
de Qumica 2011

Resultado - Fase III


Maral Buttenberg

Sagrada Famlia

Blumenau

SC

Henrique Arenare de Oliveira

Militar

Belo Horizonte

MG

Lucas Sinicio Molina

Militar

Braslia

DF

Thania Gomes Braz

Colgio Arnaldo

Belo Horizonte

MG

Posiville

Joinville

SC

Millenium Classe

Goinia

GO

Eduarda Karynne Souza

Erem Luiz Alves Silva

Recife

PE

Daniel Schulz Nogueira

IF-RJ

Rio de Janeiro

RJ

Matheus Fernando C.L. Nascimento

Dom Barreto

Teresina

PI

Amanda Pierre de Moraes Moreira

Crescimento

So Lus

MA

Franklin Mendoa Floresta

Alternativo

Aracaju

SE

Objetivo Indaiatuba

Indaiatuba

SP

Madre Villac

Teresina

PI

IF-RJ

Rio de Janeiro

RJ

So Domingos

Vitria

ES

IF-PE

Recife

PE

Eduardo Ari Turconi

Marista Rosrio

Porto Alegre

RS

Sarah Santana Diniz

Coesi

Aracaju

SE

Amanda Vielmo Sagrilo

Politcnico da UFSM

Santa Maria

RS

Eduardo Brandt Nunes

Bionatus

Campo Grande

MS

Brenda Dias Marques

Cepron

Dom Pedro

MA

Gabriel Alves Carrio

Prevest

Goinia

GO

Gustavo Coqueiro de Araujo

Anchieta

Salvador

BA

Leonardo da Vinci

Braslia

DF

Pedro Henrique Silva dos Anjos

Militar

Rio de Janeiro

RJ

Fernando Carlos Brando Filho

Expresso

Goinia

GO

Danilo Pequeno

Motiva

Campina Grande

PB

Yan Moura Quintino

IF-AL

Macei

AL

Ateneu

Londrina

PR

Lucas Tadeu Krger Poffo


Kau Ueda Silveira

Ricardo Stary Rodrigues


Rennan Martins Viana
Bernardo O. de Carvalho
Amanda Giuberti Poltronieri
Erika Rodrigues Vieira de Macdo

Jos Alberto Souza Abdon

Giovanne Adalton Keller Stivari

Ano Internacional de Qumica



| 87

Programa Nacional
Olimpadas de Qumica

Resultado - Fase III


Guilherme Lima

Bom Jesus Lourdes

Curitiba

PR

CEFET

Salvador

BA

Millenium Classe

Goinia

GO

Eduardo Jose Rocha de Moraes

Amadeus

Aracaju

SE

Alequine Batista de Lima

Lourdinas

Campina Grande

PB

Julie Fujishima

Moderno

Macap

AP

Leonardo da Vinci

Braslia

DF

Joo Pedro de C. Magalhes

Militar

Recife

PE

Joo Pedro Santos Wanderley

Motivo

Recife

PE

Marina Perbone Len

Colgio Classe A

Porto Velho

RO

Alexandre Chaves Filho

Conexo Aquarela

Macap

AP

Politcnico da UFSM

Santa Maria

RS

Jos Matheus Santos Pereira


Adonias de Freitas Nunes

Rafaela Nunes Couto

Luiz Felipe Manfio dos Santos


A cincia est destinada a desempenhar um papel cada
vez mais preponderante na produo industrial. E as naes
que deixarem de entender essa lio ho inevitavelmente de
ser relegadas posio de naes escravas: cortadoras de lenha e carregadores de gua para os povos mais esclarecidos.
Lord Rutherford
88 |

Ano Internacional de Qumica


Olimpada Brasileira
de Qumica 2011

Seletiva internacional

Processo seletivo
para as olimpadas internacionais
Aps concludas as trs primeiras fases da Olimpada Brasileira de Qumica realiza-se a solenidade nacional de premiao na qual so agraciados
com medalhas ouro, prata e bronze e com certificados de Meno Honrosa
os estudantes que tiveram maior destaque. Essa premiao acontece anualmente na ltima semana do ms de novembro. A olimpada continua com
o processo seletivo para escolher os estudantes que representam o Brasil
nas olimpadas internacionais: International Chemistry Olympiad e Olimpada Iberoamericana de Qumica. Todos os estudantes agraciados com medalhas so convocados para essa seletiva constituda, tambm, de trs fases: a
primeira, FASE IV, trata de avaliar conhecimentos de laboratrio, a FASE V
constituda pelo curso de Aprofundamento e Excelncia em Qumica o qual
rene 15 estudantes classificados na fase anterior, o curso realizado em
uma das universidades participantes que possui curso de ps-graduao em
qumica. Em 2011, o curso foi realizado no Instituto de Qumica da UNICAMP
no prximo ano ser na UFMG. A FASE VI finaliza o processo seletivo com
a escolha dos quatro estudantes que compem a delegao brasileira nas
olimpadas internacionais de qumica.
A FASE IV da OBQ-2010 foi realizada em 1 fevereiro de 2011, um exame fundamentado na exibio de um filme com experimentos de qumica
acompanhado de um questionrio. Para esta fase foram convocados todos
os estudantes medalhados na FASE III da OBQ-2010, o total de trinta e cinco estudantes, listados em www.obq.ufc.br/resultadovideo2011.htm. Quinze
desses estudantes se classificaram para o Curso de Aprofundamento e Excelncia em Qumica que aconteceu de 21/03 a 01/04/2011 nas dependndias
do Instituto de Qumica da Unicamp (Fase V da OBQ-2010).
Os professores que ministraram esse curso foram os responsveis pela
elaborao e correo do exame aplicado em 16/04/2011 (Fase VI da OBQ2010) e definiu os estudantes que representaram o Brasil nas olimpadas internacionais. Participaram do X Curso de Aprofundamento e Excelncia em
Qumica os seguintes estudantes:

Ano Internacional de Qumica



| 89

Programa Nacional
Olimpadas de Qumica

Seletiva internacional
Raul Bruno Machado da Silva

Farias Brito

CE

Davi Rodrigues Chaves

Ari de S

CE

Tabata Claudia Amaral de Pontes

Etapa

SP

Davidson Anthony Arago Freire

Ari de S

CE

Daniel Arjona de Andrade Hara

Objetivo Granja Viana

SP

Lara Mulato Lima

Ari de S

CE

Taynara Carvalho Silva

MASTER

CE

Bruno Limaverde Villar Lbo

Farias Brito

CE

Breno Saldanha Sousa

Farias Brito

CE

Sergio Pereira de Oliveira Jnior

Farias Brito

CE

Bianca Rohsner Bezerra

Farias Brito

CE

Emerson Holanda Marinho

Farias Brito

CE

Yuri Jernimo Moreira

Farias Brito

CE

Natlia Arago Dias

MASTER

CE

Pedro Victor Barbosa Nolto

Instituto Dom Barreto

PI

Os quatro primeiros alunos listados acima representaram o Brasil na 43rd International Chemistry Olympiad realizada em Ankara-Turquia e o estudante
Daniel Arjona substituiu Davidson Anthony na delegao que participou na
XVII Olimpada Iberoamericana de Qumica realizada em Teresina- Brasil.

As coisas que queremos e parecem impossveis s


podem ser conseguidas com uma teimosia pacfica.
90 |

Mahatma Gandi
Ano Internacional de Qumica

Olimpada Brasileira
de Qumica 2011

Seletiva internacional

Estudantes convocados para participar do X Curso de Aprofundamento e


Excelncia em Qumica ministrado no Instituto de Qumica da UNICAMP.

No dia em que o homem se der conta de seus profundos


equcocos, ter terminado o progesso da cincia.
Marie Curie (1867-1934)

Ano Internacional de Qumica



| 91

Programa Nacional
Olimpadas de Qumica

Seletiva internacional

Estudantes participantes no X Curso de Aprofundamento e Excelncia em Qumica e professores do


Instituto de Qumica da UNICAMP que ministraram o curso.

92 |

Ano Internacional de Qumica


Olimpada Brasileira
de Qumica 2011

Seletiva - Fase IV

Processo seletivo para as Olimpadas Internacionais


Olimpada Brasileira de Qumica - 2010 Fase IV
Seletiva para a 43rd International Chemistry Olympiad, Turquia
Filme exibido em 01.02.2011 , 14 horas
Caro estudante,
Este exame de cunho experimental tem por finalidade selecionar os 15 (quinze) estudantes que participaro do X Curso de Aprofundamento e Excelncia
que ser ministrado na UNICAMP (Campinas) no perodo de 21 de maro
a 1 de abril prximo. Um novo exame ser aplicado, aps a realizao do
curso, para definir a equipe (4 estudantes) que representar o Brasil na 43rd
IChO que acontecer em julho prximo, em Ankara - Turquia.
Voc dispe de 2h e 30 minutos para ver o vdeo e responder as questes
deste exame.
INSTRUES
1. Veja atentamente, no televisor ou na tela de projeo, as imagens do filme
que contm os fundamentos deste exame.
2. Seu coordenador, inicialmente, exibir a gravao completa do exame (18
min) e, a seguir, apresentar cada parte separadamente. Se necessrio, repassar as imagens vrias vezes at esclarecer suas dvidas.
3. Leia as perguntas relativas a cada experimento e escreva a resposta nas
folhas oficiais.
4. A prova est dividida em 7 (sete) atividades.
5. Os resultados sero encaminhados at 10/02 para seu coordenador (e tambm diretamente para voc, caso tenha e-mail). Veja o resultado na internet
em www.obquimica.org .
6. Os estudantes convocados para a etapa seguinte (Curso na UNICAMP) devem ficar atentos nossa pgina na internet. Provavelmente, na prxima
semana, ser divulgada a lista de exerccios preparatrios para a Olimpada Internacional de Qumica. importantssimo que o estudante resolva
todos esses exerccios com antecedncia e discuta-os durante o curso que
ser ministrado na Universidade Estadual de Campinas.

Ano Internacional de Qumica



| 93

Programa Nacional
Olimpadas de Qumica

Seletiva - Fase IV
Questionrio
Atividade 1

20 pontos

A - D o nome e indicaes de uso para cada uma das vidrarias aresentadas..


Atividade 2

10 pontos

A Indique o nome desse equipamento e a sua funo


Atividade 3

20 pontos

A Diferencie a utilizao de cada um dos sistemas indicados.


B D o nome de cada vidraria, material e equipamento dos sistemas A e B.
Atividade 4

20 pontos

A Explique a diferena dos pontos de ebulio dos ismeros e indique as estruturas de A e B.


B Qual a funo da coluna de Vigreaux?
Atividade 5

10 pontos

A Explique a diferena observada nas coloraes dos dois bqueres..


Atividade 6

10 pontos

A Indique o gs que formado em cada tubo de ensaio


Atividade 7

10 pontos

A Indique a representao estrutural da amida aromtica produzida (slido


formado).
94 |

Ano Internacional de Qumica


Olimpada Brasileira
de Qumica 2011

Seletiva Fase VI

Olimpada Brasileira de Qumica


OBQ - 2010 FASE VI

Seletiva para a 43rd International Chemistry Olympiad


Ankara - Turquia

16.04.2011

NOTA: Observe os algarismos significativos.

CADERNO DE QUESTES
QUESTO 1

(10 pontos)

Uma alquota de 5,00 mL de uma amostra de conhaque foi diluda para


1000,00 mL em balo volumtrico. Desta soluo diluda, uma alquota de
25,00 mL foi submetida a uma destilao. O etanol destilado foi recolhido
em 50,00 mL de uma soluo de dicromato de potssio 0,02000 mol L-1. A
oxidao do etanol leva a formao de cido actico. Aps resfriamento, foram adicionados 20,00 mL de uma soluo de Fe2+ 0,1253 mol L-1. O excesso
do Fe2+ foi titulado com 7,46 mL de uma soluo padro de dicromato de
potssio 0,02000 mol L-1.
a) Escreva todas as reaes qumicas envolvidas na anlise;
b) Especifique os agentes oxidantes e redutores;
c) Calcule a percentagem (m/v) de etanol no conhaque.

QUESTO 2

(15 pontos)

A gua presente nos poos de petrleo, denominada gua de formao


uma soluo salina muito concentrada que contm cloreto de sdio como
sal majoritrio e tambm outros ons ,como: Ca2+, Sr2+, Ba2+, carbonato e hidrogeno carbonato. O pH da gua de formao 7,2.
Uma amostra de 1L de gua de formao foi recolhida de um poo de
petrleo e acidificada. A acidificao resultou na liberao de 1L de gs
medido a 27 C e 1 atm. Aps a acidificao foi adicionado sulfato de sdio
amostra resultando em 5g de precipitado branco. A razo entre as concentraes dos ons Ca2+:Sr2+:Ba2+, na gua de formao igual a 10.000:100:1.

Ano Internacional de Qumica



| 95

Programa Nacional
Olimpadas de Qumica

Questes
a) Escreva as reaes que resultaram na liberao de gs pela acidificao da
gua de formao.
b) Escreva as reaes que resultaram na formao do precipitado branco.
c) Calcule a concentrao dos ons carbonato e hidrogeno carbonato presente na gua de formao. pKa HCO3- = 6,3.
d) Calcule a concentrao de Ca2+, Sr2+ e Ba2+ na gua de formao considerando que houve precipitao completa dos ons.
e) Para os sais insolveis dos ons dos ons clcio, estrncio e brio, que formam o precipitado branco , as constantes de solubilidade so respectivamente: 3.10-5; 3.10-7 e 10-10. Calcule a razo entre as concentraes ons na
soluo aps precipitao.

QUESTO 3

(10 pontos)

O modelo atmico proposto por Bohr limitado a sistemas hidrogenides, cuja energia (em eV) do estado fundamental dada por:
En = - 13,6 Z2 / n2
(1).
a) Calcule as 1 e 2 energias de ionizao (em kJ/mol) para 1 mol de tomos
de H e He, respectivamente.
b) Um tomo de H no estado fundamental absorve um fton de luz de comprimento de onda igual a 97,2 nm. Ele ento emite um fton de comprimento
de onda igual a 486 nm. Qual o estado final do tomo em questo?

QUESTO 4

(15 pontos)

1) Muitos compostos binrios obtidos com halognios e oxignio so


conhecidos apesar de alguns deles apresentarem instabilidade elevada.
Dentre esses compostos, o difluoreto de oxignio (PF = -224 oC, PE=-145 oC)
e o difluoreto de dioxignio (PF = -154 oC, PE=-57 oC) so os nicos exemplos
obtidos com fluor e tambm as nicas molculas em que o elemento oxignio
apresenta um Nox positivo. O primeiro composto pode ser preparado atravs
da reao entre gs fluor e uma soluo aquosa alcalina. J o segundo, apenas
pela fotlise de uma mistura gasosa dos dois elementos.
96 |

Ano Internacional de Qumica


Olimpada Brasileira
de Qumica 2011

Seletiva Fase VI
a) Apresente as equaes qumicas globais que descrevem as snteses dos
dois compostos binrios citados no texto.
b) Apresente as estruturas de Lewis, os nmeros de oxidao e as cargas formais
de ambos os compostos.
c) Apresente a geometria dos compostos e a hibridizao em ambos os
casos.
2) Em oposio restrita qumica dos compostos binrios de fluor e oxignio,
os anlogos de cloro apresentam-se em vrios estados de oxidao e uma
variedade de estruturas. O dixido de cloro o nico produzido em larga
escala e encontra aplicao como agente alvejante na indstria do papelcelulose e tambm na desinfeco de guas para consumo. A obteno
industrial realizada a partir da reduo do on clorato (de fonte mineral)
em soluo aquosa usando gs clordrico ou dixido de enxofre em meio
fortemente cido.
a) Apresente as equaes qumicas globais que descrevem as snteses do
dixido de cloro a partir dos dois agentes redutores citados no texto.
b) Apresente as estruturas de Lewis, os nmeros de oxidao e as cargas
formais de todos os compostos (reagentes e produtos) envolvidos nas
equaes do tem anterior.
c) Apresente a geometria dos compostos e a hibridizao em todos os
casos.
d) Apresente uma explicao para o fato do cloro apresentar mais compostos
binrios com o oxignio do que o flor.

Ano Internacional de Qumica



| 97

Programa Nacional
Olimpadas de Qumica

Questes
QUESTO 5

(10 pontos)

Pagina 80 - tem dados faltando no quadro

Considere
os seguintes
Considere os seguintes
dados termodinmicos:

dados termodinmicos:

substncia

f H 0298 /
(kJ mol-1)

S 0298 /

(J K-1 mol-1)

C p0 298 /

Glicose(s)
O2(g)
CO2(g)
H2O(l)
H2O(g)

1273,2
0
393,1
285,6
241,6

211,9
204,8
213,4
69,8
188,5

(J K-1 mol-1)
218,6
29,3
37,1
75,2
33,5

Responda as seguintes questes:


a) Calcule o valor da variao de energia livre molar na temperatura de
Tambem na pagina 80
298 K e 1 atm, para a reao de combusto completa para glicose.
b) Calcule
a variaodedeligaes
entropia
para
a reao
na muito
temperatura
O tratamento
espectroscpico
molar
pode ser
feito
de forma
aproximada
de 298
K e 1 atm
e justifique
com base
na reao
o valor
positivona
dacaixa
considerando-se
um modelo
denominado
de partcula
na caixa.
No modelo
da partcula
unidimensional,
a energia de um determinado nvel quntico para uma cadeia carbnica pode ser
mesma.
definida por:
En =

c) Calcule o valor da variao de energia livre molar na temperatura de

n2h2
400 K e 1 atm, para a reao de combusto completa para glicose.
8mL2

sendo h a constante de Planck, m a massa da partcula, L o tamanho da cadeia molecular e n = 1,


2, 3, ...
Este modelo pode ser aplicado considerando-se as seguintes condies: 1) Cada nvel de energia da
QUESTO 6 (15 pontos)
partcula na caixa s pode acomodar no mximo dois eltrons; 2) A energia total da distribuio
eletrnica corresponde soma da energia de cada um dos eltrons, por exemplo, para cinco eltrons
O tratamento
espectroscpico
penergia,
pode ser feito
forma muitoser:
ocupando
os
primeiros
nveisde ligaes
de
a deexpresso

aproximada
denominado de partcula na caixa.
h 2 considerando-se
h 2 modelo
2 2 h 2 32 h 2 um
Etotal

+ 2
+ na2 caixa
=
[2 12 + 2 2 2 + 3a2 ]energia
; 3) A transio
eletrnica
do ltimo
No= 2modelo
unidimensional,
de um
determina2 da partcula
2
2
8mL
8mL 8mL 8mL
do nvel quntico para uma cadeia carbnica pode ser 2definida por:
h
c
(2nmax + 1) = h = h , sendo
nvel ocupado para o primeiro nvel vazio definida como: E =
2

8mL

nmax o nmero quntico do ltimo nvel ocupado, c a velocidade da luz, o comprimento de onda e
a freqncia da radiao.
Considere os trs compostos orgnicos a seguir e responda os itens abaixo:

sendo h a constante de Planck, m a massa da partcula, L o tamanho da cadeia


molecular e n = 1, 2, 3, ...
Este modelo pode ser aplicado considerando-se as seguintes condies:
98 |

Ano Internacional de Qumica


Olimpada Brasileira
de Qumica 2011

Seletiva Fase VI
1) Cada nvel de energia da partcula na caixa s pode acomodar no mximo
dois eltrons;
2) A energia total da distribuio eletrnica corresponde soma da energia
de cada um dos eltrons, por exemplo, para cinco eltrons ocupando os
primeiros nveis de energia, a expresso ser:

3) A transio eletrnica do ltimo nvel ocupado para o primeiro nvel vazio


definida como:

sendo nmax o nmero quntico do ltimo nvel ocupado, c a velocidade da


luz, l o comprimento de onda e n a freqncia da radiao.
Considere os trs compostos orgnicos a seguir e responda os itens abaixo:
CH3 - (CH= CH)3 - CH3
CH3 - (CH3= CH)4 - CH3
CH3 - (CH = CH)5 - CH3
a) Faa um desenho de cada uma das molculas acima explicitando todas as
ligaes qumicas e indique quais so os tomos de carbono que possuem
ligaes p.
b) Determine o nmero de eltrons p em cada molcula, faa um diagrama
de energia qualitativo distribuindo esses eltrons para cada uma das trs
molculas e determine o nmero nmax do ltimo nvel de energia ocupado.
c) Calcule a energia total p para o composto com frmula CH3(CH=CH)4CH3.

Ano Internacional de Qumica



| 99

Determine o nmero de eltrons em cada molcula, faa um diagrama de energ


qualitativo distribuindo esses eltrons para cada uma das trs molculas e determine
nmax doNacional
ltimo nvel de energia ocupado.
nmeroPrograma
Olimpadas
de
Qumica
Calcule a energia total
para o composto com frmula CH3(CH=CH)4CH3.
Identifique
qual
das
Questes trs molculas deve apresentar a transio eletrnica com mai
comprimento de onda e calcule o valor desse comprimento de onda em nm (10-9m
assumindo que o comprimento de cada uma das ligaes C-C e C=C so iguais a 1,40x10-10m
d) Identifique qual das trs molculas deve apresentar a transio eletrnica
O valor com
experimental
do comprimento
de e
onda
parao avalor
transio
entre o ltim
maior comprimento
de onda
calcule
desseeletrnica
comprimento
-9
(CH=CH)
CH
orbital ocupado
e
o
primeiro
orbital
vazio
para
a
molcula
CH
3
3
3 igual a 2
de onda em nm (10 m) assumindo que o comprimento de cada uma das
-10
nm. Calcule
o comprimento
da cadeia
em nm utilizando o modelo da partcula
ligaes
C-C e C=C so
iguais acarbnica
1,40x10 m.
caixa a partir do comprimento de onda experimental. Qual deve ser a razo entre
e) O valor experimental do comprimento de onda para a transio eletrnica
comprimento da cadeia estimada a partir do dado experimental em
p entre o ltimo orbital ocupado e o primeiro orbital vazio para a molcula CH3(CH=CH)3CH3 igual a 274 nm. Calcule o comprimento da cadeia
carbnica p em nm utilizando o modelo da partcula na caixa a partir do
comprimento de onda experimental. Qual deve ser a razo entre o comprimento da cadeia p estimada a partir do dado experimental em relao
ao valor obtido como a soma dos comprimentos de ligao do item (d)?

gina 82 a 85
QUESTO 7

(15 pontos)

Bacillus
pumilus
e apresenta
ativida
bstncia denominada
AI-77-B foi AI-77-B
isoladafoido
A substncia denominada
isolada
do Bacillus
pumilus
e aprelamatria
e
gastroprotetora,
o
que
a
coloca
em
grande
vantagem
em
relao
a
senta atividade antiinflamatria e gastroprotetora, o que a coloca em grande
lamatrios
tradicionais.
vantagem
em relao aos antiinflamatrios tradicionais.

OH O
O

HO H H NH2
H H
N
CO2H
H O HO H

AI-77-B

Uma de suas snteses envolve o acoplamento dos fragmentos I, que contm


o anel aromtico, e o fragmento II.

100 |

Ano Internacional de Qumica


Olimpada Brasileira
de Qumica 2011

Uma de suas snteses envolve o acoplamento dos fragmentos


I, queFase
contm
Seletiva
VIo anel aromtic
gmento II.

OH O

OH
O

HO
NH2
H

NH2

O
II

Parte
Parte A: sntese do intermedirio
I A: sntese do intermedirio I

Na primeira sntese racmica do intermedirio I foi possvel controlar a


meira sntese racmica do intermedirio I foi possvel controlar a configurao relativa dos do
configurao relativa dos dois centros estereognicos presentes. Esta sntese
s estereognicos
Esta
comea comnoointermedirio
lcool A, que
transformado
comeapresentes.
com o lcool
A, sntese
que transformado
B, aps
reao
edirio B, com
apsPCC
reao
com PCCde(clorocromato
piridnio). Este
intermedirio
(clorocromato
piridnio). Estede
intermedirio
B ser
submetido B se
etido a umaa reao
de Wittig
com o com
reagente
C para formar
olefinaaD,olefina
cuja configurao
n
uma reao
de Wittig
o reagente
C para aformar
D, cuja
epresentadaconfigurao
no esquema.no
Aps
a obteno
de D,
hidrlise Aps
da funo
amida de
emD,
meio
est
representada
noaesquema.
a obteno
a bsi
hidrlise
da
funo
amida
em
meio
bsico
forma
o
cido
E,
que
em
seguida
o cido E, que em seguida sofre uma reao que forma predominantemente a lactona
sofre uma
reao que
forma predominantemente
lactona F, embora
a G,aps um
a a G, tambm
seja formada.
A lactona
F ser convertida noa intermedirio
H, que,
tambm seja
lactona
no intermedirio
H, que,
o de hidrogenao
e formada.
remoo Ado
grupoF ser
de convertida
proteo da
hidroxila fenlica
produz
aps uma reao de hidrogenao e remoo do grupo de proteo da hiedirio I.
droxila fenlica produz o intermedirio I.

nta-se:

al a estrutura do intermedirio B?
al a estrutura do composto C, que ir reagir com B?
l ser a configurao da ligao dupla presente na olefina D?
e reagentes so utilizados para converter o cido E nas lactonas F e G?
resente a estrutura do intermedirio da reao que transforma o cido E na lactona F?.
resente corretamente a configurao relativa dos centros estereognicos presentes na lactona
al a estrutura do produto H? Represente corretamente a configurao relativa dos se
s estereognicos.

Ano Internacional de Qumica



| 101

Programa Nacional
Olimpadas de Qumica

Questes
OG O

OG O
N(CH3)2
OH

PCC

CH2Cl2

OG O
hidrlise

N(CH3)2

OH

bsica

G = grupo de proteo
?

OG O
NaN3

DMF

intermedirio I
OG O

OG O
PCC

N(CH3)2
OH

CH2Cl2

OG O

N(CH3G
)2

hidrlise

OH

bsica

A H reage em meio cido com a cetona D


h) Pergunta-se:
O composto
J, para formar os produtos KEe L. D as
estruturas
de
Qual delesdo
serintermedirio
o predominante?B?
a)
K proteo
ae L.
estrutura
G =Qual
grupo de

b) Qual a estrutura do composto C, que ir reagir com B?


?
HBF4 OG O
c) Qual ser a configurao da+ ligao
dupla presente
H
K
+ na Lolefina D?
O so utilizados para converter
NaN3
O
d) Que reagentes
o cido
E nas lactonas F e G?
H
DMF
e) Represente a estrutura
do
intermedirio
da
reao
que
transforma o cido
J
I
intermedirio I
F
E na lactona F?
f) B:Represente
corretamente
Parte
sntese do intermedirio
II a configurao relativa+ dos centros estereognicos
presentes na lactona F.
G
As g)
duas
primeiras
etapas da sntese
do intermedirio
II so transformaes
sucessivas
do do ter de
Qual
a estrutura
do produto
H? Represente
corretamente
a configurao
enol A. Na primeira etapa (etapa A) temos uma reao de cicloadio para formar o intermedirio B,
relativa dos seus centros estereognicos.
que
convertido
na meio
lactona
C. Aps
gruposK funcionais,
h) Odepois
composto
H reage em
cido
com asucessivas
cetona J, interconverses
para formar os de
produtos
e L. D as
O
composto
H reage
em
meio
cido
com
a cetona
J, para
formar
os produtos
chegamos

lactona
D,
cuja
hidrlise
levar
ao
precursor
imediato
do
intermedirio
II, aps a
estruturas de K e L. Qual deles ser o predominante?
K e L.do grupo protetor da hidroxila secundria.
remoo
+

HBF4

Parte
B: sntese
do intermedirio
h) D
as estruturas
de K IIe

L. Qual deles ser o predominante?

As duas primeiras etapas da sntese do intermedirio II so transformaes sucessivas do do ter de


enol A. Na primeira etapa (etapa A) temos uma reao de cicloadio para formar o intermedirio B,
que depois convertido na lactona C. Aps sucessivas interconverses de grupos funcionais,
chegamos lactona D, cuja hidrlise levar ao precursor imediato do intermedirio II, aps a
Ano Internacional de Qumica
remoo
102 | do grupo protetor da hidroxila secundria.

Olimpada Brasileira
de Qumica 2011

Seletiva Fase VI
Parte B: sntese do intermedirio II
As duas primeiras etapas da sntese do intermedirio II so transformaes sucessivas do ter de enol A. Na primeira etapa (etapa A) temos uma
reao de cicloadio para formar o intermedirio B, que depois convertido
na lactona C. Aps sucessivas interconverses de grupos funcionais, chegamos lactona D, cuja hidrlise levar ao precursor imediato do intermedirio
II, aps a remoo do grupo protetor da hidroxila secundria.
BnO
etapa A

GO

etapa B

OMe

BnO
GO

Cl
*

* O
CO2Me

Cl
O

H2N
O

Bn = CH2Ph
G = grupo de proteo

furano
E

EtO

OG
D

Pergunta-se:

Pergunta-se:
a) Quais so os reagentes da etapa A e qual a estrutura da espcie que reagir com o ter de enol
A?Quais so os reagentes da etapa A e qual a estrutura da espcie que reaa)
b) D reagentes para a etapa B.
gir com o ter de enol A?

c) Qual a configurao relativa dos centros estereognicos assinalados (*) na estrutura do composto

C? D reagentes para a etapa B.


b)

d) Qual o produto formado na reao do ter de enol A com o furano?

c) Qual a configurao relativa dos centros estereognicos assinalados (*) na


estrutura do composto C?
d) Qual o produto formado na reao do ter de enol A com o furano?

Ano Internacional de Qumica



| 103

Programa Nacional
Olimpadas de Qumica

Questes
QUESTO 8

(10 pontos)

1) Escreva as estruturas e os nomes do par de ismeros (A e B), de frmula


C5H9BrO2, cujos sinais dos espectros de RMN 1H esto listados abaixo. Os
espectros de infravermelho destes ismeros no apresentam sinais de absorbncia de OH.
Ismero A:
Sinais em d (ppm) = 4,4 (q, 1H); 4,2 (q, 2H); 1,8 (d, 3H) e 1,3 (t, 3H)
Ismero B:
Sinais em d (ppm) = 4,2 (q, 2H); 3,6 (t, 2H); 2,9 (t, 2H) e 1,3 (t, 3H)
2) No espectro de RMN 1H do composto C, de frmula C10H10O2, observamse os seguintes sinais:
d ppm) = 1,4 (t, 3H); 4,3 (q, 2H); 7,4 (t, 2H); 7,5 (t, 1H); 8,0 (d, 2H).
E no espectro de RMN 13C, do mesmo composto, observam-se 7 (sete) sinais em:
d ppm) = 14,4 (CH3); 60,8 (CH2); 128,4 (CH); 129,7 (CH); 130,9 (C); 132,8
(CH) e 166,3 (C).
Escreva a estrutura e o nome do composto C
Obs:
1. Ao lado de cada sinal de RMN 1H, esto apresentadas, entre parntesis,
as letras referentes s multiplicidades desses sinais (d=dupleto, t=tripleto,
q=quarteto) e o nmero de hidrognios relacionados aos mesmos (1H, 2H
ou 3H).
2. As multiplicidades apresentadas para os sinais na regio entre 7,0 e 8,0
ppm esto levando em conta apenas os acoplamentos orto.
Valores aproximados de faixas de deslocamentos qumicos de RMN 1H e 13C

104 |

Ano Internacional de Qumica


Olimpada Brasileira
de Qumica 2011

Seletiva Fase VI
Hidrognio/Carbono

dH (ppm)

dC (ppm)

CHx-C

0,8 - 1,6

5 - 40

CHx-C=C

1,6 - 2,3

10 - 30

CHxC=O

1,9 - 3,0

23 - 48

CHx-O

1,9 - 3,0

50 - 85

Alcenos

4,5 - 8,0

95 - 155

Aromticos

4,5 - 8,0

95 - 155

H-C=O

9,0 -10,0

190 - 220

COOH

10,0 -12,0

160 - 185

C=O (cetonas)

190 - 220

C=O (steres)

160 -185

CHx-Br

3,0 - 4,5

20 - 40

CHx x= 1, 2 ou 3

----------------------------------------------------------------------------------Constantes Fundamentais e Fatores de Converso:


h = 6,62 x 10-34 J.s; NA = 6,02 x 1023 mol-1;
= 1,602 x 10-19 J

c = 3,00 x 108 m.s-1; 1 eV

Ano Internacional de Qumica



| 105

Programa Nacional
Olimpadas de Qumica

43rd IChO

43rd International
Chemistry Olympiad
Problemas Experimentais
12 de Julho de 2011
Ankara, Turkey

PROBLEMA 1
Anlise de misturas de cloretos
A composio de uma soluo que contm apenas MgCl2 e NaCl pode ser
determinada por um mtodo de titulao indireta onde se faz uma titulao com precipitao que determina a quantidade total de cloreto presente,
acompanhada por uma titulao com formao de complexo que determina
a quantidade de ons magnsio. Uma tcnica comum de titulao com precipitao usada para determinar a quantidade de ons cloreto presentes o
mtodo de Fajans. Neste procedimento argentimtrico, o nitrato de prata
usado como titulante na precipitao de ons cloreto presentes na soluo.
O ponto final detectado com o uso de um indicador de adsoro, tipicamente diclorofluorescena, um cido orgnico fraco. Antes do ponto final,
as partculas de cloreto de prata esto carregadas negativamente por causa da adsoro de ons cloreto em excesso presentes na soluo. Os nions
do indicador so repelidos pela superfcie das partculas de cloreto de prata
carregadas negativamente dando soluo uma cor amarelo-esverdeada.
Entretanto, aps o ponto de equivalncia, as partculas de cloreto de prata
adsorvem ons de prata. Desta maneira uma camada carregada positivamente formada atraindo os ons diclorofluoresceinato e exibindo uma cor rosaavermelhada. A dextrina usada para estabilizar as partculas de cloreto de
prata evitando a coagulao.
Por outro lado, a quantidade de ons magnsio presentes na soluo pode
ser determinada por uma titulao complexomtrica com cido etilenodiaminotetraactico, EDTA. Como um ligante hexadentado, EDTA forma quelatos com todos os ons metlicos, exceto com os alcalinos, numa razo de
106 |

Ano Internacional de Qumica


Olimpada Brasileira
de Qumica 2011

Exame Prtico
1:1 independentemente da carga do ction. Eriochrome Black T (EBT) um
indicador comum usado nas titulaes com EDTA. Quando o pH > 7,00 o
EBT impe uma cor azul soluo na ausncia de ons metlicos e forma uma
cor vermelha quando coordenado com os ons metlicos.
Neste experimento, ser determinado o teor de cloreto de uma soluo que
contm MgCl2 e NaCl usando o mtodo de Fajans. A concentrao de ons
magnsio ser determinada por uma titulao com EDTA.
Uma soluo de 100 mL preparada pela dissoluo de MgCl2 e NaCl em gua
fornecida como amostra desconhecida (unknown sample). O objetivo
determinar a concentrao de MgCl2 e NaCl em g/100 mL.
A. Determinao de cloreto total pelo Mtodo de Fajans
1. Usando uma pipeta de 10 mL, transfira uma alquota de 10,0 mL do frasco
rotulado como unknown solution para o Erlenmeyer de 250 mL. Complete o volume para aproximadamente 100 mL, adicionando gua destilada.
2. Pegue um dos tubos de Eppendorf fornecido em saco plstico com zipper rotulado como dextrin e transfira todo o seu contedo para o Erlenmeyer.
3. Adicione 5 gotas de soluo de indicador diclorofluorescena.
4. Anote a exata concentrao da soluo padro de AgNO3.
5. Encha uma das buretas com a soluo padro de AgNO3.
6. Titule a soluo desconhecida at toda a soluo ficar rosa-avermelhada.
7. Anote o volume de AgNO3 usado, em mL.
8. Use o mesmo Erlenmeyer, quando repetir a titulao. Antes disso, despeje
o contedo do Erlenmeyer no frasco rotulado com Aqueous Waste container e lave-o duas vezes com gua destilada.
B. Determinao de Mg2+ por titulao direta com EDTA
1. Encha a segunda bureta com a soluo padro de EDTA.
2. Anote a exata concentrao de EDTA na soluo padro.
3. Usando uma pipeta de 25 mL, transfira uma alquota 25,0 mL da soluo
desconhecida (unknown solution) para um Erlenmeyer 250 mL. Complete o volume para aproximadamente 100 mL adicionando gua destilada.
4. Usando uma pipeta de 1 mL, adicione 1,0 mL do tampo (buffer) de pH 10.
5. Adicione 3-4 gotas da soluo de indicador EBT.

Ano Internacional de Qumica



| 107

Programa Nacional
Olimpadas de Qumica

43rd IChO
6. Titule a soluo desconhecida com a soluo padro de EDTA at a cor
mudar de vermelho para azul.
7. Registre o volume de soluo de EDTA usada, em mL.
8. Use o mesmo Erlenmeyer quando repetir a titulao. Antes disso, despeje
o contedo do Erlenmeyer no frasco rotulado com Aqueous Waste container e lave-o duas vezes com gua destilada.
Tratamento de dados
1. Determine a quantidade ons Cl- em milimols, em 100 mL de soluo desconhecida (unknown solution).
2. Determine a quantidade ons Mg2+ em milimols, em 100 mL de soluo
desconhecida (unknown solution).
3. Calcule a concentrao de MgCl2 e NaCl na soluo desconhecida (unknown solution), em g/100 mL.

108 |

Ano Internacional de Qumica


Olimpada Brasileira
de Qumica 2011

Exame Prtico
PROBLEMA 2
Gerao de hidrognio a partir de amnia-borano
Hidrognio considerado como uma fonte de energia limpa e pouco agressiva ao meio ambiente e caminha para se tornar a energia sustentvel do
futuro. Uma estocagem segura e efetiva de hidrognio a chave para a
economia de hidrognio. Dentre os hidretos qumicos, considerados como
um potente material para estocagem de hidrognio slido, a amnia-borano
(H3NBH3) tem despertado uma grande ateno, devido ao seu alto contedo de hidrognio e estabilidade nas condies de operao de clulas combustveis. Sob hidrlise, amnia-borano pode liberar hidrognio, Equao 1:
H3NBH3(aq) + 2H2O(l)

NH4BO2(aq) + 3H2(g)

(1)

Solues aquosas de amnia borano so estveis e sua hidrlise ocorre somente na presena de um catalisador adequado. Recentes estudos mostraram
que nanoclusters de paldio(0) estabilizados por polmeros solveis em gua
so catalisadores altamente ativos na hidrlise de amnia-borano. Nanoclusters de paldio(0) so gerados in situ pela reduo de tetracloropaladato(II)
de potssio com amnia-borano na presena de copolmero de cido 4-estirenossulfnico e cido maleico.
Neste experimento, a hidrlise cataltica de amnia-borano ser realizada
com tetracloropaladato(II) de potssio em uma soluo contendo copolmero de cido 4-estirenossulfnico e cido maleico. O tetracloropaladato(II) de
potssio ser usado como pr-catalizador, o qual ser reduzido por amniaborano formando nanoclusters de paldio(0), estabilizados pelo copolmero
de cido 4-estirenossulfnico e cido maleico. Estes nanoclusters catalisaro
a hidrlise de amnia-borano.
I. Verificao do Sistema Experimental
1. Verifique se o sistema experimental, mostrado abaixo, est fixo no suporte,
se o tubo graduado est conectado ao tubo de Schlenk pela mangueira de
plstico e se a barra magntica est dentro do tudo de Schlenk.

Ano Internacional de Qumica



| 109

Programa Nacional
Olimpadas de Qumica

43rd IChO
2. O septo deve estar fora e a vlvula deve estar aberta.
3. Acerte a altura do bulbo para ajustar em zero o nvel da gua no tubo
graduado.
4. Feche a vlvula no tubo de Schlenk.

II. Hidrlise de amnia-borano


A. Na ausncia de catalisador
1. Usando o funil, transfira todo o contedo da soluo de amnia-borano
contida no frasco (Solution-A) para o tubo de Schlenk.
2. Adicione a soluo de polmero contida no frasco (Solution-B) para o
tubo de Schlenk atravs do funil.
3. Feche o tubo de Schlenk com o septo, ajuste a agitao em 600 rpm (como
marcado no agitador), e abra a vlvula conectada ao tubo graduado. Registre o nvel da gua no tempo zero como Vo. Acione o cronmetro.
4. A cada minuto, leia o volume total de gs produzido e escreva na tabela
dada na folha de respostas. Faa isto por 10 minutos. Pare o cronmetro.
110 |

Ano Internacional de Qumica


Olimpada Brasileira
de Qumica 2011

Exame Prtico
B. Na presena de catalisador
1. Enquanto agita, transfira todo o contedo da soluo de tetracloropaladato(II)
de potssio contida no frasco (Solution-C) para o tubo de Schlenk, injetando atravs do septo com a seringa de 2.0 mL. Mantenha a seringa inserida no septo durante todo o experimento. Acione o cronmetro.
2. A cada minuto, leia o volume total de gs produzido e escreva o valor na
tabela dada na folha de respostas. Faa isto por 10 minutos. Pare o cronmetro.
Tratamento de dados
A. Reao de amnia-borano sem catalisador
1. Plote o volume de gs registrado versus tempo no Grfico 1.
2. Anote o volume de gas produzido como Vsem-catalisador.
B. Reao de amnia-borano com catalisador
1. Plote o volume de gs registrado versus tempo no Grfico 2.
2. Calcule o nmero mximo de mols e o volume mximo (mL) de gs hidrognio que seria produzido teoricamente a partir da hidrlise de 29,5 mg
amnia-borane com pureza de 97% m/m a 25 C. A presso atmosfrica
de 690 torr.
3. Calcule a velocidade de gerao de hidrognio em seu experimento
i) em mL H2/ min.
ii) em mmol H2/ min, assumindo que a temperatura de 25 C. A presso
atmosfrica de 690 torr.
4. Calcule a velocidade de produo de hidrognio por mol de paldio em (mol H2)(mol Pd)-1(min)-1 em seu experimento. A pureza do
tetracloropaladato(II) de potssio 98% m/m.

Ano Internacional de Qumica



| 111

Programa Nacional
Olimpadas de Qumica

43rd IChO
PROBLEMA 3
Sntese, purificao e separao de uma mistura diastereomrica
A natureza tem muitos compostos enantiomrica e diastereomericamente
puros, como por exemplo acares, aminocidos, esterides, etc. Alguns destes compostos so biologicamente ativos e usados em medicamentos. Consequentemente a sntese orgnica assimtrica muito importante. Um dos
mtodos usados na sntese assimtrica de compostos orgnicos utiliza catalisadores que contm metais, nos quais, o metal est coordenado com uma
molcula orgnica quiral, designada por ligante quiral. Nesta experincia
dois ligantes quirais sero sintetizados.

A. Sntese
1. Com o auxlio da seringa e atravs do septo transfira a soluo de trietilamina contida em V1, para o balo de fundo redondo de 10 mL (Rxn RB)
que contm 0,50 mmol 2,3-dibromo-1-ferrocenilpropan-1-ona.
2. Agite a mistura, colocando a barra magntica a 600 rpm (marcada na placa de agitao), temperatura ambiente durante 30 minutos.
3. No final dos 30 minutos, transfira atravs do septo, a soluo de (R)-1feniletanamina contida em V2 para o balo da reao, usando a mesma
seringa.
4. Agite a mistura, temperatura ambiente, durante mais 60 minutos.
5. No final dos 60 minutos desligue a agitao e efetue a anlise por cromatografia em camada delgada, TLC. Efetue a anlise da seguinte forma:
i) Verifique o estado das placas de TLC antes de us-las. As placas estragadas sero substitudas sem punio.
ii) Desenhe, com o lpis, a linha de aplicao na parte inferior da placa de
TLC (Fig. 2.1).
112 |

Ano Internacional de Qumica


Olimpada Brasileira
de Qumica 2011

Exame Prtico
iii) Aplique o material de partida contido no frasco etiquetado como SM,
duas vezes no lado esquerdo da linha de aplicao e duas vezes no meio
da linha. Na mesma placa aplique, uma vez na parte direita da linha de
aplicao e outra no meio da linha, a mistura reacional (RM) retirada do
balo da reao. Faa como mostrado na Figura 2.1 e usando diferentes
capilares para cada amostra.
iv) Elua a placa de TLC na cmara de eluio, usando o eluente fornecido.
Marque com o lpis a linha do front do solvente.
v) Quando a placa de TLC estiver seca, coloque-a no saco de plstico com
zipper, etiquetado TLC1. .

Figure 2.1. Placa de TLC

Fig. 2.2 Placa de TLC colocada na cuba de eluio.

Ano Internacional de Qumica



| 113

Programa Nacional
Olimpadas de Qumica

43rd IChO
B. Coluna de Cromatografia
1. Retire a rolha, abra a torneira e deixe o eluente correr at atingir o topo da
slica gel.
2. Feche a torneira e usando uma pipeta de Pasteur aplique a mistura reacional (Fig. 2.3).

Figura 2.3. Coluna Flash de cromatografia

3. Lave o balo da reao com 0,5 mL de eluente retirados do frasco rotulado


ELUENT. Utilize uma seringa limpa para isso. Utilize a mesma pipeta de
Pasteur para aplicar esta soluo de lavagem no topo da coluna.
4. Abra a torneira da coluna e deixe o solvente correr at atingir o topo da
slica gel.
5. Feche a torneira e adicione com uma pipeta de Pasteur 1,0 mL de eluente.
Abra a torneira. Quando o eluente estiver no topo da slica, adicione lentamente 2 a 3 mL de eluente sem fechar a torneira.
114 |

Ano Internacional de Qumica


Olimpada Brasileira
de Qumica 2011

Exame Prtico
6. Encha a coluna com mais eluente. ATENO: Seja cuidadoso ao adicionar o eluente de forma a no perturbar o topo da coluna de slica
gel.
7. De forma a acelerar a purificao, aplique um pouco de presso no topo
da coluna, usando para o efeito o adaptador com a pera. ATENO: Seja
cuidadoso ao aplicar presso. Adicione eluente de tempos em tempos para evitar que a slica seque.
8. Espera-se que duas fraes majoritrias sejam recolhidas: as fraes A e B.
Coloque no recipiente dos resduos etiquetados Organic Waste, o que
for eluido antes da frao A e entre as fraes A e B.
9. Recolha a primeira frao majoritria num Erlenmeyer de 100 mL e identifique-a como frao A.
10. Recolha a segunda frao majoritria num Erlenmeyer de 250 mL e identifique-a como frao B.
11. Depois de recolher a frao B, termine a eluio fechando a torneira.
C. Anlise
1. Use a segunda placa de TLC aplicando o material de partida (SM) duas vezes no lado esquerdo da linha de aplicao, a frao A duas vezes no meio
da linha e a frao B, cinco vezes no lado direito da linha. Aps a eluio
e a secagem da placa de TLC coloque-a no saco de plstico com zipper,
etiquetado com TLC2.
2. Mea o volume da frao A recolhido usando a proveta de 50 mL e escreva
esse volume na folha de respostas.
3. Mea o volume da frao B recolhido usando a proveta de 250 mL e escreva esse volume na folha de respostas.
4. Usando a pipeta de 2 mL transfira 2,0 mL da frao A para o balo volumtrico de 10 mL e complete o volume at 10 mL por adio de eluente.
Agite a soluo preparada e encha usando uma pipeta de Pasteur encha
uma cubeta de UV-visvel (pelo menos do seu volume total). Pea ao
supervisor que determine a absorbncia,a 450 nm, usando um espectrofotmetro e escreva o valor obtido na folha de respostas.
5. Com a frao B (no necessria diluio), encha, usando uma pipeta de
Pasteur, a outra cubeta de UV-visvel (pelo menos do seu volume total).
Pea ao supervisor que determine a absorbncia, a 450, nm usando um
espectrofotmetro e escreva o valor obtido na folha de respostas.

Ano Internacional de Qumica



| 115

Programa Nacional
Olimpadas de Qumica

43rd IChO
Tratamento dos dados
1. Desenhe o resultado da placa TLC1 na folha de respostas.
2. Desenhe o resultado da placa TLC2 na folha de respostas.
3. Calcule e escreva os valores de Rf das manchas (frao A, frao B e material de partida SM) usando a placa de TLC2.
4. O coeficiente de extino molar, e, a 450 nm : 404 L.mol-1cm-1 para A e
400 L.mol-1cm-1 para B. Calcule:
i) Tendo em conta a quantidade de material de partida calcule o rendimento da frao A.
ii) Tendo em conta a quantidade de material de partida calcule o rendimento da frao B.

As coisas que realizamos, nunca so to belas


quanto s que sonhamos.
Mas s vezes, nos acontecem coisas to belas,
que nunca pensamos em sonh-las.
(desconhecido)
116 |

Ano Internacional de Qumica


Olimpada Brasileira
de Qumica 2011

Exame Terico

43rd International
Chemistry Olympiad
Problemas Tericos
14 de Julho de 2011
Ankara, Turquia

PROBLEMA 1

Problema 1

7,0 % do total

Problema 1
a

i
6

d
ii
1,5

iii
1

Problema 1

2,5

22

x%

7,0 % do total
7,0

i
6

d
ii
1,5

iii
1

e
2,5

Problema 1

x%

22

7,0

Os xidos deOs
nitrognio,
comuns do ar
atmosfrico comuns
so formados
por
xidospoluentes
de nitrognio,
poluentes
doprincipalmente
ar atmosfrico
so formados
O xido ntrico
essencialmente
xido ntrico,principalmente
NO, e dixido de nitrognio,
NO2. ntrico,
por xido
NO, atmosfrico
e dixido de
nitrognio, NO2. O xido
formado durante as tempestades e nos motores de combusto interna. Em altas temperaturas o
ntrico atmosfrico essencialmente formado durante as tempestades e nos
NO reage com H2 e produz xido nitroso, N2O, um gs de efeito estufa.

motores
de combusto
Emcomuns
altas temperaturas
o NO
com
H2 e
2 NO(g)
+ H2(g)interna.
N
Os xidos
de nitrognio,
poluentes
do ar atmosfrico
so reage
formados
principalmente
p
2O(g) + H2O(g)
produz
xido
nitroso,
O,
um
gs
de
efeito
estufa.
.
O
xido
ntrico
atmosfrico

essencialmen
xido
ntrico,
NO, eNdixido
de
nitrognio,
NO
2
2

No estudo da cintica desta reao a 820 C, as velocidades iniciais para a formao de N2O
durante
as tempestades
foram medidas
H2. motores de combusto interna. Em altas temperaturas
2 usando
NOformado
(g)vrias
+ Hpresses
(g) parciais
N iniciais
O(g)+deHNOeOenos
(g)
2

NO reage com H2 e produz xido nitroso, N2O, um gs de efeito estufa.

Presso inicial, torr


Velocidade inicial
produo
a de
820
asvelocidades
iniciais para a forma2 NO(g)
+ C,
H2(g)
N2O(g) + H2O(g)
Exp.No estudo da cintica desta reao
de N2O, torrs-1

PNO
1
2
3

H2 medidas usando vrias presses parciais iniciais de NO e H .


o de N2O foram
2
-2

120,0
60,0cintica desta reao
8,6610 a 820 C, as velocidades iniciais para a formao de N
No
estudo da
-2
60,0 medidas
60,0usando vrias 2,1710
foram
presses
parciais iniciais de NO e H2.
60,0

180,0

Exp.

6,6210-2

Presso inicial, torr

Velocidade inicial de produo


-1

de eNtempo
Neste problema no use concentraes. Use
presso em torr
em
2O, torrs
PNO unidades de
H2
segundos.
-2
1
120,0
60,0
8,6610
a. Determine a lei de velocidade experimental e calcule a constante de velocidade.
b.

-2

60,0 2 torr de NO e 1,0010


2,1710
2
torr H2
Calcule a velocidade inicial 2
de consumo 60,0
de NO, se 2,0010
-2 7
forem misturados a 820 C. (Se
tiver o valor da180,0
constante de velocidade,
use 210
3 voc no60,0
6,6210
com a unidade apropriada.)

Neste problema no use concentraes. Use unidades de presso em torr e tempo e

Calcule o temposegundos.
gasto para reduzir a presso parcial de H2 a metade do seu valor, se
NO eDetermine
1,0 torr de H
forem
misturados a experimental
820 C. (Se voc
no tiveraoconstante
valor da de velocidade.
8,00102 torr de a.
a2lei
de velocidade
e calcule
Ano Internacional de Qumica

| 117 2
constante de velocidade, use 2107 com a unidade apropriada.)
2
c.

b.

Calcule a velocidade inicial de consumo de NO, se 2,0010 torr de NO e 1,0010 torr


forem misturados a 820 C. (Se voc no tiver o valor da constante de velocidade, use 21

Programa Nacional
Olimpadas de Qumica

43rd IChO
Neste problema no use concentraes. Use unidades de presso em
torr e tempo em segundos.
a. Determine a lei de velocidade experimental e calcule a constante de velocidade.
b. Calcule a velocidade inicial de consumo de NO, se 2,00102 torr de NO e
1,00102 torr H2 forem misturados a 820 C. (Se voc no tiver o valor da
constante de velocidade, use 210-7 com a unidade apropriada.)
c. Calcule o tempo gasto para reduzir a presso parcial de H2 a metade do
seu valor, 8,00102 torr de NO e 1,0 torr de H2 forem misturados a 820 C.
(Se voc no tiver o valor da constante de velocidade, use 210-7 com a
unidade apropriada.)

d. Um
mecanismo
para aNO
reao
NOabaixo:
e H2 dado abaixo:
ecanismo
proposto
para aproposto
reao entre
e H2entre
dado

2 NO(g)

k1

N2 O2(g)

k-1

k2

N2O2(g) + H2(g) N2O(g) + H2O(g)

duza a i.
lei Deduza
de velocidade
para a formao
de N2Odea Npartir
do mecanismo
a lei de velocidade
para a formao
O a partir
do meca- proposto
2
nismo
proposto
usando
a
aproximao
do
estado
estacionrio
para o
ando a aproximao do estado estacionrio para o intermedirio.
intermedirio.

ii. Sob que condies esta lei de velocidadede se reduz lei obtida expee condies esta lei de velocidadede se reduz lei obtida experimentalmente na Parte a?
rimentalmente na Parte a?

<< k2PH2Se k << k P_( )


-1
2
H2

Se k-1 >> k2P_(H_2 )

>> k2PH2

> k2

Se k-1 > k2

Se k1 > k-1

> k-1

iii. Expresse a constante de velocidade determinada experimentalmente k


em termos de k1, k-1 e k2.
presse a constante de velocidade determinada experimentalmente k em termos de k1
118 |

Ano Internacional de Qumica


nale no quadro a letra correspondente ao diagrama de energia que consistente com o

Se k-1 > k2
Se k1 > k-1

Olimpada Brasileira
de Qumica 2011

iii. Expresse a constante de velocidade determinada experimentalmente k em termos de k1,


k1 e k2.

Exame Terico

e. e. Assinale
Assinale no no
quadro
a letra
correspondente
ao diagrama
de energia de
queenergia
consistente
quadro
a letra
correspondente
ao diagrama
que com o
mecanismo
de reao
e comde
a lei
de velocidade
experimental.
consistente
com o proposto
mecanismo
reao
proposto
e com a lei de velocida-

de experimental.

a.

b.

energy

energy

energy

e.

reaction coordinate

a)

f.

energy

energy

b)

reaction coordinate

reaction coordinate

reaction coordinate

d.

c.

energy

reaction coordinate

c)

d)

reaction coordinate

e)

f)

Ano Internacional de Qumica



| 119

Programa Nacional
Olimpadas de Qumica

43rd IChO

Problema 2

7,0 % do total

PROBLEMA 2

ma 2

a
7,0 % do total
6

a
6

b
ii

iii

b
ii

iii

Problema 2

x%

23

7,0

Problema 2
23

Amnia anidra um combustvel lquido alternativo muito limpo e de alta densidade de ene
Ela no libera nenhum gs estufa na sua combusto.

Amnia
anidra umNH
combustvel
lquido alternativo muito limpo e de alta
Em um experimento,
3 gasosa queimada com O2 em um recipiente de volume fixo de ac
densidade
de
energia.
Ela
no
libera
nenhum
gsdeestufa
na sua combusto.
mbustvel
lquido
alternativo
muito limpo e de alta
densidade
energia.
com
a equao
abaixo.

s estufa
suaexperimento,
combusto. NH gasosa queimada com O em um recipiente de
Emnaum
3
2

NH
3 abaixo.
O
N2(g)
+ 6 H2O(l)
3(g) + de
2(g)
queimada
com
O2 emcom
um4 recipiente
volume
fixo2 de
acordo
H3 gasosa
volume
fixo de
acordo
a equao

Os estados iniciais
e 3finais
298 K. Aps
a combusto
com 14,40 g de O2, uma part
4 NH
(g) +esto
3 O2a(g)
2 N2(g)
+ 6 H2O(l)

4 NHNH
+ 3Oconsumida.
3(g)
2(g) 2 N2(g) + 6 H2O(l)
3 no

Os estados iniciais e finais esto a 298 K. Aps a combusto com 14,40 g de


O2,auma
parte
dea NH
no com
consumida.
nais esto
298 K.
Aps
combusto
14,40 g de O2, uma parte de
3
a. Calcule o calor liberado no processo.
a. Calcule o calor liberado no processo.
Dados: fH(NH3(g)) = -46.11 kJmol-1 e fH(H2O(l)) = -285.83 kJmol-1

ado no processo.

b. Para determinar a quantidade de gs NH3 dissolvida na gua produzida

-1
)) = -46.11durante
kJmol-1 o
e processo
fH(H2O(l))
-285.83 kJmoluma
de= combusto,
amostra de 10,00 mL da soluo

produzida
durante o proc
b. aquosa
Para determinar
a quantidade
de gs
NH3 dissolvida
foi retirada
do frasco onde
ocorreu
a reaonae gua
adicionada
a 15,0
de
combusto,
uma
amostra
de
10,00
mL
da
soluo
aquosa
foi
retirada
mL de uma soluo 0,0100 M de H2SO4. A soluo resultante foi titulada do frasco
uma
padronizada
0,0200
M de
e o ponto
de equivaocorreu
asoluo
e adicionada
a 15,0 mL
deNaOH
uma
soluo
0,0100
M de H2SO4. A sol
dissolvida
na gua produzida
durante
o processo
quantidadecom
de
gs
NH
3 reao
foi atingido
em 10,64
mL.
(NH
) = padronizada
1,8
x 10-5; onde
Ka(HSO
) = 1,1 x 10-2) e o pont
a amostra lncia
de
10,00
mLfoidatitulada
soluo
aquosa
foibsoluo
retirada
do frasco
resultante
com
uma(K
0,0200
3
4- M de NaOH

.=A
soluo
adicionada
a 15,0 mL
defoida
uma
soluo
0,0100
M de
2SO
(NH
1,8
10-5; Ka(HSO4-) = 1,1 10-2)
equivalncia
atingido
emno
10,64
mL.
(KbH
3)4a
i. Calcule
o pH
soluo
recipiente
aps
combusto.
da com uma soluo padronizada 0,0200 M de NaOH e o ponto de

ii. No ponto final da titulao, ons NH + e SO 2- esto presentes na soluo. Escreva as equaes para os equilbrios relevantes que mostrem
comoaps
a presena
destes dois ons afeta o pH e calcule as suas constantes
uo no recipiente
combusto.
ii. de
Noequilbrio.
ponto afinal
da titulao, ons NH4+ e SO42- esto presentes na soluo. Escrev

i. Calcule
pH da soluo-5no recipiente
aps
a-2combusto.
4
4
gido em 10,64
mL. (Ko
b(NH3) = 1,8 10 ; Ka(HSO4 ) = 1,1 10 )

equaes para os equilbrios relevantes que mostrem como a presena destes dois

2- esto que
Assinale
a alternativa
corretamente
o pH as
da soluo no ponNHo4+pH
e
na
soluo.
Escreva
titulao,iii.
ons
4
afeta
eSOcalcule
as presentes
suasindica
constantes
de equilbrio.
to de equivalncia.
s equilbrios relevantes
que mostrem como a presena destes dois ons
le as suas constantes de equilbrio.

pHa>alternativa
7,0
=7,0
pH <7,0
iii. Assinale
que indicapH
corretamente
o pH da soluo
no ponto de equivalnci

tiva que indica corretamente o pH da soluo no ponto de equivalncia. Ano Internacional de Qumica

120 |

Olimpada Brasileira
de Qumica 2011

Exame Terico
PROBLEMA 3

Problema
38,0 % do total
8,0 % do total
lema
3

Problema 3
a

c
b

d
c7

ii

i 5

8,0
% cdo total
Problema 3
d
b
i

Problema 3

ii

23

ii

x%
5

Problema 3

x%

23

8,0

8,0

23

x%

8,0

A zero Kelvin, a energia total de uma molcula diatmica gasosa AB dada aproximadame
Apor:
zero
Kelvin,
a energia
total gasosa
de umaAB
molcula
diatmica gasosa AB dada
a energia total
de uma
molcula
diatmica
dada aproximadamente

A zero Kelvin,aproximadamente
a energia total de uma
por:molcula diatmica gasosa AB dada aproximadamente
+
E
E
=
E
o
vib
por:

E = Eo + Evib

E = Eo + Evib

onde Eo a energia eletrnica do estado fundamental, e Evib a energia vibracional.

onde Eo a energia eletrnica do estado fundamental, e Evib a energia

ergia eletrnica
do eletrnica
estado
fundamental,
e Evib
a energia
vibracional.
vibracional.
Valores
permitidos
de energias
vibracionais
dados pela
expresso:
do estado
fundamental,
e E so
a energia
vibracional.
onde E a energia
o

vib

Valores
permitidos
de energias
vibracionais
so dados
dos de permitidos
energias
vibracionais
so dados
pela
expresso:
Valores
de energias
vibracionais
so dados
pela expresso:

pela expresso:

mA mB

(AB) =
v = 0, 1, 2,

=
mA + mB
2
2A mB
mA mB
m
1
(AB)

(AB)
=
=
Evib = (v + v =
) 0, 1, 2,v = 0, 1, 2,
= = 2
mA + mBmA + mB
2
2
Onde
h

a
constante
de
Planck,
v

nmero
quntico vibracional, k a consOnde h a constante de Planck, v nmero quntico vibracional, k a constante de fora, e
Evib = (v +

tante de fora, e m a massa reduzida da molcula. A zero Kelvin, pode ser

massa reduzida
davmolcula.
A
Kelvin,
pode
assumido
Onde h dea Planck,
constante
Planck,
nmero
quntico
vibracional,
k a seguramente
constante
e substituique
a v zero, e E
seguramente
assumido
que
vzero
zero,
e k ser
so
independentes
nstante
v de
nmero
quntico
vibracional,
keaEconstante
de fora,de
e fora,
ade
o
k so
independentes
de
substituio
na molcula.
o
nazero
molcula.
massa reduzida
daisotpica
molcula.
A
Kelvin,
pode serisotpica
seguramente
assumido que v zero, e Eo e
a da molcula. A zero Kelvin, pode ser seguramente assumido que v zero, e Eo e

k so independentes
de substituio
isotpica
na molcula.
-1
a. Calcule
a avariao
de entalpia,
entalpia,
D em
H, em
kJmol-1
a seguinte
H,
kJmol
para a para
seguinte
reao a reao
0 K.
a. Calcule
variao
de
entes de substituio
isotpica
na molcula.

0 K.

Calcule a variao de entalpia, H, em kJmol-1 para a seguinte reao a 0 K.


+ D2(g)

HD(g)
H2(g)
a seguinte
reao
a2 0HD
K.2(g)
ariao de entalpia, H, em kJmol-1 para
H (g)
+ D2(g)

a.

H2(g) + D2(g) 2 HD(g)


Deutrio,
D, +um
um
istopo
dodo
hidrognio
com nmero
de massa
Para a2.
molcula
Deutrio,
istopo
hidrognio
com nmero
de2.massa
Para a de H2 ,
(g)
D2(g)
2 HD(g)
H2D,
-1
2,0141
Deutrio, D, molcula
um
istopo
com nmero
2. de
Para
de H2 , kand
e, k
asmassas
molares
H aemolcula
D so
1,0078
575,11
Nm
dedoH,hidrognio
575,11
Nm-1,deisotpicas
e massa
as massas
molares
isotpicas
de
H e gm
2
-1do hidrognio com nmero de massa
-1
,
k

D, 575,11
um istopo
2.
Para
a
molcula
de
H
-1
2
D
so
1,0078
and
2,0141
gmol
,
respectivamente.
Dados:
Nm , e as massas molares isotpicas de H e D so 1,0078 and 2,0141 gmol ,
= 1,1546 HD e D = 0,8167 HD
a zero Kelvin.
respectivamente. Dados:
2
and 2,0141
gmol-1,
m-1, e as massas molares isotpicas de HHe2 D so 1,0078
zero Kelvin.
Kelvin.
respectivamente. Dados: H = 1,1546 HD e D = 0,8167 HD a azero
2
2

mente. Dados:

b.

H2 = 1,1546 HD

D2 = 0,8167 HD

Calcule a frequncia, em

s-1,

a zero Kelvin.

de ftons infravemelhos que podem se absorvidos p

Internacional de
Qumica
-20
b. Calcule aAnofrequncia,
s-1, (Se
de voc
ftonsno
infravemelhos
que podem
se absorvidos pela

| 121
molculaem
de HD.
tem o valor para
HD ento use 8,00010 J para o clcul
-20
molcula de
voc
no tem
o valor para que
use 8,00010
J parapela
o clculo.)
frequncia,
emHD.
s-1(Se
, de
ftons
infravemelhos
podem
se absorvidos
HD ento

Deutrio, D, um istopo do hidrognio com nmero de massa 2. Para a molcula d


-1

575,11Nacional
Nm , e
Programa
Olimpadas de Qumica

as massas molares isotpicas de H e D so 1,0078 and 2,014

respectivamente. Dados:

43rd IChO

H2 = 1,1546 HD

D2 = 0,8167 HD

a zero Kelv

b. Calcule a frequncia, em s-1, de ftons infravemelhos que podem se abmolcula de


voc
no tem
o valor para que
eHDento
use se absorv
b. sorvidos
Calcule pela
a frequncia,
emHD.
s-1(Se
, de
ftons
infravemelhos
podem

20
8,000 10- J para o clculo.)
molcula de HD. (Se voc no tem o valor para HD ento use 8,00010-20 J para o
c. As energias eletrnicas permitidas para o tomo de H so dadas pela exc. presso
As energias eletrnicas permitidas para o tomo de H so dadas pela expresso

E=

RH
, n = 1, 2,
n2

onde RH = 13,5984 eV, 1 eV = 1,60210-19 J

estadofundamental
fundamental -31,675
-31,675 eV, em
A energia
total
molculadedeH2Hno
i.i. A energia
total
dada
molcula
estado
2 no
eV,mesma
em relao
a mesma
referncia
do tomo Calcule
de hidrognio.
Calcule
a
referncia
do tomo
de hidrognio.
a energia
de dissociao,
e
energia de dissociao, em eV, da molcula de hidrognio no estado
fundamental de forma que os tomos de H sejam produzidos nos seus
estados fundamentais.

ii. Uma molcula de H2 no estado fundamental dissocia em seus tomos aps a absoro de ftons no comprimento de onda de 77,0 nm.
Determine todas as possibilidades para os estados eletrnicos dos tomos de H produzidos. Em cada caso, qual a energia cintica, em eV,
dos tomos de hidrognio dissociados?
d. Calcule a afinidade eletrnica do on H2+, em eV, sabendo que sua energia
de dissociao de 2,650 eV. (Se voc no tem o valor da energia de dissociao do H2 ento use 4,500 eV para o clculo.)

122 |

Ano Internacional de Qumica


Olimpada Brasileira
de Qumica 2011

Exame Terico
PROBLEMA 4

9,0% do total
a

Problema 4

x%

30

9,0

Para a energia sustentvel, o hidrognio torna-se a melhor opo. O mais


eficiente caminho para o uso de hidrognio a gerao de energia eltrica
em clulas combustveis. Entretanto, estocar hidrognio em grandes quantidades um desafio na aplicao de clulas combustveis. Dentre os hidretos
qumicos, considerados como um material para estocagem de hidrognio
slido, o boro-hidreto de sdio (NaBH4), sendo no-txico, estvel e noagressivo ao meio ambiente, aparece como o mais promissor. A hidrlise
de boro-hidreto de sdio, que libera H2, uma reao lenta temperatura
ambiente e, assim, precisa ser catalisada.

Nanoclusters de rutnio(0) so os catalisadores mais ativos nestas hidrlises, temperatura ambiente, e leva completa liberao de H2 a partir do
boro-hidreto de sdio. Estudos cinticos mostram que a hidrlise cataltica
de NaBH4 de primeira ordem em relao ao catalisador, porm de ordem
zero em relao ao substrato. A velocidade de produo de hidrognio por
mol de rutnio de 92 mol H2-(mol Ru)-1-min-1 a 25 0C.
a. Calcule a quantidade do catalisador de rutnio (em mg) que deve ser adicionado a 0,100 L de uma soluo de NaBH4 1,0 mol-L-1 para produzir o
hidrognio gasoso, necessrio para uma clula combustvel porttil, a uma
velocidade de 0,100 L-min-1 a 25 C e 1,0 atm.
b. Durante quantos minutos este sistema produzir hidrognio gasoso a esta
velocidade?
c. A energia de ativao de Arrhenius para esta hidrlise cataltica de borohidreto de sdio de Ea = 42,0 kJ-mol-1. Calcule a temperatura necessria para obter a mesma velocidade de produo de hidrognio, usando a
metade da quantidade de catalisador de rutnio usada anteriormente a
25,0 C.

Ano Internacional de Qumica



| 123

d.

A clula combustvel constituda de trs


d. A clula combustvel constituda de trs
segmentos juntos (na forma de sanduiche): o
segmentos juntos (na forma de sanduiche): o
Programa Nacional
anodo,
o
eletrlito,
e
o catodo. O hidrognio
Olimpadas de Qumica
anodo, o eletrlito, e o catodo. O hidrognio
usado
como
combustvel
e oxignio como
rd
usado como combustvel e oxignio como
43
IChO
oxidante. Nas interfaces dos trs diferentes
e
anodo
ea
oxidante. Nas interfaces dos trs
diferentes
+
segmentos ocorrem duas reaes qumicas.
electrlito
H
ocorrem
duas reaes
d. A clula combustvel segmentos
constituda de
trs segmentos
juntosqumicas.
(na forma de
- electr
-o eletrlito,
catodo
sanduiche):
o
anodo,
e
o
catodo.
O
hidrognio

usado
como
e
O2(g) + 2H2O(l) + 4e 4OH (aq)
cat
2H2O(l) +Nas
4e-interfaces
4OH-(aq)
2(g) +oxidante.
combustvel e oxignioOcomo
dos trs diferentes
ocorrem
duas
reaes
(aq) 2H
+ 2e-qumicas.
Hsegmentos
2(g) + 2OH
2O(l)
H2(g) + 2OH-(aq) 2H2O(l) + 2eOO
(g) + 2H2O(l)liquido
+ 4e- das duas
4OH-(aq)
2 resultado
rees :
H2O
O
resultado
liquido das duas rees :
H2(g) + 2OH (aq)
2H2O(l) + 2e-
2 H2(g) + O2(g) 2 H2O(l)
(g) + O:2(g) 2 H2O(l)
2 H2rees
O resultado liquido das duas
O hidrognio para a clula combustvel produzido a partir da hidrlise do b
2 H2(g) + O2(g)
2 H2O(l)hidrognio para a clula combustvel produzido a p
sdio.
sdio.
o potencial
padro
para a semi-reao
do catodo
se o potencial
padr
OCalcule
hidrognio
para a clula
combustvel
produzido
hidrlise
do do catod
Calcule
o potencial
padroa partir
para adasemi-reao
-1
para a semi-reao
boro-hidreto
de sdio. do anodo de 0,83 V e fG (H2O(l)) de -237 kJmol .
para a semi-reao do anodo de 0,83 V e fG (H2O(l))
o potencial
semi-reao
catodo se o
potencial
e. Calcule
Calcule
o volumepadro
de ar para
a 25aC
e 1,0 atmdo
necessrio
para
gerar pauma corrente
Calcule o volume
deardea-0,83
25 CDefG1,0
pa
dro de reduo parae.a semi-reao
do ano do
(H2atm
O(l) necessrio
de
2,5 kJ-mol
A por-13,0
h nesta clula combustvel. Considere que o ar contm 20%
-237
.
2,5 A por 3,0 h nesta clula combustvel. Considere q
volume.
e. Calcule o volume de arvolume.
a 25 C e 1,0 atm necessrio para gerar uma corrente constante de 2,5 A por 3,0 h nesta clula combustvel. Considere que
o ar contm 20% de O2(g) por volume.
f. A eficincia de uma clula combustvel dada pela razo entre o trabalho produ
A eficincia
de uma
clula
combustvel
dada pela razo
f. A eficincia de umaf. clula
combustvel
dada
pela razo
entre o
trabalho
dissipado
pela
reao
da
clula.
Assim,
a
eficincia
mxima
para
a clula comb
produzido e o calor dissipado
pelapela
reao
da clula.
Assim,Assim,
a eficincia
dissipado
reao
da clula.
a eficincia mxim
por:
mxima para a clula combustvel dada por:
por:
clula combustvel =

trabalho

calorclula combustvel =

trabalho

calor
eficincia mxima
mximapara
paraa aclula
clulacombustvel
combustvel
usando
dados fornec
f. Calcule
Calcule a eficincia
usando
os os
dados
Calcule a eficincia mxima para a clula combustvel us
fornecidos
abaixo,
a
25
C
e
presso
padro.
25 C e presso padro.

25 C e presso padro.

S (Jmol-1K-1)
H2(g)

130,7

O2(g)

205,2

H2O(l)

70,0

124 |

g.

S (Jmol-1K-1)

H2(g)

130,7

O2(g)

205,2

H2O(l)

70,0

Ano Internacional de Qumica


A segunda lei da termodinmica diz que impossvel converter todo o cal

Olimpada Brasileira
de Qumica 2011

Exame Terico
g. A segunda lei da termodinmica diz que impossvel converter todo o
calor, qH, de um reservatrio a alta temperatura, TH, em trabalho. Pelo
menos, parte da energia, qC, deve ser transferida para um reservatrio de
baixa temperatura, TC. Assim, uma mquina trmica (motor) com eficincia
de 100% termodinamicamente imposssvel. Quando a mquina trmica
(motor) est trabalhando reversivelmente, como em um ciclo de Carnot, a
eficincia ser mxima.
Para o motor trabalhar reversivelmente entre dois reservatrios, aplica-se a seguinte relao:

h. Qual deve ser a temperatura do reservatrio quente, TH, de uma mquina


trmica de Carnot para manter a eficincia da clula combustvel calculada
no item (f), se a temperatura do reservatrio frio TC de 40 C? (Se voc
no tem o valor para a eficincia, ento use 0,80).

Ano Internacional de Qumica



| 125

Programa Nacional
Olimpadas de Qumica

43rd IChO
PROBLEMA 5

7.0% do total
a
i

ii

Problema 5

x%

30

7.0

5
3

Compostos de Polinitrognio tm sido potencialmente usado como materiais
de alta densidade de energia. Eles so termodinamicamente instveis. Uma
grande quantidade de energia liberada de sua reao de decomposio levando produtos mais estveis. As espcies de polinitrognio conhecidas so
somente N2, N3- and N5+, isoladas em 1772, 1890 e 1999, respectivamente, e
recentemente o nion cclico, N5-.
a. (i) Escreva a estrutura de Lewis para N5+ com as tres formas de ressonncia
energeticamente favorveis. Indique os pares isolados e as cargas formais.
Desenhe a geometria molecular do N5+ cclico.
(ii) Escreva a estrutura de Lewis para o N5+ cclico com as cinco formas
de ressonncia energeticamente favorveis. Indique os pares isolados e as
cargas formais. Desenhe a geometria molecular do N5- cclico.
b. A sntese do [N5+][AsF6-], um slido inico branco, foi obtida pela reao
do [N2F+][AsF6-] com o cido hidrazico, HN3, no lquido HF a -78 C.
Escreva a equao qumica balanceada para esta reao.
A preparao do [N2F+][AsF6-] requer a reao do N2F2 com o forte cido de
Lewis , AsF5, como segue:
x C(grafite) + AsF5
Cx-AsF5
(intercalado com grafite x = 10-12)
2 Cx-AsF5 + N2F4
2 [Cx+][AsF6-] + trans-N2F2
trans-N2F2 + AsF5
[N2F+][AsF6-]
Na sntese do N2F2, formado o ismero trans, que termodinamicamente
menos estvel do que o ismero cis-N2F2. Contudo, a converso do transN2F2 para o cis-N2F2 passa por uma alta barreira energtica de 251 kJ/mol,
126 |

Ano Internacional de Qumica


Olimpada Brasileira
de Qumica 2011

Exame Terico
de forma que, o equilbrio entre os ismeros cis e trans no acontece significativamente sem o uso de um catalizador apropriado.
Quando o trans-N2F2 mantido em um recipiente fechado por 6 dias, temperatura ambiente, em presena de pequenas quantidades de SbF5 como
catalizador, o equilbrio trmico cis-trans estabelecido.

A entalpia padro de formao do trans- e cis-N2F2 so, respectivamente,


67,31 e 62,03 kJ/mol, e suas entropias padro a 25 C so respectivamente,
262,10 e 266,50 JK-1mol-1.
c. Encontre a razo do nmero de molculas do cis-N2F2 sobre o nmero de
molculas do trans-N2F2 na mistura em equilbrio, a 25 C.
d. Escreva as estruturas de Lewis mostrando a geometria do on N2F+ e dos
ismeros cis-trans do N2F2. Inclua todos os pares isolados e as cargas formais. Sugira uma hibridizao para cada tomo de nitrognio no N2F2 e
no N2F+.
O slido [N5+][AsF6-] um pouco estvel temperatura ambiente, mas reage
explosivamente com gua para produzir pentafluoreto de arsnico, fluoreto
de hidrognio, nitrognio molecular e oxignio molecular.
e. Escreva a equao balanceada para a reao entre [N5+][AsF6-] e gua.
A converso do [N5+][SbF6-] em outros sais de N5+ pode ser feita pela reao
de mettese:
[N5+][SbF6-] + [M+][X-]

[N5+][X- ] + [M+][SbF6-]

M+ = Na+, K+, Cs+; X- = um nion grande tal como SnF62- and B(CF3)4-.
Desde que o [Cs+][SbF6-] tem uma baixa solubilidade em HF anidro e [K+]
[SbF6-] tem uma baixa solubilidade em SO2, estes dois solventes foram usados
extensivamente para produzir, respectivamente, a reao de mettese a -78
C and -64 C.
f. Escreva a equao balanceada para a preparao do [N5+]2[SnF62-] e [N5+]
[B(CF3)4-] em soluo, iniciando, respectivamente, com [Cs+]2[SnF62-] e [K+]
[B(CF3)4-]. Indique o solvente apropriado.

Ano Internacional de Qumica



| 127

Programa Nacional
Olimpadas de Qumica

43rd IChO
Quando o [N5+]2[SnF62-] se decompe sob condies cuidadosamente controladas, a 25-30 C, [N5+][SnF5-] e N5F so formados. O sal [N5+][SnF5-] um
slido branco e tem uma estabilidade trmica comparvel do [N5+][SbF6-]
(50 60 C). O espectro de RMN da soluo do 119Sn tem mostrado que o
nion SnF5- neste composto , de fato, uma mistura de polinions dimricos
e tetramricos. Em ambos os polinions, o nmero de coordenao do tomo de Sn 6 e existem pontes de tomos de flor.
g. Desenhe as estruturas dos polinions dimricos e tetramtricos.

PROBLEMA 6

7,0% do total
a

Problema 6

x%

23

7,0

A extrao do ouro usando cianeto de sdio, um reagente muito txico,


causa problemas ambientais e provoca srias preocupaes nas populaes
devido ao uso deste mtodo, usualmente designado por processo cianeto.
A extrao do ouro com tiossulfato tem sido considerada uma alternativa.
Neste processo o reagente principal o tiossulfato de amnio, (NH4)2S2O3,
que no txico. Apesar deste processo ser amigo do ambiente, a qumica
envolvida complexa e necessita ser bem estudada. A soluo usada para a
complexa
e necessita
bem estudada.
para a extrao
do ouro
extrao
do ouro ser
contm
S2O32-, CuA2+soluo
, NH3, eusada
O2 dissolvido.
A soluo
devecontm
2+
2-ter um
pH
superior
a
8,5
e
amnia
livre.
S2O3 , Cu , NH3, e O2 dissolvido. A soluo deve ter um pH superior a 8,5 e amnia livre.
De acordo
o mecanismo
proposto,
uma micro-clula
De acordo
com ocom
mecanismo
proposto,
forma-se forma-se
uma micro-clula
voltaica na voltaica
superfcie das
na superfcie das partculas de ouro durante o processo de extrao que
pode ser traduzida da seguinte forma:

partculas de ouro durante o processo de extrao que pode ser traduzida da seguinte forma:
:
AnodoAnodo:
Au(s) + 2 NH3(aq) [Au(NH3)2]+(aq) + e[Au(NH3)2]+(aq) + 2 S2O32-(aq)

Catodo:

128 |

[Au(S2O3)2]3-(aq) + 2 NH3(aq)

Ano Internacional de Qumica


[Cu(NH3)4]2+(aq) + e- [Cu(NH3)2]+(aq) + 2 NH3(aq)

Au(s) + 2 NH3(aq) [Au(NH3)2]+(aq) + e+

2-

[Au(NH3)2] (aq) + 2 S2O3 (aq)

Olimpada Brasileira
de Qumica
3- 2011

[Au(S2O3)2] (aq) + 2 NH3(aq)

Exame Terico

Catodo:
Catodo:
[Cu(NH3)4]2+(aq) + e- [Cu(NH3)2]+(aq) + 2 NH3(aq)
[Cu(NH3)2]+(aq) + 3 S2O32-(aq)
a.
b.
c.

[Cu(S2O3)3]5-(aq)

+ 2 NH3(aq)

Escreva
a equao
que traduz
a reao
clula da
voltaica.
a. Escreva
a equao
que traduz
a global
reaodaglobal
clula voltaica.
5-

2+

5presena
de amnia,
o Oo2 [Cu(S
oxida2Oo3)[Cu(S
O ) ] a 2+[Cu(NH3)4]a equao
. Escreva
qumica
Nab. Na
presena
de amnia,
o O2 oxida
3] a [Cu(NH
2 3 3 3)4] . Escreva
a
equao
qumica
devidamente
balanceada
que
traduz
esta
reao
devidamente balanceada que traduz esta reao de oxidorreduo em meio bsico. de

oxidorreduo em meio bsico.

2+
Neste
processo
de extrao,
o on
complexo
[Cu(NH
como como
catalisador,
3)4]
c. Neste
processo
de extrao,
o on
complexo
[Cu(NH
)funciona
]2+ funciona
3 4
acelerando
a
dissoluo
do
ouro.
Escreva
a
equao
qumica
global
que
traduz
a
reao de
catalisador, acelerando a dissoluo do ouro. Escreva a equao qumica
oxidorreduo
da
dissoluo
do
ouro
metlico,
quando

catalisada
pelo
on
complexo
global que traduz a reao de oxidorreduo da dissoluo do ouro met2+
2+
]
[Cu(NH
3)4quando
lico,
catalisada pelo on complexo [Cu(NH ) ]

3 4

d. Escreva a geometria de coordenao do metal nos ons complexos


[Au(NH
)2]+ e [Au(S
) ]3- , e indique
tomos
coordenao.
Escreva
a 3geometria
de2Ocoordenao
do os
metal
nos de
ons
complexos [Au(NH3)2]+ e
3 2
3[Au(S
os tomos
de coordenao.
2O3)constantes
2] , e indiquede
e. As
formao,
Kf, dos ons complexos [Au(NH3)2]+ e
326
[Au(S2O3)2] so 1,0010 e 1,001028, respectivamente. Considere a so+
326
luo de extrao,
na qual
asons
concentraes
das espcies
em equilbrio
so
e. As constantes
de formao,
Kf, dos
complexos [Au(NH
3)2] e [Au(S2O3)2] so 1,0010
as
seguintes:
28
e 1,0010 , respectivamente. Considere a soluo de extrao, na qual as concentraes

d.

das espcies
equilbrio
[S2O32-] em
= 0,100
M; so
[NHas
] seguintes:
= 0,100 M; concentrao total de espcies de
3

ouro(I) = 5,5010-5 M.
Calcule a porcentagem do on ouro(I) que existe na forma de complexo
Calcule
a porcentagem
com
tiossulfato. do on ouro(I) que existe na forma de complexo com tiossulfato.

[S2O32-] = 0,100 M; [NH3] = 0,100 M; concentrao total de espcies de ouro(I) = 5,5010-5 M.

f. Quando a concentrao de O2 no suficiente e o pH superior a 10, a


espcie S2O32- reduz o on [Cu(NH3)4]2+ a [Cu(S2O3)3]5- com a formao
do
espcie S2O32- reduz o
Quando a concentrao
de O2 no
2-: suficiente e o pH superior a 10, a 22+
5on
tetrationato,
S
O
6
on [Cu(NH3)4] a [Cu(S2O3)3]4 com
a formao do on tetrationato, S4O6 :
52- 22[Cu(NH
) ]2+2+(aq) + 8 S2O
(aq)
2 [Cu(S
) ] (aq) +2-S4O628
2 [Cu(S
(aq)
+ (aq)
8 NH3+(aq)
2 [Cu(NH
3 3(aq)
2O3)3]2O
3 34)4] (aq) + 8 S2O
3 3 5-+ S4O6 (aq)
NH3(aq)
2-

Numa soluo alcalina o on tetrationato desproporciona a on tritionato, S3O6 , e a

Numa soluo alcalina o on tetrationato desproporciona a on tritionato,


S3O62-, e a tiossulfato.

tiossulfato.

Escreva a equao qumica devidamente balanceada que representa esta reao de

Escreva a equao qumica devidamente balanceada que representa esta

desproporcionamento
reao de desproporcionamento

Ano Internacional de Qumica



| 129

Programa Nacional
Olimpadas de Qumica

43rd IChO
g. Quando a concentrao de O2 muito elevada, o on S2O32- oxida-se aos
ons tritionato e sulfato. Escreva a equao qumica devidamente balanceada que traduz esta reao.

PROBLEMA 7

8,5% do total
A S B C D E F G1 G2 H I
2 2 2 2 2 2 1

J K L M 1a 1b

2 2 2 1 1 1

Problema 7

x%

26

8,5

Sntese de um carbasugar
Os hidratos de carbono so componentes essenciais para as clulas e fonte de
energia para os animais. Nesta classe esto incluidos desde acares simples
com pequenas molculas at estruturas macromoleculares. No monossacardeo, quando o oxignio do anel (oxignio endocclico) substituido pelo
grupo metilnico, formam-se compostos designados por pseudosugars
ou carbasugars.
Uma vez que os carbasugars no so hidrolisados na presena de cidos e
enzimas, eles tm sido aplicados como inibidores de glicosidases.
A sntese total de duas formas isomricas do carbasugars 1 descrita a
seguir.

130 |

Ano Internacional de Qumica


Olimpada Brasileira
de Qumica 2011

Exame Terico

1
A sntese de 1 inicia-se com a reduo do benzeno com sdio na presena de
amnia lquida obtendo-se o composto A. O espectro de RMN de C-13 de A
apresenta dois sinais em 124,0 e 26,0 ppm.
O cloreto de tricloroacetila em presena de Zn origina a espcie reativa S. Um
equivalente de S promove uma ciclo-adio [2+2] com A e forma um produto racmico B. A reao de B com Zn em cido actico origina o composto
C. O composto C contm apenas tomos de carbono, hidrognio e oxignio.
O espectro de RMN de C-13 de C apresenta trs sinais correspondentes a
carbonos sp2, em 210,0; 126,5 e 125,3 ppm.

A reao de C com um equivalente de cido m-cloroperbenzico (m-CPBA)


em diclorometano origina majoritariamente o composto D. O espectro de
RMN de C-13 de D apresenta, tambm, trs sinais na regio de carbonos sp2
em 177,0; 125,8 e 124,0 ppm.
Escreva as estruturas dos compostos A, B, C, D, e do intermedirio S.
A reduo do composto D com LiAlH4 origina E, que reage com um excesso
de cloreto de acetila em piridina para dar o composto F. Escreva as estruturas
(use apenas um enantimero) de E e F, usando as notaes cunha tracejada
e cunha cheia. Indique ainda, para o composto E as configuraes (R ou S)
dos carbonos assimticos.
O composto F (utilize o enantimero anteriormente representado) reage com

Ano Internacional de Qumica



| 131

Programa Nacional
Olimpadas de Qumica

43rd IChO
bromo originando os estereoismeros G1 e G2. Escreva as estruturas destes
compostos usando novamente a notao tracejada/cheia.
A mistura de G1 e G2 reage com dois equivalentes de 1,8-diazabiciclo[5.4.0]
undec-7-eno (DBU), que uma amina e simultaneamente uma base forte,
originando o composto H. Escreva a estrutura do composto H usando novamente a notao tracejada/cheia.

Escreva as estruturas dos compostos E, F, G1, G2 e H usando a notao de


cunha tracejada/cheia.
A reao do composto H com oxignio singleto (originado no meio reacional) origina o composto I. Embora nesta reao seja possvel teoricamente a
formao de dois ismeros, forma-se apenas o composto I devido ao impedimento estereoqumico e repulso eletrnica.
Da reao do composto I com excesso de LiAlH4 resulta a formao do composto J. O espectro de RMN de C-13 de J apresenta 8 sinais, dois deles na
regio de carbonos sp2.
A reao do composto J com excesso de cloreto de acetila na presena de
piridina origina o composto K. A reao subsequente de K com OsO4, na presena de 4-metilmorfolina 4-xido (NMO) origina os esterioismero L e M.
Por reduo de excesso de LiAlH4, os compostos L e M originam os esterioismeros 1a e 1b, respectivamente.

132 |

Ano Internacional de Qumica


Olimpada Brasileira
de Qumica 2011

Exame Terico

Escreva as estruturas dos compostos I, J, K, L, M, 1a, e 1b usando a notao


de cunha tracejada/cheia.

PROBLEMA 8

6,5% do total
B

1,5 1,5 1,5 1,5

Problema 8
18

x%
6,5


Click chemistry um conceito qumico introduzido por K. B. Sharpless, em
2001, que descreve um conjunto de reaes qumicas que possibilitam a obteno rpida, eficiente e quantitativa, de um conjunto de novas substncias,
por juno de pequenas unidades estruturais moleculares e em condies
suaves. Esta metodologia foi recentemente aplicada como um passo chave
na sntese de compostos bicclicos.
O cido mandlico um composto natural verstil e muito usado como fonte de quiralidade em sntese orgnica. A reduo do cido (R)-mandlico
com LiBH4 origina o composto A.

Ano Internacional de Qumica



| 133

Programa Nacional
Olimpadas de Qumica

43rd IChO

A reao do composto A com 1 equivalente de cloreto de p-toluenosulfonila


origina o composto B. O aquecimento de B em piridina produz o composto
C. Durante esta transformao, os compostos B e C mantm as suas configuraes absolutas.

Escreva as estruturas dos compostos B e C representando com a estereoqumica correta. Utilize a notao cunha tracejada/cheia no desenho das estruturas deste problema.
A reao do composto C com azida de sdio em soluo aquosa de acetonitrila origina uma mistura de regioismeros enantiomericamente puros,
D e E, na razo 3:1. Por outro lado, o composto B, nas mesmas condies
reacionais, origina somente o composto E.

Escreva as estruturas dos compostos D e E representando com a estereoqumica correta.


Parte I: Os compostos D e E so separados atravs da reao com 3-bromoprop-1-ino na presena de NaH, originando os compostos G e G, respectivamente. Os compostos G e G quando aquecidos em tolueno originam os
compostos bicclicos H e I, respectivamente.
134 |

Ano Internacional de Qumica


Olimpada Brasileira
de Qumica 2011

Exame Terico

Escreva as estruturas dos compostos F,G,H e I representando com a estereoqumica correta.


Parte II: A reao de cada um dos compostos D e E com acetilenodicarboxilato de dimetila em gua a 70C origina, respectivamente, os regioismeros
monocclicos J e K, opticamente ativos. Os compostos J e K, quando tratados
com NaH, originam, respectivamente, os produtos bicclicos L e M, ambos
com a frmula molecular C13H11N3O4.

Escreva as estruturas dos compostos J,K,L e M representando com a estereoqumica correta.

Se j sabes o que tens a fazer e no o fazes,


ento estais pior que antes.
(Confucio).

Ano Internacional de Qumica



| 135

Programa Nacional
Olimpadas de Qumica

16a OIAQ

XVI Olimpada Iberoamericana de Qumica


Problemas Tericos
20 de setembro de 2011
Teresina, Piau, Brasil

PROBLEMA N 1: QUMICA INORGNICA


Estequiometria e Geometria Molecular

6% do total
Questes

II

III

IV

VI

total

Pontuao

1,0

1,0

1,0

1,0

1,0

1,0

6,0

As plantas necessitam de diversos elementos qumicos para sua sobrevivncia, so eles: carbono, hidrognio, oxignio, nitrognio, fsforo, enxofre, clcio, magnsio e potssio, boro, cobalto, cobre, ferro, mangans, molibdnio
e zinco. Para suprir a deficincia desses elementos qumicos so aplicados
ao solo e/ou plantas os chamados fertilizantes ou adubos sintticos, com o
intuito de melhorar a produo. A indstria de fertilizante uma das maiores
do mundo, visto que a populao precisa de alimentos mais que qualquer
outro bem de consumo. S no ano de 2009 cerca de 200 milhes de toneladas de fertilizante, da classe de potssio, nitrognio e fsforo, foram consumidas no mundo, sendo a China o maior consumidor.
A indstria BOSS produz o fertilizante chamado de superfosfato. A produo realizada tratando-se o fosfato de clcio com 92% de pureza pelo cido
sulfrico concentrado, de acordo com a seguinte reao:
Ca3(PO4)2(s) + 2 H2SO4(l)
136 |

2 CaSO4(s) + Ca(H2PO4)2(s)

Ano Internacional de Qumica


Olimpada Brasileira
de Qumica 2011

Exame Terico
Em uma batelada de superfosfato produzida pela BOSS, foram misturados
0,50 Mg (megagrama) de fosfato de clcio com 0,26 Mg (megagrama) de cido sulfrico, obtendo-se 0,28 Mg (megagrama) de superfosfato Ca(H2PO4)2.
Responda os itens abaixo.
I) Qual o reagente limitante?
II) Qual a % do reagente em excesso?
III) Qual o rendimento percentual do fosfato em superfosfato?
IV) Desenhe todas as possveis estruturas de Lewis para o on PO4-3.
V) Qual a carga formal ao redor dos tomos de fsforo e de oxignio no
on PO4-3?
VI) Qual a geometria molecular para o on PO4-3?
M (g.mol-1): Ca3(PO4)2 = 310,10; H2SO4 = 98,04; CaSO4 = 136,10;
Ca(H2PO4)2 = 233,98

PROBLEMA N 2: QUMICA INORGNICA


Estrutura Cristalina de xidos Metlicos

9% do total
Questes

II

III

IV

total

Pontuao

2,0

2,0

1,5

1,5

2,0

9,0

Os xidos metlicos nanoestruturados, que apresentam suas dimenses menores que 100 nanmetros, constituem uma promissora classe de materiais
avanados, que exibem propriedades fsicas e qumicas melhoradas com
aplicaes nas reas da nanocincia e nanotecnologia. Dentre estes xidos,
as estruturas cristalinas do trixido de rnio (ReO3) e o xido de titnio (IV)
- TiO2 apresentam clulas unitrias distintas. O TiO2 ocorre em trs formas
cristalinas diferentes: rutilo, anatase e brookita, porm o rutilo a forma
mais comum encontrada, sendo utilizado como pigmento branco em tintas,
plsticos e papel. A estrutura cristalina do rutilo (Figura 1) um exemplo de
uma rede de nions de estrutura hexagonal densa com ctions ocupando
somente metade dos buracos octadricos.

Ano Internacional de Qumica



| 137

Programa Nacional
Olimpadas de Qumica

16a OIAQ

Figura 1. Clula unitria do TiO2.

O ReO3, de colorao vermelha, o xido mais estvel do grupo 7 e pode ser


obtido atravs da reao do xido de rnio (VII) com monxido de carbono
de acordo com a reao:
Re2O7 + CO

2ReO3 + CO2

A clula unitria do ReO3 cbica com Re em cada vrtice da clula unitria


e um tomo de O em cada aresta da clula unitria a meio caminho entre os
tomos de Re.
I) Esboce a clula unitria do ReO3.
II) Calcule o nmero total de tomos presente na clula unitria do ReO3.
III) Determine o nmero de coordenao do ction e do nion na estrutura
do ReO3.
IV) Determine o nmero de coordenao (N.C.) do ction e do nion na estrutura do TiO2.
V) Calcule o nmero total de tomos presente na clula unitria do TiO2.

PROBLEMA N 3: FSICO QUMICA


Termodinmica e Equilbrio de Fases

8% do total
Questes

Pontuao
138 |

I
2,0

II
2,0

III
2,0

IV
2,0

total
8,0

Ano Internacional de Qumica


Olimpada Brasileira
de Qumica 2011

Exame Terico
Os xidos met
Por ser um pas de grande extenso territorial e cortado pela linha do equador
e o trpico de capricrnio, o Brasil apresenta uma considervel diversidade
climtica. Em decorrncia disso, possvel observar, em um nico dia, grandes
diferenas de temperaturas entre duas cidades brasileiras. Tal fato pode representar um problema para algumas atividades, como o transporte de materiais
volteis, j que pode haver um considervel aumento da presso de vapor
destes materiais com o aumento da temperatura, o que pode provocar exploses. Para evitar tal consequncia, os reservatrios dos caminhes que transportam materiais volteis so providos de um dispositivo de segurana, que
ativado para aliviar a presso interna quando a mesma atinge certo valor.
Tendo conhecimento das informaes mencionadas acima, um caminhoneiro saiu de Caxias do Sul (Rio Grande do Sul, Brasil), transportando 1,0 t de
ter etlico em um reservatrio cilndrico de 2,0 m3, em um dia cuja temperatura era de 5,0 C, tendo como destino final, Teresina Piau. O reservatrio
de carga do caminho era provido do dispositivo de segurana mencionado
anteriormente, que era ativado automaticamente quando a presso interna
do reservatrio atingisse 5 atm.
I) Admitindo que todo ter etlico (1,0 tonelada) tenha sido colocado no
reservatrio no estado lquido, e que o caminho tenha ficado parado
tempo suficiente em Caxias do Sul (T = -5,0 C) para que a carga tenha
entrado em equilbrio trmico com o ambiente, determine a frao, em
massa, de ter que se encontrava no estado de vapor momentos antes do
incio da viagem para Teresina.
II) Considerando as informaes anteriores e sabendo que, ao chegar a Teresina, quatro dias depois da sada, o caminhoneiro deparou-se com uma
temperatura ambiente de 40 C, demonstre, com os clculos necessrios,
se a vlvula de segurana do reservatrio foi ativada, ou no, quando o caminhoneiro chegou a Teresina. Considere que durante todo o trajeto entre
Caxias do Sul e Teresina a temperatura no tenha ultrapassado os 25 C.
III) Imagine que a temperatura do caminho tenha atingido uma temperatura tal, em determinada situao, que a vlvula de segurana tenha sido
ativada. Considerando esta informao e que o vapor do ter etlico comporta-se como um gs ideal, determine a temperatura mnima necessria
para que a vlvula tenha sido ativada.

Ano Internacional de Qumica



| 139

III) Imagine que a temperatura do caminho tenha atingido uma temperatura t


determinada situao, que a vlvula de segurana tenha sido ativada. Considerand
informao e que o vapor do ter etlico comporta-se como um gs ideal, deter
Programa Nacional
Olimpadas de Qumica
temperatura mnima necessria para que a vlvula tenha sido ativada.

16a OIAQ
IV) Considerando que toda a carga de ter etlico estivesse no estado lquido no in

de temperatura entre 5,0 C e 10 C, sob presso constante, determine as varia


IV) Considerando
carga de
ter
etlico estivesse
no estado
lquido
entalpia que
(H)toda
e de aentropia
(S)
associadas
a esta variao
de temperatura.
no intervalo de temperatura entre 5,0 C e 10 C, sob presso constante,
determine as variaes de entalpia (H) e de entropia (S) associadas a
esta variao de temperatura.
Dados sobre o ter etlico:
Dados sobre o ter etlico:
Presso de vapor = 58,9 kPa a 20 C;
Presso de vapor = 58,9 kPa a 20 C;

-x
p = p*e
p de
a presso
vapor a temperatura
p* adepresso de v
p = p*e-x, onde
p ,aonde
presso
vapor adetemperatura
T, p* a T,
presso

vapor a temperatura
temperaturaT*
T*ee =

Hvap = 29,0 kJ/mol;


densidade = 0,750 g/mL (-5,0 C);
Hvap = 29,0 kJ/mol;
densidade = 0,750 g/mL (-5,0 C);
Tcrtica = 193 C; Pcrtica = 3,64 MPa; R = 8,314 J mol-1 K-1, -11 atm
=101325
Tcrtica = 193 C; Pcrtica = 3,64 MPa; R = 8,314 J mol K-1, 1 atm =101325 Pa
Pa
-1 -1J mol-1 K-1;
= 107,1
Cp,m = 107,1Cp,m
J mol
K ;

Para todasPara
as respostas,
considere
que oque
vapor
de ter
etlico
todas as respostas,
considere
o vapor
de ter
etlicotenha
tenha comporta
comportamento
ideal
e
que
o
reservatrio
do
caminho
tenha
ideal e que o reservatrio do caminho tenha somente ter somente
etlico.
ter etlico.

PROBLEMA N 4: FSICO-QUMICA
Equilbrio
Qumico
PROBLEMA
N 4: FSICO-QUMICA
Equilbrio Qumico

7% do total

total 7% do t
B.IV
1,5
7,0
Pontuao
0,6
0,9
1,0
1,0
0,5
1,5
Questes
A.I. A.II A.III B.I B.II B.III B.IV
Para um sistema qualquer, o equilbrio termodinmico se estabelece quando
Pontuao
0,6so alcanados,
0,9
1,0 ou
1,0seja,0,5
1,5
1,5
os equilbrios mecnico, trmico
e qumico
nenhuma mudana de propriedade fsica ou qumica varia com o tempo. Podemos
ter uma viso do equilbrio qumico tanto nas reaes sem transferncia de
Para para
um sistema
qualquer,
o equilbrio termodinmico
se estabelece
os equ
eltrons como
aquelas
com transferncia
de eltrons. Nestas
ltimas,quando
a
mecnico,
trmico
e qumico
so alcanados, ou seja, nenhuma mudana de propr
transformao
de energia
mais
evidente.
fsica ou qumica varia com o tempo. Podemos ter uma viso do equilbrio qumic
PARTE A) As mudanas no ponto de equilbrio podem ser avaliadas pela
aplicao do Princpio de Le Chatelier. Nos sistemas do quadro, que se enQuestes

140 |

A.I.

A.II

A.III

B.I

B.II

B.III

Ano Internacional de Qumica


Olimpada Brasileira

nas reaes sem transferncia de eltrons como para aquelas


com2011
transferncia de eltrons.
de Qumica
Nestas ltimas, a transformao de energia mais evidente.

Exame Terico

PARTE A) As mudanas no ponto de equilbrio podem ser avaliadas pela aplicao do


Princpio de Le Chatelier. Nos sistemas do quadro, que se encontram em equilbrio, se
contram
em
equilbrio,
realizam as
seguintes
aes:se realizam as seguintes aes:
Sistema
1. CO2 (g) + H2 (g) H2O (g) + CO (g)
2. H2 (g) + I2 (g) 2HI (g)
3. 2 NaCl(s) + H2SO4(l) Na2SO4(s) + 2 HCl(g)

Ao
Um agente secante adicionado.
Gs nitrognio adicionado.
A reao realizada em recipiente
aberto.

A.I)
aoanterior,
anterior,
indique
na folha
de respostas
se as afirmaes
A.I)Para
Para cada
cada ao
indique
na folha
de respostas
se as afirmaes
a, b e c soa,
b e c so verdadeiras ou falsas.
verdadeiras ou falsas.

a - A reao 1 no se altera, pois o secante mantm o mesmo volume de


a - A reao 1 no se altera, pois o secante mantm o mesmo volume de gua;
gua;
b - Na reao 2, o gs adicionado provoca uma mudana para a direita;

b - Na reao 2, o gs adicionado provoca uma mudana para a direita;


reao 3 3permanece
sem alterao.
cc-- AA reao
permanece
sem alterao.

AII) Assinale na tabela da folha de respostas o resultado esperado para cada


ao
indicada
quadro
acima.
AII) Assinale
nano
tabela
da folha
de respostas o resultado esperado para cada ao indicada
no quadro
acima.
A.III)
Quantitativamente
possvel avaliar qual reao ocorreu completamente? Ou seja, em qual delas um dos reagentes foi consumido 99,9%? Em caso
afirmativo, indique na folha de respostas o nmero do sistema.
A.III) Quantitativamente possvel avaliar qual reao ocorreu completamente? Ou seja,
em qual delas um dos reagentes foi consumido 99,9%? Em caso afirmativo, indique na

PARTE
A obteno
energia limpa pode ser conseguida atravs das
folha de B)
respostas
o nmero de
do sistema.
reaes de oxi-reduo, como no caso do acumulador de Edson:
Fe(s)|FeO(s)|KOH(aq, c)|Ni2O3(s)|NiO(s)|Ni(s)

PARTE B) A obteno de energia limpa pode ser conseguida atravs das reaes de oxiB.I)
Escreva
asnoreaes
meia pilha;
reduo,
como
caso do de
acumulador
de Edson:

B.II) Qual a reao da


pilha?
Fe(s)|FeO(s)|KOH(aq,
c)|Ni2O3(s)|NiO(s)|Ni(s)
B.III)
Indiqueasna
folha
respostas
qual a relao entre o potencial da pilha
B.I) Escreva
reaes
dede
meia
pilha;
e a concentrao, c, do KOH?
B.II) Qual a reao da pilha?

B.IV) Quanto de energia (G) se obter por quilograma de todos os reagenB.III) Indique na folha de respostas qual a relao entre o potencial da pilha e a
tes da pilha?
concentrao, c, do KOH?

Ano Internacional de Qumica



| 141

Programa Nacional
Olimpadas de Qumica

16a OIAQ

B.IV) Quanto de energia (G) se obter por quilograma de todos os reagentes da pilha?
Dados:
Dados:
MA em
= =15,99
MAg/mol:
em g/mol: H =H1,00
= 1,00 O O
15,99 K K==39,10
39,10
K+ + e K

= 0,440 V

FeO + H2O+ 2e Fe +2OH

Ni + 2e Ni
2+

Ni
58,71
Ni == 58,71

= 2,925 V

Fe + 2e Fe
2+

FeFe==55,85;
55,85;

= 0,877 V

= 0,250 V

Ni2O3 + H2O + 2e 2NiO + 2OH

= +0,40 V

4
Constante
de Faraday
9,648
.104-1C mol-1
Constante
de Faraday
= 9,648= .10
C mol

PROBLEMA N 5: QUMICA ANALTICA


Equilbrio Qumico Kps do Cd(OH)2

7% do total
Questes
I
Pontuao
1,0
PROBLEMA N

II
III
1,0
2,0
5: QUMICA

IV
V
1,0
2,0
ANALTICA

Total
7,0

Equilbrio
Qumico
Kps
do112,41)
Cd(OH)
2 metal pesado
O elemento cdmio
(n atmico
48, massa
atmica
um
altamente txico que ocorre naturalmente como on Cd2+, quase sempre associado com os minrios de zinco, cobre, chumbo e mercrio. Apesar
baixa
7%dado
total
ocorrncia natural, sua presena no ambiente preocupante, pois a toxicidade
alta e a eliminaoQuestes
lenta (efeito cumulativo).
I
II Algumas
III plantas
IV(inclusive
V esp-total
cies comestveis) so bastante tolerantes sua presena no solo e o acumulam
Pontuao
1,0Arroz, 1,0
2,0 so1,0
2,0
7,0
s vezes em quantidades
apreciveis.
trigo e batata
alguns exemplos
bem conhecidos dessa propenso acumulao de cdmio. Peixes pescados em
guas prximas a garimpos ou indstrias metalrgicas ou eletrnicas (baterias
O elemento
cdmiodo(n
atmico
massarepresentam
atmica 112,41)
metal pesado
altamente
recarregveis
tipo
nicad)48,
tambm
riscos um
segurana
alimentar.
2+
Noque
Japo,
uma
doena conhecida
por
itai-itai
atribuda
sempre
associado
comao
os alto
minrios
txico
ocorre
naturalmente
como onlocalmente
Cd , quase
consumo de peixe e arroz pela populao exposta a esses dois tipos de risco.
de zinco,
chumbo e mercrio.
Apesar
ocorrncia
natural,
sua presena
Outrocobre,
fato mundialmente
conhecido
odadabaixa
insanidade
mental
do famoso
pin- no
ambiente preocupante, pois a toxicidade alta e a eliminao lenta (efeito cumulativo).
Ano Internacional de Qumica
142 |

Algumas plantas (inclusive espcies comestveis) so bastante tolerantes sua presena no

Olimpada Brasileira
de Qumica 2011

Exame Terico
tor holands Vincent van Gogh, ao pintar Os Girassis, uma de suas principais
obras, cujos tons amarelos eram obtidos com pigmentos base de cdmio.
Uma poro de hidrxido de cdmio (Kps = 4,5 x 10-15) foi colocada em
gua pura a 25 C e agitada energicamente at a saturao da soluo. A
soluo saturada foi separada do excesso de slido e uma alquota de 100,0
mL foi retirada e reservada para uso posterior.
I) Escreva a equao balanceada para a dissociao do hidrxido de cdmio
em gua;
II) Escreva a expresso do Kps para a soluo saturada de hidrxido de cdmio;
III) Calcule as concentraes dos ons OH-, H+ e Cd2+ na soluo;
IV) Calcule o pH da soluo, a partir da [H+];
V) alquota de 100,0 mL acrescentou-se 0,10 mL de soluo de NaOH 1,00
M; quais as novas concentraes dos ons OH-, H+ e Cd2+? Ignorar a variao de volume.

PROBLEMA N 6: QUMICA ANALTICA


Condutimetria

7% do total
Questes
Pontuao

I
4,0

II
1,0

III
1,0

IV
1,0

Total
7,0

A condutimetria uma tcnica analtica bastante til, tanto na determinao


de parmetros fsico-qumicos de solues eletrolticas, quanto na anlise
quantitativa de espcies inicas em soluo. No primeiro caso, a tcnica ,
s vezes chamada de condutimetria direta e se aplica, por exemplo, na determinao de pK de bases e cidos fracos e de Kps de sais pouco solveis;
no segundo caso, fala-se de titulao condutimtrica, cujas aplicaes so
basicamente as mesmas das outras titulaes feitas com auxlio de instrumentos. Neste caso, mede-se a condutncia da soluo (inverso da resistncia passagem de corrente) em funo do volume de titulante. A diferena
fundamental que se obtm retas bem definidas, cuja interseo (ge-

Ano Internacional de Qumica



| 143

Programa Nacional
Olimpadas de Qumica

16a OIAQ
ralmente obtida por extrapolao) corresponde a um ponto final. Experimentalmente, considera-se o volume gasto neste ponto.
Em uma aula de laboratrio um estudante desatento misturou restos de uma
soluo de HCl com soluo de cido actico (HAc, pKa=4,74). Como castigo
pela desateno, foi-lhe dada a tarefa de determinar a porcentagem de cada
cido na mistura. O aluno, muito esperto, pipetou 10,0 mL da mistura, diluiu
para 100 mL e titulou condutimetricamente com soluo de NH4OH 0,50
M padronizada recentemente. Os valores de volume de titulante (V) e das
respectivas leituras de condutncia (G), em unidade arbitrria (u.a), esto no
quadro abaixo.
V
(mL)

0,0

1,0

2,0

2,5

3,0

3,1

3,2

3,5

4,0

4,5

5,0

6,0

7,0

G 2,85 2,50 2,10 1,90 1,70 1,66 1,70 1,75 2,00 2,15 2,15 2,14 2,16
(u.a)
I) Trace o grfico de G versus V e encontre os pontos de equivalncia correspondentes neutralizao de cada cido na soluo da amostra;
II) Calcule a massa de cada cido na amostra;
III) Determine a composio da mistura (% m/v);
IV) A diluio da amostra na proporo de 1:10 e o uso de uma soluo
concentrada (0,50 M) como titulante, foi intencional e teve por objetivo:
Marque a alternativa correta na folha de respostas.

144 |

Ano Internacional de Qumica


Olimpada Brasileira
de Qumica 2011

Exame Terico
PROBLEMA N 7: QUMICA ORGNICA
Transformaes do Lquido da Castanha de Caju

8% do total
Questes

AI

AII

AIII

AIV

BI

BII

BIIII

BIV

total

Pontuao

1,0

1,0

1,0

1,0

1,0

1,0

1,0

1,0

8,0

O lquido da casca da castanha do caju (LCC) ou cashew nut shell liquid (CNSL)
uma das fontes mais ricas de lipdeos fenlicos no-isoprenides de origem natural, cuja composio qumica principal cido anacrdico, cardol,
2-metilcardol e cardanol. Uma vez separado, o cardanol, pode ser empregado no setor da qumica fina, onde os preos dos produtos finais: aditivos, surfactantes, frmacos, pesticidas, dentre outros, so elevados. Por ser
um subproduto da indstria de castanha, qualquer melhoria: concentrao,
separao e preparao de derivados se caracteriza verdadeiramente como
uma inovao tecnolgica.
PARTE A) Considerando a relevncia das snteses orgnicas envolvendo estes compostos, complete a sequncia de reaes do cardanol, indicando as
estruturas dos compostos I, II, III e IV:
OH

10% Pd/C, Mg, CH3COONH4

CH3SO2Cl

Hidrogenlise

C15H31
Composto I

Composto II

HNO3 H2SO4(cat)

H2, EtOH
10% Pd/C
C21H34N2O4
Composto IV

Composto III

PARTE B) As xantonas tm sido relatadas por possurem diversos efeitos biolgicos tais

como:
propriedades
antitumorais, anti-inflamatrios, antitrombtico, antimicrobial e
Ano
Internacional
de Qumica

| 145
neurofarmacolgicas. Algumas destas xantonas podem ser sintetizadas utilizando-se

Programa Nacional
Olimpadas de Qumica

16a OIAQ
PARTE B) As xantonas tm sido relatadas por possurem diversos efeitos
biolgicos tais como: propriedades antitumorais, anti-inflamatrios, antitrombtico, antimicrobial e neurofarmacolgicas. Algumas destas xantonas
podem ser sintetizadas utilizando-se transformaes estruturais de lipdios
fenlicos isoladas do LCC, os quais so obtidos em abundncia e com baixo
custo. Dentre eles podemos destacar o cido anacrdico (1), o cardol (2) e
anlogos sintticos no intuito de se obter xantonas alquiladas de interesse
farmacolgico.
Complete a seguinte sequncia de sntese de xantonas (3,5,7 e 9) a partir de
derivados do LCC, indicando as estruturas dos compostos 3, 4, 6 e 8:
C15H31

OH

COOH

OH

ZnCl2, POCl3, 60 C, 2 h

HO

OH

70%

OH

OH

+
HO

2
OH

HO

C15H31

C15H31

ZnCl2, POCl3, 60 C, 2 h

65%
2

OH

ZnCl2, POCl3, 60 C, 2 h

65%

OH

ZnCl2, POCl3, 60 C, 2 h

146 |

3
H31C15

C15H31

COOH

75%

C15H31

C15H31

OH

C15H31

PROBLEMA N 8: QUMICA ORGNICA


Ano Internacional de Qumica

Sntese Assimtrica

Olimpada Brasileira
de Qumica 2011

Exame Terico
PROBLEMA N 8: QUMICA ORGNICA
Sntese Assimtrica

8% do total
Questes

AI

AII

AIII

AIV

AV

AVI

BI

BII

BIII

BIV

total

Pontuao 1,0

1,0

1,0

1,0

1,0

1,0

0,8

0,4

0,4

0,4

8,0

As snteses assimtricas so de grande importncia para indstria farmacutica na preparao de frmacos quirais, opticamente puros. Essas snteses
so realizadas, muitas vezes, partindo-se de um produto natural opticamente
ativo. Na sequncia de reaes apresentada abaixo, tem-se um exemplo de
sntese assimtrica que leva formao da olefina VII a partir do cido L(+)
tartrico, comercialmente disponvel, que por sua vez pode ser extrado do
suco da uva. Essa sntese foi realizada como parte de um projeto de tese de
doutorado que tinha como objetivo final a olefina VII que pode ser usada
tese de doutorado que tinha como objetivo final a olefina VII que pode ser usada como
como
substrato para a obteno de uma srie de compostos de interesse
substrato para a obteno de uma srie de compostos de interesse farmacolgico.
farmacolgico.
HOOC
H
HO

H
MeOH, H2SO4
OH
Composto II
refluxo 18 h
COOH

Etapa 2

COOMe

LiAlH4, Et2O
1. DMSO, (COCl)2, -60 C, 1,5 h
2. TEA, -60 C, 15 min
Oxidao de Swern (oxidao branda)

Etapa 6
O

COOMe

Composto III

Composto I

Composto VI

CH2OCH2C6H5

CH2OH

refluxo, 3 h
Etapa 4

Composto IV

Composto V

CH2OCH2C6H5

Composto VII

Onde TEA= trietilamina


A) Complete a sequncia de reaes acima, indicando os reagentes apropriados para as
etapas 2, 4 e 6 e os produtos que se formam nas etapas 1, 3 e 5 (compostos II, IV e VI,
respectivamente).

B) Escreva
os nomes
do composto I incluindo a estereoqumica e indique a configurao
Ano
Internacional
de Qumica

| 147
dos carbonos assimtricos nos compostos III, V e VII, incluindo as configuraes dos
carbonos assimtricos.

Programa Nacional
Olimpadas de Qumica

16a OIAQ
A) Complete a sequncia de reaes acima, indicando os reagentes apropriados para as etapas 2, 4 e 6 e os produtos que se formam nas etapas 1, 3 e
5 (compostos II, IV e VI, respectivamente).
B) Escreva os nomes do composto I incluindo a estereoqumica e indique a
configurao dos carbonos assimtricos nos compostos III, V e VII, incluindo as configuraes dos carbonos assimtricos.

Para se ter o que nunca se teve


preciso fazer o que nunca se fez
148 |

Ano Internacional de Qumica


Olimpada Brasileira
de Qumica 2011

Exame Prtico

XVI Olimpada Iberoamericana de Qumica


Problemas Prticos
PROBLEMA EXPERIMENTAL N 1
Determinao da pureza de uma amostra de sulfato de cobre
Perguntas

Total

Pontuao

13

25

Introduo
O sulfato de cobre comercial ou de uso tcnico apresenta-se geralmente
como cristais azuis de CuSO4.5H2O, s vezes acompanhado de material amorfo de tonalidade acinzentada. largamente usado em agropecuria como
micronutriente de plantas e animais e tambm como fungicida. A indstria
tambm o utiliza com graus de pureza e hidratao variveis, em diversas
aplicaes tecnolgicas, como: petroqumica, qumica fina, corantes e pigmentos, curtume, detergente, galvanoplastia, txtil, papel e celulose, etc.
O sal de alta pureza (reagente P.A.) extremamente til em pesquisa e em
trabalhos prticos de ensino, especialmente no campo da eletroqumica e
qumica eletroanaltica. Os potenciais de reduo do cobre envolvendo os
estados de oxidao (0), (I) e (II) so tais que a interconverso entre eles e
tambm as reaes destes com outras espcies eletroativas, permitem sua
utilizao em condies relativamente brandas, dispensando, em muitos casos, a necessidade de se usar eletrodos e/ou solventes especiais.

Ano Internacional de Qumica



| 149

Programa Nacional
Olimpadas de Qumica

16a OIAQ
Neste experimento ser feita a anlise de uma amostra de CuSO4.5H2O, cujos
sinais de contaminao e/ou deteriorao eram bem visveis no sal slido e
na soluo recm-preparada: no slido era visvel a predominncia de um p
amorfo cinzento-esbranquiado com raros cristais pequenos e mal formados; na soluo recm-preparada (as amostras como esto agora), observa-se slido depositado no fundo do recipiente e a ausncia da cor azul brilhante caracterstica.
Em casos assim, recomenda-se que seja feito algum tratamento, visando a
purificao do composto. Recomenda-se tambm uma anlise prvia do
composto impuro, a fim de que a eficcia do tratamento tambm seja avaliada, mediante anlise posterior do composto tratado e comparao dos
resultados. Para conseguir esse objetivo, o mtodo iodomtrico, que faz uso
de tiossulfato de sdio como redutor de iodo, e goma de amido como indicador, mostra-se bastante eficaz, rpido e seguro (desde que entre as impurezas no haja qualquer interferente).

Procedimento Experimental
Realize o experimento em triplicata.
1. Transfira 25,0 mL de soluo da amostra (sulfato de cobre impuro) usando
uma pipeta volumtrica de 25 mL, para um bquer de 150 mL e acrescente
3,0 g de iodeto de potssio slido por experimento e agite bem com basto de vidro e deixe a mistura repousar por cerca de 5 minutos. Aproveite
esse tempo para encher a bureta com a soluo de tiossulfato de sdio
0,05 M, tendo o cuidado prvio de lavar a bureta com a soluo de tiossulfato de sdio.
2. Separe o precipitado de CuI slido da soluo-me, usando papel de filtro
quantitativo e recolha o filtrado em erlenmeyer de 250 mL. Lave bem o
precipitado e o papel, usando uma soluo mista* de KI e Na2S2O3 (3 a
5 pequenas pores, ou at o desaparecimento das manchas escuras do
papel, use conta gota, se necessrio) e, finalmente, com soluo diluda de
KI sem tiossulfato.
3. *SOLUO MISTA: retire 2,0 mL da soluo de tiossulfato da bureta, recebendo-a em um bquer de 25 mL; acrescente cerca de 10 mL da solu150 |

Ano Internacional de Qumica


Olimpada Brasileira
de Qumica 2011

Exame Prtico
o diluda de KI e faa uma ligeira agitao circular ou com basto para
homogeneizar. Esta soluo deve ser usada integralmente, mesmo que o
precipitado e o papel fiquem limpos antes de us-la toda. (caso necessrio, preparar mais soluo mista).
4. Coloque o erlenmeyer com o contedo sob a bureta e titule com a soluo de tiossulfato de sdio 0,05 M (agitao manual), at o contedo do
erlenmeyer tornar-se amarelo bem claro. Acrescente, ento, 3,0 mL do indicador (goma de amido) e continue titulando at a viragem do indicador
de azul para incolor.
5. Reaes qumicas no balanceadas:
I) Cu2+(aq) +

I- (aq) CuI (s) + I3- (aq)

II) I3-(aq) + S2O32-(aq) I-(aq) + S4O62-(aq)


Questes
1. A soluo mista usada na lavagem do papel de filtro com precipitado
composta de KI e Na2S2O3, ambos diludos. A funo de cada reagente,
nesta ordem :
a) Inibir a dissoluo do CuI e reduzir o iodo (impregnado no papel) a on
iodeto ( ).
b) Inibir a dissoluo do CuI e solubilizar o iodo (impregnado no papel),
sem que este seja reduzido a on iodeto ( ).
c) Solubilizar o iodeto de cobre (I) e reduzir o on Cu+ a cobre metlico (
+

2+

d) Solubilizar o iodeto de cobre (I) e oxidar o on Cu a on Cu

).

( ).

2. O volume de soluo de Na2S2O3 que foi retirado da bureta para preparar


a soluo mista deve ser considerado no volume total de titulante gasto na
titulao?
3. A) Balancear as equaes I e II.
B) Registre na tabela os volumes totais utilizados de tiosulfato de sdio em
cada titulao.
C) Qual o volume de tiosulfato de sdio que ser usado nos clculos?

Ano Internacional de Qumica



| 151

Programa Nacional
Olimpadas de Qumica

16a OIAQ
4. Se o sulfato de cobre usado na preparao da soluo analisada fosse puro
(MM = 249,68 g.mol-1), a massa de CuSO4.5H2O existente na alquota analisada seria_________g; a massa encontrada foi __________ g. Com base nestes valores, o teor de impurezas do sal analisado de__________% (m/m).
5. O valor de Kps do CuI 1x10-12. Com base neste valor, responda:
a) A massa de KI usada no experimento garante suficiente excesso de on
iodeto, para que a reao seja seguramente quantitativa?
b) Admitindo, mais uma vez, que o sulfato de cobre usado fosse puro, qual
seria o volume esperado de titulante que voc usou neste experimento?
c) A baixa solubilidade do CuI em gua contribui para a espontaneidade
da reao?

PROBLEMA EXPERIMENTAL N 2
Identificao de grupos funcionais por testes qumicos
Perguntas

Pontuao

1
6

2
6

3
3

Total
15

Atualmente os compostos orgnicos so identificados pelo uso de mtodos fsicos, tais como: a espectroscopia de ressonncia magntica nuclear
de hidrognio e carbono-13 (RMN1H e RMN13C), espectroscopia no infravermelho (IV) e no ultravioleta (UV) e espectrometria de massas (EM). No
entanto, alguns testes qumicos simples, de fcil execuo, continuam sendo
teis para a rpida caracterizao da presena de certos grupos funcionais
em uma molcula orgnica, uma vez que um grupo funcional a parte da
molcula onde as suas reaes qumicas ocorrem; a parte que efetivamente
determina as propriedades qumicas do composto (e muitas das suas propriedades fsicas tambm).
Dentre estes testes podemos destacar os testes de Lucas, de Jones, de Tollens,
do Iodofrmio, da 2,4-Dinitrofenil-hidrazina e do Cloreto frrico.
Neste experimento so fornecidas 6 (seis) amostras desconhecidas e uma
lista de 12 (doze) compostos, entre os quais se encontram os compostos
152 |

Ano Internacional de Qumica


Olimpada Brasileira
de Qumica 2011

Exame Prtico
correspondentes a cada uma dessas amostras. Voc dever, com base nos
resultados obtidos com a realizao dos testes acima citados, identificar cada
uma dessas amostras.
Os procedimentos utilizados em cada um dos testes citados acima so os
seguintes:
Teste de Lucas: Misturar, em um tubo de ensaio seco, 1 mL do reagente de
Lucas com 4 ou 5 gotas da amostra a ser analisada. A turvao da soluo ou
o aparecimento de duas camadas indicam TESTE POSITIVO.
Teste de Jones: Colocar em um tubo de ensaio, 4 ou 5 gotas da amostra a
ser analisada e juntar 4 ou 5 gotas da soluo de cido crmico. O aparecimento imediato de um precipitado verde indica TESTE POSITIVO. No realizar esse teste com a amostra A: reao altamente exotrmica.
Teste de Tollens: Colocar em um tubo de ensaio, cerca de 0,5 mL do reagente Tollens e 0,5 mL da amostra. A formao de um precipitado escuro
de prata e/ou a formao de espelho de prata so resultados indicativos de
TESTE POSITIVO.
Teste com 2,4-dinitrofenil-hidrazina (2,4-DNF): Colocar em um tubo de
ensaio, 4 ou 5 gotas da amostra a ser analisada em cerca de 2 mL da soluo
de 2,4-dinitrofenil-hidrazina. Agitar e deixar em repouso por 15 minutos. Um
precipitado amarelo-avermelhado considerado TESTE POSITIVO.
Teste do iodofrmio (CHI3): Colocar em um tubo de ensaio, 4 ou 5 gotas
da amostra a ser examinada e juntar 1 mL de soluo de NaOH a 10%. Em
seguida adicionar soluo de iodo, gota a gota, agitando sempre, at um
leve excesso, evidenciado pela colorao tpica do iodo, persistente por 5
minutos. O aparecimento de um precipitado amarelo de iodofrmio considerado TESTE POSITIVO.
Teste com cloreto frrico (FeCl3): Colocar em um tubo de ensaio, cerca de
1 mL da soluo de cloreto frrico a 3%, adicionar 4 ou 5 gotas da amostra e
observar o desenvolvimento de cor. A colorao violeta ou preta indica TESTE
POSITIVO.

Ano Internacional de Qumica



| 153

Programa Nacional
Olimpadas de Qumica

16a OIAQ
PROCEDIMENO EXPERIMENTAL
Utilizando os tubos de ensaio disponveis, realize, dentre os testes acima citados, aqueles que forem necessrio para a identificao de cada uma das
amostras (A a F) que lhe forem fornecidas, dentre os compostos citados a
seguir:
01
02
03
04
05
06
07
08.
09.
10
11
12

cido glico (cido 3,4,5-tri-hidroxibenzico)


lcool terc-butlico
Benzaldedo
Cardanol (3-pentadecilfenol)
n-Butanol
Ciclo-hexeno
D-Glicose
D-Manitol
cido L-tartrico
Hexano
Pentan-3-ona
Propanona

1. Preencha a tabela abaixo, com (+) se o teste for considerado positivo, (-)
se o teste for considerado negativo e (NR) se no foi necessrio realizar o
teste para a identificao da amostra:
Amostra

RESULTADO DOS TESTES


Lucas

Jones

Tollens 2,4-DNF

CHI3

FeCl3

Composto

A
B
C
D
E
F

154 |

Ano Internacional de Qumica


Olimpada Brasileira
de Qumica 2011

Exame Prtico
Observaes:
1. NO REALIZAR O TESTE DE JONES COM O COMPOSTO A.
2. Os compostos slidos esto fornecidos na forma de soluo aquosa.
3. O composto A apresenta a atividade ptica.
4. O composto B reage com bicarbonato de sdio.
5. O composto C d teste de Bayer (reao com KMnO4) negativo.
6. O composto F no pticamente ativo.

2. Dados os reagentes utilizados nos testes
Lucas: Cloreto de zinco (ZnCl2 ) em HCl concentrado.
Jones: Trixido de cromo (CrO3) em H2SO4 diludo.
Cloreto frrico: Cloreto de ferro III (FeCl3.6H2O) em gua, acidificada com
HCl.
2,4-Dinitrofenil-hidrazina: 2,4-dinitrofenil-hidrazina em etanol 95% acidificado com HCl.
Iodofrmio: Soluo de NaOH e Soluo de iodo em iodeto de potssio.
Tollens: Soluo amoniacal de nitrato de prata, [Ag(NH3)2] OH.
Escreva as reaes qumicas gerais que traduzam reaes de compostos orgnicos que do teste positivo com os reagentes indicados.
3. Escreva a reao qumica que poderia ser utilizada para a identificao de
um alceno e outra reao qumica que poderia ser utilizada na identificao de um cido carboxlico.

Ano Internacional de Qumica



| 155

Programa Nacional
Olimpadas de Qumica

Destaques

Destaques Olmpicos - 2011


Ao iniciar o processo seletivo para a escolha da equipe que representa
o Brasil nas olimpadas internacionais de qumica em 2011 tnhamos 35 estudantes dos quais quinze participaram do 10 Curso de Aprofundamento
e Excelncia em Qumica ministrado no Instituto de Qumica da UNICAMP.
Durante 15 dias e com a participao de professores do Programa de Ps-graduao do IQ-UNICAMP esse curso transcorreu com a participao de
estudantes das seguintes unidades federativas: Cear: Bianca Rohsner Bezerra, Breno Saldanha Sousa, Bruno Limaverde Villar Lbo, Davi Rodrigues Chaves, Davidson Anthony Arago Freire, Emerson Holanda Marinho, Lara Mulato Lima, Natlia Arago Dias , Raul Bruno Machado da Silva, Sergio Pereira de
Oliveira Jnior, Taynara Carvalho Silva e Yuri Jernimo Moreira; Piau: Pedro
Victor Barbosa Nolto; So Paulo: Daniel Arjona de Andrade Hara, Tbata
Cludia Amaral de Pontes. Encerrado o curso, os estudantes se prepararam
para o ltimo exame, FASE VI da OBQ-2010 que definiu aqueles que representaram o Brasil nas olimpadas internacionais realizadas em 2011: 43rd
International Chemistry Olympiad (Ankara) e 16 Olimpada Iberoamericana
de Qumica, realizada em Teresina (os assinalados em negrito integraram as
delegaes).
Para Ankara, local da 43rd International Chemistry Olympiad, a delegao brasileira seguiu com quatro estudantes, Davi, Davidson, Raul e Tbata,
esta do estado de So Paulo e os trs primeiros do estado do Cear. A equipe
brasileira conquistou duas medalhas de bronze e uma medalha de prata, esta
conquistada pelo estudante cearense Davi Chaves.
Na 16 Olimpada Iberoamericana de Qumica, realizada em Teresina, o
Brasil esteve representado pelos estudantes cearenses, Davi, Raul e Tbata e
pelo paulista Daniel Arjona.
A equipe conquistou 3 medalhas de ouro e 1 de bronze, primeiro lugar
geral individual e melhor desempenho por equipe.

156 |

Ano Internacional de Qumica


Olimpada Brasileira
de Qumica 2011

Destaques

EDavi Chaves, medalha de prata na 43rd IChO e


medalha de ouro na 14a OIAQ.

Raul Bruno, medalha de bronze na 43rd IChO e


medalha de ouro na 14a OIAQ.

Tbata Cludia, medalha de bronze


na 43rd IChO e medalha de ouro na 14a OIAQ.

Davidson Anthony, representou o Brasil


na 43rd IChO .

Ano Internacional de Qumica



| 157

Programa Nacional
Olimpadas de Qumica

Destaques

Estudantes que representaram o Brasil na


16a Olimpada Iberoamericana de Qumica
realizada em Teresina-Brasil.
Da esquerda para a direita: Tbata, Davi e
Raul e Daniel.

Daniel Arjona,
medalha de bronze
na 14a OIAQ.

158 |

Ano Internacional de Qumica


Olimpada Brasileira
de Qumica 2011

Destaques

Estudantes que representaram o Brasil na 43rd International Chemistry Olympiad realizada em


Ancara-Turquia. Da esquerda para a direita: Davidson, Tbata, Davi e Raul.

Ano Internacional de Qumica



| 159

Programa Nacional
Olimpadas de Qumica

Depoimentos

Da olimpada para a docncia

inha participao no Programa Nacional de Olimpadas de


Qumica representa um marco que contribuiu para vrias escolhas
e oportunidades. Mal sabia que ao assistir s aulas introdutrias
Qumica no oitavo ano, essa disciplina representaria futuramente
minha vida pessoal, social e profissional. Aps dez anos, ainda h louros por
conta do programa.
O incio se deu em 2002, quando aos doze anos comecei a entender o
que era Qumica, onde poderia us-la e como us-la em nosso favor. A partir
da, ocorreu a premiao da Maratona Cearense de Qumica, o primeiro
estmulo a continuar na carreira. medida que o ano se passava, havia um
maior entendimento entre mim e essa disciplina; foi-se estabelecendo uma
relao ntima, auxiliada por todos os excelentes professores que tive. No outro
ano, mais uma Maratona e uma Olimpada Cearense. J havia sido cativado.
Estudar Qumica virou prazer; curiosidade que me parece infinita de entender
e entender mais e mais. Em outra Olimpada Cearense de Qumica, conquistei
a oportunidade de participar da Fase III nacional da Olimpada Brasileira de
Qumica (OBQ) e, em seguida, a classificao para a Fase IV. J havia uma meta
a cumprir nesse momento: compor a delegao brasileira para as Olimpadas
Internacionais. A meta foi cumprida e participei da 38 Olimpada Internacional
de Qumica (38th IChO) na cidade de Gyeongsan, Coria do Sul, alm da 11
Olimpada Ibero-Americana de Qumica (11 OIAQ), conquistando medalhas
de bronze e ouro, respectivamente.
No entanto, notei que usava a Qumica para crescer, quando participei
da Fase V da OBQ. Junto a um grupo de dezesseis estudantes, criava meus
primeiros laos sociais intermediados por essa cincia. Criei laos de amizade
e at pequenos laos profissionais nesse momento, que me seriam bem
importantes para o futuro. Outros foram criados ao participar da IChO e
fortificados durante a OIAQ. Nesse sentido, houve at um prmio inusitado que
ganhei na Coria, o de participante mais socivel. J havia ento um grande
grupo de contatos espalhados pelo mundo, o que me permitiu estabelecer um
imenso intercmbio cultural. Visitei e fui anfitrio de amigos estrangeiros feitos
ali; a olimpada me havia mostrado que o mundo no to grande assim.

160 |

Ano Internacional de Qumica


Olimpada Brasileira
de Qumica 2011

Depoimentos
Se no to grande, nada melhor que poder escolher onde gostaria de
cursar minha graduao em Qumica, claro. Alm de sociais, haviam as relaes
profissionais criadas, que me conduziram ao maior centro de pesquisa em
Qumica Orgnica do Brasil, qui da Amrica Latina. Cursei minha graduao na
Universidade Estadual de Campinas (UNICAMP), fui recebido com entusiasmo em
seu laboratrio pelo Prof. Dr. Ronaldo Pilli, um dos maiores qumicos orgnicos
sintticos do pas. No somente, as olimpadas tambm me levaram ao Leibniz
Institut fr Pflanzenbiochemie (IPB Instituto Leibniz de Bioqumica Botnica,
traduo livre) em Halle, Alemanha, onde desenvolvi um trabalho de pesquisa
por seis meses. Acredite: nunca pensei que duas medalhas pudessem me levar
to longe.
O crescimento no foi apenas social ou profissional, foi tambm pessoal.
Ser um aluno de olimpadas implica disciplinar-se, criar mtodo e desenvolver
o respeito. Fundamental foi a intensidade do apoio familiar e do incentivo dos
meus eternos mestres, os professores que me mostraram exatamente como
exercer a nobre profisso com dedicao e amor.
Minha participao efetiva em olimpadas se encerrou h seis anos, mas
sou eternamente grato ao que o programa me proporcionou. A maneira que
encontrei de poder retribuir as oportunidades que tive foi escolher ser professor.
Com 22 anos, desenvolvo hoje um trabalho de mestrado em Qumica Orgnica
com o Prof. Pilli, leciono em uma escola de Campinas e sou um entusiasta do
Programa Nacional de Olimpadas de Qumica.
Lucas M. Lira
Bel. em Qumica - UNICAMP

Ano Internacional de Qumica



| 161

Programa Nacional
Olimpadas de Qumica

Depoimentos

Penetrando no mundo
microscpico da qumica
Comecei a participar das olimpadas de qumica quando fazia a 8 srie, em
2000. Decidi participar da competio, pois seria uma forma de me preparar
para a prova especfica de Qumica do vestibular para medicina. Antes disso mal
entendia o que essa cincia estudava, nem o que era tomo!
O que me estimulava a estudar e me aprofundar na qumica e participar
das olimpadas era o prprio cerne da Qumica (o mundo submicroscpico,
as misturas, o uso de muito raciocnio, matemtica, fsica), mas o ambiente
agradvel das aulas preparatrias para a olimpada aliado forma empolgante
que os professores ensinavam foram muito importantes para essa dedicao. No
decorrer do Ensino Mdio, vi que no tinha vocao pra ser mdico, e pensei
em fazer Engenharia Qumica, depois Farmcia, at que escolhi me formar como
bacharel em Qumica. Eu queria continuar a estudar Qumica. Para mim era muito
bom ler os livros de Qumica do Ensino Superior, pensar nos seus desafios e nas
questes referentes a essa cincia.

Participar das olimpadas de Qumica me trouxe muitos benefcios. Na poca


do ensino mdio eu conseguia ver as boas amizades que fiz, a alegria das vitrias, o
prazer do estudo, e o direcionamento na escolha da profisso. Hoje vejo o grande
benefcio do conhecimento bem fundamentado de Qumica adquirido durante as
olimpadas, pois isso me ajudou durante a graduao e tem me ajudado no exerccio
do magistrio superior. Apoio o crescimento do projeto da Olimpada Brasileira
de Qumica, pois acredito que esse projeto contribui para uma nova gerao de
profissionais brasileiros de alta qualidade que traro um progresso ao Brasil, alm
de despertar o interesse pelo conhecimento cientfico nos adolescentes do nosso
pas. Acredito que devemos ser estudantes/profissionais que aliam conhecimento
e trabalho a valores importantes como respeito, humildade, honestidade e
excelncia. Gastar tempo e recursos em avanos cientficos e tecnolgicos que
podem trazer progresso econmico no valer a pena se no contribuirmos para
uma sociedade com mais paz e justia, menos opresso e misria. Nossa profisso
pode ser parte de um grande propsito na vida.
Igor Marques Cavalcante
Bacharel, Mestre e Doutorando em Qumica UFC
Professor substituto Setor de Qumica Geral e Inorgnica DQOI UFC
Medalha de ouro na OBQ 2002 (Mod. A) e na OBQ 2003 (Mod. B)
Medalha de prata na VIII ONNeQ - 2002
Ano Internacional de Qumica
162 |

Olimpada Brasileira
de Qumica 2011

Consideraes Finais

Um ano para ficar na histria


Qumica a chave que abre os horizontes do conhecimento que permitem
vivermos com melhor qualidade de vida em sintonia com a conservao da
natureza. As pesquisas que resultaram em avanos na medicina, na produo em
larga escala de alimentos, nas fontes sustentveis de energia ou na criao de
novos materiais tiveram substancial participao da qumica. So descobertas,
procedimentos e produtos essenciais para o desenvolvimento sustentvel do
nosso mundo derivadas do potencial criativo da qumica.
A UNESCO entendeu que essas importantes contribuies deveriam ser
amplamente divulgadas e mereciam dar amplo conhecimento populao,
assim encaminhou proposta para a ONU destacar 2011 como Ano Internacional
da Qumica - AIQ. A escolha da data deveu-se ao centenrio do Prmio Nobel
de Qumica recebido por Mme Curie, agraciada em 1911 por suas pesquisas
com radioatividade e a descoberta dos elementos rdio e polnio. Dessa forma
destacou o papel da mulher na cincia e a participao feminina para o avano
da qumica e para o bem-estar da sociedade. Apesar da reduzida quantidade
de mulheres laureadas com a premiao Nobel (5,2%) e apenas quatro dentre
162 laureados na rea de qumica, no Brasil, estima-se que a participao da
mulher na rea de qumica ultrapassa 1/3 do total de pesquisadores, um nmero
bastante expressivo. As justas homenagens prestadas Mme Curie no corrente
ano so motivo de estmulo a uma maior participao dos jovens, especialmente
as mulheres, no mundo cientfico e incentivo para seguirem carreira na qumica.
A contribuio feminina se destaca em praticamente todas as coordenadorias
estaduais do Programa Nacional Olimpadas de Qumica, esto elas quer como
coordenadoras ou participando da equipe gestora nos projetos.
Sob o slogan Chemistry - our life, our future qumicos de todas as partes do
mundo movimentaram a sociedade com palestras, exposies, cursos, encontros
cientficos e uma infinidade de experimentos postados na internet para
demonstrar como avanos da qumica impactaram na qualidade de vida do ser
humano. Sociedades qumicas, escolas, universidades e instituies brasileiras que
ocupam dos fenmenos qumicos uniram-se UNESCO e IUPAC para celebrar as
contribuies vitais da qumica. A preocupao com a potabilidade da gua que
consumimos foi um dos temas de grande repercusso, um experimento Global
sobre a Qualidade da gua no planeta movimentou milhes de estudantes no
mundo todo, seus resultados iro integrar um banco de dados sobre a situao
da gua de rios e mananciais de todos os continentes.

Ano Internacional de Qumica



| 163

Programa Nacional
Olimpadas de Qumica

Consideraes Finais
Importantes setores da indstria qumica brasileira associaram-se s
comemoraes do Ano Internacional da Qumica para divulgar a atuao
responsvel e inovadora da indstria nacional, a oitava do mundo e em franco
crescimento. Esse trabalho ter continuidade, em 2012, com a colaborao do
ex-olmpico de qumica, Levindo Garcia Quarto, embaixador da qumica no
Brasil. Ele um dos quinze jovens que, por indicao da IUPAC, foram nomeados
embaixadores em seus pases. Promover a aproximao da indstria com a
academia ser uma de suas misses durante 2012. Para receber orientaes,
juntamente com os outros embaixadores participou de 1 a 5 de outubro passado,
em Berlim, da reunio anual da European Petrochemical Association (EPCA) uma rede global com sede em Bruxelas que movimenta um volume de negcios
de 3,2 trilhes de euros com 3,5 milhes de empregados em todo o mundo.
Embalado nas comemoraes do AIQ, o Programa Nacional Olimpadas de
Qumica estimulou as coordenadorias estaduais a intensificarem as atividades de
promoo da qumica especificamente nas escolas de ensino mdio e fundamental
de forma a proporcionar o interesse dos alunos por essa cincia e, para o grande
pblico, demonstrar os benefcios que a qumica oferece humanidade. Essas
iniciativas tiveram maior alcance nos perodos comemorativos ao Dia do Qumico
e durante a Semana Nacional de Cincia e Tecnologia. Alm das olimpadas
estaduais e regionais apoiadas pelo Programa Nacional Olimpadas de Qumica
os eventos de abrangncia nacional Olimpada Brasileira de Qumica (ensino
Mdio) e a Olimpada Brasileira de Qumica jnior tiveram expressiva participao
dos jovens estudantes.
Neste ano, com incentivo da CAPES, iniciamos ousada ao de apoio ao
ensino e ao estudo da qumica nas escolas pblicas com a participao de
estudantes de licenciatura em qumica, ex-olmpicos do Programa Nacional
Olimpadas de Qumica. Trata-se de projeto que familiariza os licenciandos com as
dificuldades do ensino pblico, contribui para a formao dos futuros professores
de qumica e facilita o acesso de alunos das escolas pblicas s universidades. A
prtica docente acompanhada por professores das prprias escolas pblicas
sob a orientao de docentes dos cursos universitrios de qumica. Quinhentos
e setenta turmas de alunos foram formadas em treze estados brasileiros, um
investimento que se estender durante 2012 proporcionando slida formao
naqueles que optaram pela carreira docente em qumica e estmulos ao estudo
da qumica nos estudantes das escolas pblicas.
O ano se encerra com o anncio de um novo parceiro no Programa Nacional
Olimpadas de Qumica. Vem a Fundao Estudar trazer sua experincia de 20
anos de atuao na rea educacional e conosco colimar esforos no sentido de
164 |

Ano Internacional de Qumica


Olimpada Brasileira
de Qumica 2011

Consideraes Finais
despertar vocaes cientficas, apoiar a formao profissional desses jovens e
incentivar talentos com potencial para liderana. A partir deste ano os participantes
das olimpadas de qumica que atingem a excelncia do rendimento de seus
estudos concorrem ao Prmio Estudar Cincia, oferecido pela Fundao Estudar.
Seja voc um dos selecionados.
Ao concluir mais um ciclo do Programa Nacional Olimpadas de Qumica
sentimo-nos recompensados pelo sucesso alcanado nas diferentes etapas,
traduzido pelo incremento no nmero de participantes, na destacada atuao
nas olimpadas internacionais, na crescente capilarizao do projeto e na exitosa
captao de talentos para a qumica. Um esforo nacional desenvolvido pelos
coordenadores do projeto para vencer o desafio de tornar o pas competitivo nas
reas de mdia e alta tecnologia.

A qumica uma cincia belssima cujo objeto de estudo


est sempre ao nosso redor: as transformaes da matria.
No pude deixar de me apaixonar por esta fascinante
matria a qual, sabia eu desde o princpio, no poderia
larg-la jamais.
Que o gosto pela Qumica seja algo que atravesse
geraes e seja sempre o propulsor para descobertas dos
detalhes do Universo que o homem desconhece. Essa
propulso que me foi dada pela Olimpada de Qumica
no se trata de algo efmero, mas de algo que perdurar
enquanto existir um esprito desbravador dentro de mim.
Raul Bruno Machado da Silva
Estudante olmpico

Ano Internacional de Qumica



| 165

Programa Nacional
Olimpadas de Qumica

Endereos
NCLEO

COORDENADOR

ENDEREO PROFISSIONAL

COORDENADORIA GERAL

Prof. Srgio Maia Melo


melo@ufc.br
sergio@funcap.ce.gov.br

FUNCAP - Fundao Cearense de Apoio ao


Desenvolvimento Cientfico e Tecnolgico
Av. Oliveira Paiva, 941
60.822-130 Fortaleza - CE
(85) 3101. 2170 Ramal 120 3275.7862 (Fax)

COORDENADORIA
REGIES SUL/SUDESTE

Prof. lvaro Chrispino


chrispino@infolink.com.br

CEFET - RJ
(21) 642.6644 (Telefax)

Vice-COORDENADORIA

Prof. Jos Arimatia Dantas Lopes


arilopes@gmail.com
arilopes@ufpi.br

Universidade Federal do Piau


Centro de Cincias da Natureza
64.049-550 Teresina - PI
(86) 3215.5840 3215.5692 (Telefax)

ESTADO

COORDENADOR

ENDEREO PROFISSIONAL

ACRE

Dlcio Dias Marques


delciomarques@globo.com
Prof. Rogrio A. Sartori
rogerio_sartori@yahoo.com.br

Universidade Federal do Acre - CCN


Depto. de Ciencias da Natureza BR 364 Km 4
63.915-900 Rio Branco - AC
(68) 3901.2591

ALAGOAS

Prof. Joacy Vicente Ferreira


joacyferreira@ifal.edu.br

IFAL - Campus Macei


Instituto Federal de Alagoas
Rua Mizael Domingues, 75 Poo
57.020-600 Maceio - AL
Fone: (82) 2126-7000 / 7024 2126.7050 (fax)

AMAZONAS
www.oaq.ufam.edu.br

Prof. Paulo Rogrio da Costa Couceiro


couceiro@ufam.edu.br

Universidade Federal do Amazonas


Bloco 10 - Departamento de Qumica/ICE/UFAM
Setor Norte do Campus Universitrio Sen. Arthur
Virglio Filho
Av. Gal. Rodrigo Otvio Jordo Ramos, 6.200
69.077-000 Manaus - AM
Coroado
(92) 3305.2874 (telefax)

Prof. Lafaiete Almeida Cardoso


lafaiete@ufba.br

Universidade Federal da Bahia


Instituto de Qumica - Depto. Qui. Orgnica
Rua Baro de Geremoabo, s/n (Ondina)
40.170-115 Salvador - BA
(71) 3283.6813 - 3237.4117 (Fax)
Colgio Estadual da Bahia - Central
Av. Joana Anglica, Praa Carneiro Ribeiro
Salvador - BA
Fone: (71) 3237.4124 (LAQUAM)

oaq@ufam.edu.br

BAHIA
www.obaq.ufba.br

Profa. Sonilda Maria Teixeira da Silva


sonilda@laquam.qui.ufba.br

CEAR
http://www.necim.ufc.br

166 |

Prof. Leonilde Maria Cmara Jatahy


necim@bol.com.br
leojatahy@ig.com.br
Prof. Cludia Christina B. S. Carneiro

Universidade Federal do Cear


NECIM - Ncleo de Ensino de Cincias e
Matemtica
Av. da Universidade, 2470
60020-180 Fortaleza - Cear (85) 3366.7796

Ano Internacional de Qumica


Olimpada Brasileira
de Qumica 2011

Endereos
DISTRITO FEDERAL
www.unb.br/iq/pet

Profa. Elaine Rose Maia


emaia@unb.br
elainerm@terra.com.br
petqui@unb.br

Universidade de Braslia - Instituto de Qumica


Campus Universitrio Darcy Ribeiro 70910-970 ICC Sul - Asa Norte Cx.Postal: 04478
(61) 3107-3895 /2150 /2147 3273.4149 (FAX)

ESPRITO SANTO
www.cce.ufes.br/dqui/ocq-es

Prof. Carlos Vital Paixo de Melo


cvpaixao@npd.ufes.br

Universidade Federal do Esprito Santo


Departamento de Qumica - CCE
Av. Fernando Ferrari, 514 Goiabeiras
29.075-910 Vitria ES
(27) 3335.2486 - 3335.2826

GOIS
www.obqgoias.com.br

Prof. Renato Cndido da Silva


obqgoias@yahoo.com.br

Universidade Federal de Gois, Instituto de Qumica


Campos II Samambaia Bloco I
74001-970 Goiania, GO
Telefone: 62 3521.1167
Centro Federal de Educao Tecnolgica de Gois
UNED JATA
Rua Riachuelo, 2090 Setor Samuel Graham
75.800-000 Jata - GO
(64) 3632.8600
Centro Federal de Educao Tecnolgica de Gois
Campus Goinia
Rua 75, n 46, Centro.
74055-110. Goinia - GO
(62) 3227-2700

Prof. Carlos Czar da Silva


cefetjatai@yahoo.com.br

Prof. Hernane de Toledo Barcelos

MARANHO

Prof. Jean Carlo Antunes Catapreta


jcac889@hotmail.com

Universidade Federal do Maranho


Departamento de Qumica
Av. dos Portugueses, s/n Campus da Bacanga
65.080-040 So Luis - MA
Fone: (98) 3301.8280

MATO GROSSO

Prof. Luiz Both


bothluiz@ibest.com.br ou luiz_both@
hotmail.com

IFMT
Rua 28, Quadra 38, Casa 14, Jardim Universitrio
78.075-592 Cuiab - MT
Fone: 65 3653.9206 (IFMT) 3663.1374

MATO GROSSO DO SUL

Prof. Onofre Salgado Siqueira


onofre.s.siqueira@gmail.com

Universidade Federal do Mato Grosso do Sul


Centro de Cincias Exatas e Tecnologia Departamento de Qumica (Cidade universitria)
79.070-900 Campo Grande - MS
(67) 345.3556 - 345.3552 (FAX)

MINAS GERAIS
http://www.qui.ufmg.br/
omq/

Profa. Ana Luiza de Quadros


omq.ufmg@gmail.com

Universidade Federal de Minas Gerais


Departamento de Qumica
Instituto de Cincias Exatas (ICEx)
Av. Pres. Antnio Carlos, 6627 - Pampulha
31.270-901 Belo Horizonte - MG
(31) 3409.7558 Fax: (31) 3499.5700

Ano Internacional de Qumica



| 167

Programa Nacional
Olimpadas de Qumica

Endereos
PAR

Prof. Mrcio de Souza Farias


toraqk@yahoo.com.br
Profa. Patrcia da Luz
pdaluz@yahoo.com

PARABA

Prof. Vimario Simes Silva


vimario@deq.ufcg.edu.br

Universidade Federal de Campina Grande - Unidade


Acadmica de Engenharia Qumica
58.109-900 Campina Grande (83) 2101.1115

Prof. Francisco Dantas Filho

Universidade Estadual da Paraba-UEPB


Centro de Cincias Exatas e Sociais Aplicadas
Campus VII
58.500-000 Patos - PB 83 3421.1475
Universidade Federal de Campina Grande
Rua Srgio Moreira de Figueiredo, S/N
Bairro: Casas Populares
58.900-000 Cajazeiras - PB
(83) 3532-2040 3532.2063 3531.3940 (FAX)

Prof. Jos Estrela dos Santos


santosje@gmail.com

PARAN
http://www.oprq.daqbi.
ct.utfpr.edu.br/

Prof. Jos Carlos Colombo

PERNAMBUCO
olimpiadasdequimica.pe@
gmail.com

Antnio Carlos Pavo


pavao@ufpe.br

www.espacociencia.pe.gov.br/
index.php/atividade/quimica/

Associao Brasileira de Qumica


Rua O de Almeida, 490, Ed. Rotary, Sala 404
66017- 050 Belm - Par.
Fone/ Fax: (091) 3222.0870
91 8112.5382 TIM 91 87435407 (OI)
Fone: 91 8215.4852 (TIM)

colombo@utfpr.edu.br

Vice-coordenadora:
Profa. Karina Maia Batista de Oliveira
kmboliveira@gmail.com

Universidade Tecnolgica Federal do Paran


Av. 7 de setembro, 3165
80.230-010 Curitiba - PR
(41) 3310.4666 3310.4787 (fax)
Espao Cincia: 81 3383.5525 3185.5528
Memorial Arcoverde, Parque 2, S/N, Complexo de
Salgadinho Olinda -PE
Universidade Federal de Pernambuco - CCEN
Depto. de Qumica Fundamental - Cidade
Universitria
50.740-521 Recife - PE
Fone:(81) 2126.7415 2126.4646 (fax)

PIAU
http://www.ufpi.br/quimica/
opq/

Prof. Jos Milton Elias de Matos


jmematos@gmail.com

Universidade Federal do Piau


Depto de Qumica - SG2 CCN - Campus da Ininga
64.049-480 Teresina - PI
Fone: (86) 3215.5620 telefax

RIO DE JANEIRO
http://sites.google.com/site/
olimpiadadequimicarj/

Prof. Paulo Chagas


paulocha@gmail.com ou
luisquimica@globo.com

IFRJ - Instituto Federal de Cincia


e Tecnologia do Rio de Janeiro
Rua Senador Furtado, 121 Praa da Bandeira
20.270-021 Rio de Janeiro - RJ
(21) 3978. 5918 3567.0283 (Fax)

oqrj2006@gmail.com

168 |

Ano Internacional de Qumica


Olimpada Brasileira
de Qumica 2011

Endereos
RIO GRANDE DO NORTE

Prof. Kassio Michel Gomes de Lima


kassiolima@gmail.com
Profa. M de Ftima Vitria de
Moura
mfvmoura@quimica.ufrn.br

RIO GRANDE DO SUL


http://gaia.liberato.com.br/
olimpiada/index.php
Twitter: http://twitter.com/
oqdors
RONDNIA

Universidade Federal do Rio Grande do Norte


Centro de Cincias Exatas e da Terra
Depto de Qumica
Av. Senador Salgado Filho, 3000 Lagoa Nova
Campus Universitrio
59.072-970 Natal - RN
Fone: (84) 3215.3828 R: 222 3215.9224 (Fax)

Coordenao Colegiada:
Prof. Daniel Jacobus
Prof. Fvio Roberto Becker Dillio
Prof. Nair Cristina Muller
oqdors@gmail.com
quimica@liberato.com.br

Fundao Escola Tcnica Liberato Salzano V. da


Cunha
Rua Inconfidentes, 395 Primavera
93.340-140 - Novo Hamburgo - RS
(51) 3584.2027

Profa. Adaiane Spinelli


obqrondonia@gmail.com

Fundao Universidade Federal de Rondnia- UNIR


Nvleo de Cincias Exatas e da Terra - NCET
Departamento de Qumica BR 364 km 9
78.000-000 Porto Velho - RO
Fone (69) 2182.2193

Prof. Jandi Costa


jand_costa@hotmail.com
RORAIMA

Profa. Maria Lcia Taveira


taveiraml@ig.com.br

Universidade Federal de Roraima


Departamento de Qumica - Campus do Paricarana
69.301-270 Boa Vista - RR
Fone: (95) 621.3140/621.3137/623.1581 - 224.7302
(Fax)

SANTA CATARINA

Prof. Gilson Rocha Reynaldo


gilson@unisul.br
Jos Maximiliano Muller Netto
max@crq.org.br

Universidade do Sul de Santa Catarina - UNISUL


Av. Jos Accio Moreira, 787 - Caixa postal 370
88.704-900 Tubaro - SC Bairro Dehon Fone: (48)
621.3116 Fax (48) 621 3021

SO PAULO
http://allchemy.iq.usp.br

Prof. Ivano G. R. Gutz


gutz@iq.usp.br
abqsp@iq.usp.br

Instituto de Quimica da Universidade de So Paulo


Av. Prof. Lineu Prestes, 748 sala 1274
05.508-000 Sao Paulo - SP
(11) 3091.2150 (Telefax)

SERGIPE
www.osequim.hd1.com.br

Prof. Juvenal Carolino da Silva Filho


jcarolino@hotmail.com
Profa. Eliana Midori Sussuchi
esmidori@gmail.com

Universidade Federal de Sergipe - Depto. de


Qumica
Campus Prof. Alberto Carvalho
Av. Vereador Olimpio Grande, s/n Centro
49.500.000 Itabaiana - SE
Fone: (79) 3431.8216 3432.8200

TOCANTINS

Profa. Vanessa Viebrantz Oster


vanessaoster@gmail.com

Instituto Federal do Tocantins - Depto. de Qumica


AE 310 Sul, Av. LO 05 s/n
Plano diretor Sul Campus Universitrio
77.020-210 Palmas - TO
(63) 3233.1300 e 3233.1309 (Fax)

Ano Internacional de Qumica



| 169

Programa Nacional
Olimpadas de Qumica

Endereos
PATROCINADORES
ABICLOR - Associao Brasileira da Indstria de
lcalis e Cloro Derivados
www.abiclor.com.br

Rua Chedid Jafet, 222 Bloco C 4 andar Vila Olmpia


04.551-065 So Paulo SP
(11) 2148.4780 FAX 2148.4788

ABIQUIM - Associao Brasileira da Indstria


Qumica
www.abiquim.org.br

Rua Chedid Jafet, 222 Bloco C 4 andar Vila Olmpia


04.551-065 So Paulo SP
(11) 2148.4700 FAX 2148.4760

Banco do Nordeste do Brasil SA


www.bnb.gov.br

Av. Paranjana, 5700 Castelo


60.180-420 Fortaleza - CE
(85) 4005.3300
PROMOTORES

Universidade Federal do Cear


Pr-Reitoria de Extenso

Av. da Universidade, 2932 - Campus do Benfica


60.020 Fortaleza - CE
Fone: (85) 3366.7300

Universidade Estadual do Cear


Pr-Reitoria de Extenso

Av. Dede Brasil, 1700 Paranjana


60.740-000 Fortaleza- CE
Fone: (85) 3299.2555

Universidade Federal do Piau


Pr-Reitoria de Extenso

Campus da Ininga
64.049-550 Teresina - PI
Fone: (86) 3215.5692 Fax: (86) 215.5570

Fundao Cearense de Apoio ao Desenvolvimento


Cientfico e Tecnolgico

Av. Oliveira Paiva, 941


60.822-130 Fortaleza - CE
Fone: (85) 3101.2170 Fax: 3275.7862
APOIO

CNPq - Conselho Nacional de Desenvolvimento


Cientfico e Tecnolgico

SHIS QI 1 Conjunto B - Bloco D, 2 andar


Edifcio Santos Dumont, Lago Sul
71605-190 Braslia - DF
Fone: (61) 3211-9408

CAPES - Coordenao de Aperfeioamento de


Pessoal de Nvel Superior

Ministrio da Educao - MEC


Setor Bancrio Norte, Quadra 02, Bloco L, Lote 6, 4. Andar
70.040-020 - Braslia/DF

Associao Brasileira de Engenharia Qumica


abeq@abeq.org.br

Rua Lbero Bardar, 152 - 11 andar - Centro


01008-903 So Paulo - SP
Fone: (11) 3107-8747 ou Telefax: (11) 3104-4649

Associao Norte-Nordeste de Qumica


www.annq.com

Caixa Postal 167


69.301-970 Boa Vista RR
Fone: (95) 3224.7128

170 |

Ano Internacional de Qumica


Olimpada Brasileira
de Qumica 2011

Endereos
Espao Cincia
www.espacociencia.pe.gov.br

Complexo de Salgadinho s/n


53.111-970 Olinda PE
Fone: (81) 3301-6139

Fundao Estudar
http://www.estudar.org.br

http://twitter.com/fundacaoestudar
Website: http://www.estudar.org.br
Tel: +55 11 7124-4187

Associao Brasileira de Qumica


www.abq.org.br

Parque 2

REALIZADOR
Av. Presidente Vargas, 633 sala 2208
20071-004 Rio de Janeiro RJ
Telefone: 21 2224-4480 Fax: 21 2224-6881
E-mail: abqrj@alternex.com.br

Ano Internacional de Qumica



| 171

Você também pode gostar